You are on page 1of 251

APPLIED MATHEMATICS I MODULE

APPLIED MATHEMATICS I
(MATHS 102)

MODULE

FDRE Ministry of Education


Technical Vocational Education and Training Institute

August 2017

1
FTVETI
APPLIED MATHEMATICS I MODULE

APPLIED MATHEMATICS I
(MATHS 102)

WRITTEN BY:

MATHEMATICS DEPARTMENT MEMBERS

2
FTVETI
APPLIED MATHEMATICS I MODULE

Table of Contents

Chapter One: Vectors ..........................................................................................................6

1.1 Definition of points in n-space .............................................................................................. 6

1.2 Vectors in n-space, Geometric interpretation in 2 and 3- spaces ........................................ 8

1.3 Scalar product and norm of vector, orthogonal projection, and direction cosines 12
1.4 Vector product ...................................................................................................................19

1.5 Application on area and volume ........................................................................................21

1.6. Lines and planes in space ...................................................................................................22

Chapter Two: Matrices and Determinants .........................................................................29

2.1 Definition of a matrix and basic operations.......................................................................30

2.1.1 Concept of matrix ..................................................................................................................30

2.1.2 Operation on matrices ..........................................................................................................32

2.1.3 Properties of matrix addition and scalar multiplication........................................................34

2.2 Product of matrices and some algebraic properties, transpose .......................................35

2.2.1 Matrix multiplication .............................................................................................................35

2.2.1 Properties of matrix operation ..............................................................................................38

2.2.2 Transpose of a matrix and its properties ..............................................................................40

2.3 Types of matrices ...............................................................................................................42

2.4 Elementary row operations ..............................................................................................47

2.4.1 Row echelon form of a matrix ..............................................................................................47

2.5 Rank of a matrix ................................................................................................................57

2.6 Inverse of a matrix and its properties ................................................................................59

2.7 Determinants and its properties ........................................................................................65

2.7.1 Determinant of a matrix ........................................................................................................65

2.7.2 Properties of a determinant ..................................................................................................72

3
FTVETI
APPLIED MATHEMATICS I MODULE

2.8 Determinant method of finding inverse matrices .............................................................78

2.9 Solving System of linear equations ....................................................................................80

2.9.1 The Gaussian elimination and Gauss-Jordan Elimination ....................................... 83


2.9.2 Cramer’s rule ........................................................................................................... 85
2.9.3 Inverse matrix method ............................................................................................ 87
Chapter Three: Limit and Continuity.................................................................................89

3.1 Definitions of limit .............................................................................................................90

3.2 Examples on limits .............................................................................................................91

3.3 One-sided limit ...................................................................................................................98

3.4 Infinite limits and infinite limits at infinity .......................................................................100

3.5 Basic limit theorems.........................................................................................................103

3.6 Continuity of a function and the Mean Value Theorem .................................................108

3.6.1 One-sided continuity ...........................................................................................................110

3.6.2 Continuity on an interval ....................................................................................................111

Chapter Four: Derivatives ................................................................................................115

4.1 The tangent lines..............................................................................................................115

4.2 Definition of derivatives...................................................................................................117

4.3 Basic differentiation rules ................................................................................................124

4.4 The chain rule...................................................................................................................125

4.5 Higher order derivatives ..................................................................................................127

4.6 Implicit differentiation .....................................................................................................129

4.7 Derivatives of inverse trigonometric functions ..............................................................135

4.8 Inverse hyperbolic functions............................................................................................138

Chapter Five: Applications of Derivatives ......................................................................142

5.1 Extrema of a function.......................................................................................................143

5.2 The Rolle’s Theorem and the Mean-Value Theorem .......................................................145

4
FTVETI
APPLIED MATHEMATICS I MODULE

5.3 Applications of the Mean Value Theorem .......................................................................148

5.4 First and second derivative test .......................................................................................149

5.5 Graph sketching ..............................................................................................................158

5.6 Indeterminate forms and L ’Hôpital’s rule ...................................................................162

Chapter Six: Integrations ................................................................................................174

6.1 Anti derivatives; indefinite integrals ................................................................................176

6.2 Partitions .........................................................................................................................183

6.2.1 Partitions, Upper and Lower sum, Riemann Sums .............................................................183

6.2.2 Upper and Lower sum integrals ..........................................................................................185

6.2.3 Riemann Sums ....................................................................................................................190

6.3 Definite integral ...............................................................................................................194

6.4 Fundamental theorem of calculus ...................................................................................198

6.5.Technics of integration ...................................................................................................201

6.5.1 Integration by substitution ..................................................................................................202

6.5.2 Integration by parts ............................................................................................................206

6.5.3 Integration by partial fractions ...........................................................................................213

6.6 .Trigonometric integrals ...................................................................................................218

6.7.Integration by Trigonometric substitutions .....................................................................227

6.8 Improper integrals ..........................................................................................................233

6.8.1 Integrals over unbounded intervals ....................................................................................233

6.8.2 Integrals with unbounded integrands .................................................................................236

Chapter Seven: Application of the integral ................................................................................241

7..1 Area .......................................................................................................................................241

7..2 Volume ..................................................................................................................................244

5
FTVETI
APPLIED MATHEMATICS I MODULE

CHAPTER 1
VECTORS
Introduction
Certain Physical quantities such as mass, area, density, volume, etc., that possess only
magnitude are called scalars. On the other hand, there are physical quantities such as force,
displacement, velocity, acceleration, etc that has both magnitude and direction. Such
quantities are called vectors.
The concept of a vector is essential for the next two chapters. It provides the foundation
and geometric motivation for everything that follows. Hence the properties of vectors, both
algebraic and geometric, will be discussed in this unit.
Objectives
At the end of this chapter students will be able to:-
 Define vector.
 List properties of vectors.
 Add vectors and multiply a vector by a scalar.
 Determine the length and direction of vectors.
 List properties of scalar product and vector product.
 Find the scalar and vector product of vectors.
 Find the volume of a parallelepiped generated by three vectors.
1.1 Definition of Points in n-space
We know that, once a unit length is selected, a number x can be used to represent a point
on a line. A pair of numbers (i.e. a couple of numbers) (x, y) can be used to represent a
point in the plane. A triple of numbers (x, y, z) can be used to represent a point in space.
The following pictures illustrate these representations:

6
FTVETI
APPLIED MATHEMATICS I MODULE

We can say that a single number represents a point in 1-space (A), a couple represents a
point in 2-space (B) and a triple represents a point in 3-space (C).
Although we cannot draw a picture to go further, we can say that a quadruple of numbers
(𝑥, 𝑦 𝑧, 𝑤) or ( 𝑥1 , 𝑥2 , 𝑥3 , 𝑥4 ) represent a point in 4-sapce.

Activity 1.1
Define a point in n-space, where n is a positive integer.

The set of all points in n-space is represented by . For the point 𝑋 in , represented by n-tuple of real
numbers (𝑥1 , 𝑥2 , … , 𝑥𝑛 ), the numbers 𝑥1 , 𝑥2 , … , 𝑥𝑛 are called the coordinates of 𝑋.

Example 1: The space we live in can be considered as a 3 space. After selecting an origin
and a coordinate system, we can describe the position of a point (body, particle, etc.) by 3
coordinates. We can extend this space to a 4 dimensional space, with a fourth coordinate,
for example, time. If you select the origin of the time axis as the birth of Christ, how do we
describe a body with negative time co-ordinate? What if the birth of the earth is taken as
the origin of time?
If A = ( 𝑎1 , 𝑎2 , … , 𝑎𝑛 ) and B = ( 𝑏1 , 𝑏, … , 𝑏𝑛 ) are points in the same space  n , and if c is a
real number then
I. A and B are equal (or represent the same point) if , 𝑎1 =𝑏1 , 𝑎2 =𝑏2 , … and 𝑎𝑛 =𝑏𝑛

7
FTVETI
APPLIED MATHEMATICS I MODULE

II. A + B, A – B and cA are defined to the points whose coordinates are (𝑎1 +𝑏1, 𝑎2
=+𝑏2 , …, 𝑎𝑛 +𝑏𝑛 ), (𝑎1 −𝑏1, 𝑎2 −𝑏2 , …, 𝑎𝑛 -𝑏𝑛 ) and ( 𝑐𝑎1 , 𝑐𝑎2 , … , 𝑐𝑎𝑛 ),
respectively.
Example 2: 1) Let 𝐴 = (1, 2),𝐵 = (−3, 4),then𝐴 + 𝐵 = (−2,6), 𝐴 − 𝐵 = (4, −2),

and −3𝐴 = (−3, −6)


2) Let 𝑋 = (1, 0, 𝜋, 4), 𝑌 = (2, 4, −2𝜋, −6), then 2𝑋 + 𝑌 = (4, 4, 0, 2)
and 𝑋 − (1/2) 𝑌 = (0, −2, 2𝜋, 7).
1.2. Vectors in n-space, Geometric interpretation in 2 and 3- spaces
Every pair of distinct points A and B in  n determines a directed line segment with initial
point at A and terminal point at B. We call such a directed line segment a vector and denote
⃗⃗⃗⃗⃗ . The length of the line segment is the magnitude of the vector. Although 𝐴𝐴
it by𝐴𝐵 ⃗⃗⃗⃗⃗ has

zero length, and strictly speaking, no direction, it is convenient to view it as a vector. It is

called a zero or a null vector. It is often denoted by O .


⃗⃗⃗⃗⃗ and CD
Two vectors AB ⃗⃗⃗⃗⃗ will be considered to be equal (or equivalence), AB
⃗⃗⃗⃗⃗ = CD
⃗⃗⃗⃗⃗ if they

have the same magnitude and direction.


Notice that the definition of equality of two vectors does not require that the vectors have
the same initial and terminal points. Rather it suggests that we can move vectors freely
provided we make no change in magnitude and direction.

Activity 1.2
Let (a1, a2) be the coordinate representation of A and let (b1, b2) be that

of B. Let P be the point (b1-a1, b2 –a2). If O is the origin, is in the

direction of ?

Is length of equal to the length of ? Is ⃗⃗⃗⃗⃗ ?


=𝐴𝐵

8
FTVETI
APPLIED MATHEMATICS I MODULE

If your answer for the above questions is yes, then we can conclude that any vector V=
⃗⃗⃗⃗⃗ in the plane is a vector OP with initial point at the origin. This is the only vector whose
𝐴𝐵

initial point is the origin and P = B – A, which is equal to AB . Moreover, V = OP is

uniquely determined by its terminal point P. If P = (x, y), then we shall write V = (x, y)
and refer to it as the coordinate representation of V relative to the chosen coordinate system.
In view of this, we shall call (x, y) either a point or a vector, depending on the interpretation
⃗⃗⃗⃗⃗ , then we can write V = B – A. In view of this two
which we have in mind. So if V =𝐴𝐵
⃗⃗⃗⃗⃗ , and CD are equal (or equivalence) if B – A = D – C.
vectors𝐴𝐵

Example 3: If P = (1, 3), Q = (-1,0), R = (0, -1) and S = (-2, -4) then OP  RS
As numbers can be added subtracted and multiplies, vectors can be combined in the
following ways.
Let 𝐴 = (𝑎1 , 𝑎2 ) and 𝐵 = (𝑏1 , 𝑏2 ) vectors and 𝑡 be a real number. The sum
𝐴 + 𝐵 = (𝑎1 + 𝑏1 , 𝑎2 + 𝑏2 )
The difference 𝐴 − 𝐵 = (𝑎1 − 𝑏1 , 𝑎2 − 𝑏2 )
The scalar multiple 𝑡𝐴 = (𝑡𝑎1 , 𝑡𝑎2 )
The geometric interpretation of the above vector operations is that A + B is a vector
obtained by placing the initial point of B on the terminal point of A.
If 𝑡 > 0, then 𝑡𝐴 is a vector in the direction of A. What about if 𝑡 < 0? 𝐴 and 𝑡𝐴 are
said to have opposite direction. (See figure a and b)

tA
for t > 0 Fig. a for t < 0 Fig. b
We can extend the above notions to vectors in n but the geometric interpretations for
for 𝑛 > 3 are difficult. Hence we focus on algebraic aspects of vectors.

9
FTVETI
APPLIED MATHEMATICS I MODULE

If 𝐴 = (𝑎1 , 𝑎2 , … , 𝑎𝑛 ) and 𝐵 = (𝑏1 , 𝑏2 , … , 𝑏𝑛 ) are vectors in  n and if 𝑡 is any real


number, then
A  B  a1  b1 , a 2  b2 ,.., a n  bn 

𝑡𝐴 = (𝑡𝑎1 , 𝑡 𝑎2 , … , 𝑡𝑎𝑛 )
A and B are said to be parallel vectors iff either A = tB for some real number t or B =
tA. In other words, A and B are parallel iff one is a scalar multiple of the other.

Using the arrow notation PQ and RS are parallel if and only if


𝑄 – 𝑃 = 𝑡(𝑆 – 𝑅) 𝑜𝑟 𝑆 – 𝑅 = 𝑡(𝑄 – 𝑃) for some real number 𝑡.

Activity 1.3
1) Let A = (6, -2, 4). Find two vectors C and D which are parallel to A.

Are C and D also parallel to each other?

2) Let P = (3,7), Q = (-4,2), R = (5,1), S = (-16,-14).

Is parallel to ?

Using the above definitions and applying the associative and commutative properties of
real numbers, one can prove the following theorem.

Let A, B and C be any members of  , and let m and n be any real


n
Theorem 1.1:
numbers. Then
a) m(nA)  (mn) A Associativity
b) (A  B)  C  A  (B  C)
c) A  B  B  A Commutativity
d) (m  n) A  mA  nA
Distributive property
e) m( A  B)  mA  mB
f) 0.A  O
g) A  O  A
Proof c) A  B  (a1 , a 2 ,..., a n )  (b1 , b 2 ,..., b n )

10
FTVETI
APPLIED MATHEMATICS I MODULE

= (a1  b1 , a 2  b 2 ,..., an  bn )
= (b1  a1 , b 2  a 2 ,..., bn  an )
= (b1 , b 2 , ..., b n )  (a1 , a 2 ,..., a n )
= BA

Example 4:
A boat captain wants to travel due south at 40 knots.
If the current is moving northwest at 16 knots, in
what direction and magnitude should he works the
engine?

Solution: we have u = v + w where u corresponds to the engine’s vector and w corresponds

to the velocity of the current. We have u = -40j and w =  8 2 i + 8 2 j

Hence v = u – w = -40j – (  8 2 i + 8 2 j) = 8 2 i – (40 + 8 2 )j. The magnitude is

(8 2) 2  (40  8 2 ) 2 = 52.5.

 40  8 2 
The direction is arctan   = -1.35 radians

 8 2 

Activity 1.4
1. Given three vectors A = (1, 1, 1), B = (-1, 2, 3) and C = (0, 3, 4), find
a. A+B c. A+B – C
b. 2A – B d. A – 3B + 10C
2. Determine whether and can be found to satisfy the vector equations

a. (2, 1, 0) = (-2, 0, 2) + (1, 1, 1)

11
FTVETI
APPLIED MATHEMATICS I MODULE

The Distance Formula


The distance formula is derived from the three dimensional version of the Pythagorean
theorem, which is displayed below,

x2  y2  z2

x z x2  y2  z2

y
The distance between two points (x1, y1, z1) and (x2, y2, z2) is given by:

D  ( x2  x1 ) 2  ( y2  y1 ) 2  ( z 2  z1 ) 2
1.3 Scalar product and norm of vector, orthogonal projection, and direction cosines

Let A = (a1, a2,…,an) and B = (b1, b2, …, bn) be two vectors. The scalar product of A and
B is the number A.B defined by

A.B  a1b1  a 2b 2  ...  anbn

Note: The scalar product is also called a dot product or inner product.

Example 1:

1) Given A = (3, -1) and B = (2, 3), then A.B = 3

2) i.j  O , where i = (1, 0,0) and j = (0,1,0).

The scalar product satisfies many of the laws that hold for real numbers. The basic ones
are:

a) A.B  B.A

b) t(A.B)  (tA).B  A.(tB)

12
FTVETI
APPLIED MATHEMATICS I MODULE

c) (A  B).C  A.C  B.C

d) If A  O is the zero vector, then A.A  0 , and otherwise A.A  0

Proof: .the above properties.

Example 2: Given 𝐴 = (3, 2, −1) and 𝐵 = (2,0, 3), and 𝐶 = (1, −1,1), then
a. 𝐴. 𝐵 = 𝐵. 𝐴 = 3
b. 2(𝐴. 𝐵) = 6
c. (𝐴 + 𝐵). 𝐶 = 𝐴. 𝐶 + 𝐵. 𝐶 = 5

Activity 1.5

1. Find A.A, B.B, and C.C. Are all positive values?

2. Given the vectors A = (1,-1,2), B = (-2,0,2), C = (3,2,1). Evaluate

a. A.C b. B.C c. (A + B).C d. A.(2B + 3C) e. (A.B)C

The length, or norm or magnitude of vector A = (a1, a2, …, an), denoted by ||A||, can be
expressed in terms of the scalar product. By definition

 a12  a 22  ...  an2


2
A

and A.A  (a1 , a2 ,..., an ).(a1 , a2 ..., an )  a12  a22  ...  an2
Thus, the norm of the vector A is the number:

||A|| = a12  a 22  ...  an2

Example 3: If A = (-1, 2, 3) then A  1  4  9  14

Note: 1. A unit vector is a vector having norm or length 1.


2. A  0 A0

13
FTVETI
APPLIED MATHEMATICS I MODULE

 1 1 1 
Example 4: If A   , ,  , then A  (1 / 3)  (1 / 3)  (1 / 3)  1
 3 3 3

Any non-zero vector can be fully represented by providing its magnitude and a unit

vector along its direction. Let A be a unit vector in the direction of A. Then

 A
A
A

Example 5: Given a vector A = (1, 1, 1). Find a unit vector in the direction of A.
Solution:
A  1  1  1  3 , then the unit vector in the direction of A is:
 A (1,1,1)  1 1 1  1
A    , ,   (1,1,1)
A 3  3 3 3 3

Activity 1.6
1. Given three vectors A = (1, 1, 1), B = (-1, 2, 3) and C = (0, 3, 4), find the unit vector in the direction of A+B
–C

2. The vectors i (1, 0, 0), j = (0, 1, 0) and k = (0, 0, 1) are unit vectors in the direction of positive x, y and z
axis, respectively. Find a unit vector in the direction of A = (-1, 2, 3).

Let A, B be two n-tuples of vectors. We define the distance between A and B to be


A  B  ( A  B).( A  B)

Note: Let A be any vector and x   , then A   A and xA  x A

The following theorem gives us a geometric interpretation for the scalar product.

Theorem 1.2: Let A  (a1 , a 2 , a3 ) and B  (b1 , b 2 , b 3 )

be non-zero vectors and let  be

14
FTVETI
APPLIED MATHEMATICS I MODULE

the angle between A and B (0    ) .

Then A.B  A B cos 

Proof: Consider a triangle formed by A, B and A  B

A  B  A  B  2 A B cos 
2 2 2
(Why?)

 ( a1  b1 )2  ( a 2  b2 )2  ( a3  b3 )2  a12  a 22  a32  b12  b22  b32  2 A B cos 


After cancellation, we get, a1b1  a 2b 2  a3b 3  A B cos 

 A.B  A B cos 

Activity 1.7
Given two non-zero vectors A and B, how do you find the angle between them? Take, for example,
A = (2, -1, 2), B = (1, -1, 0) and find the angle between them.

Two non-zero vectors are said to be orthogonal (Perpendicular) if the angle between

them is .
2

Note: Two non-zero vectors A and B are said to be orthogonal (Perpendicular) if A.B= 0.

Activity 1.8
1. Which of the following pairs of vectors are perpendicular?
a. (1,-1,1), and (2,1,5) b. (1,-1,1), and (2,3,1)

c. (-5,2,7), and (3,-1,2) d. ( ,2,1), and (2,- ,0)

2. Suppose A.B = A.C, what can you deduce about A, B and C?

3. Show that if and only if A.B=0.

4. Let A1, A2, …, Ar be non-zero vectors such that Ai.Aj = o if .

15
FTVETI
APPLIED MATHEMATICS I MODULE

Te following are two of the important inequalities

Theorem 1.3: Let A and B be vectors. Then

a) A.B  A B (Schwarz inequality)

b) A  B  A  B (Triangle inequality)

Proof: a) If one of A or B is a zero vector then both sides of the inequality are equal
to 0. Suppose both A and B are non-zero.

From A.B  A B cos  ,

A.B  A B | cos  | Which implies A.B  A B (Why?)

 V.V ,
2
b) From the fact that V

2
AB  ( A  B) . ( A  B)
 A.A  2A.B  B.B
2 2
 A  2A.B  B
2 2
 A  2 A.B  B (why ?)
2 2
 A 2 A B  B (why ?)
 A  B 
2

By taking the square roots,

AB  A  B

Remark: The inequalities of Theorem 1.3.2 hold true also for any vectors A and B in Rn.

We now define the component of a vector in the direction of another vector.

Let A and B be two non-zero vectors.

16
FTVETI
APPLIED MATHEMATICS I MODULE

Let  be the angle between them.

From the terminal point of B drop a perpendicular to the

Line containing A.

The vector OD has magnitude OD  B cos  , 0     , and its direction is either

the same as that of A or opposite to it depending on whether  is acute or obtuse.

1
Since A is a unit vector in the direction of A and since OD has magnitude B cos 
A

and is in the direction of A or opposite to A, we can write

 A.B 
⃗⃗⃗⃗⃗⃗ 
𝑂𝐷  B cos   1 A ⃗⃗⃗⃗⃗⃗
Or 𝑂𝐷   2 A
 A 
(why?)
A
 

This vector is called the projection of B along A and is denoted by Proj A B .

 A.B 
That is, Pr oj AB   2  A
 A 
 

A.B
We call t  2
the component of B along A.
A

Note: 1. Pr oj AB is parallel to A. That is Pr oj AB  tA , for some t  

2. B - Pr oj AB is orthogonal (perpendicular) to A.

 1
Example 6: Let A = (3, -1, -2) and B =  2,  3,  , then
 2

 
Proj A B =  A.B2 A   6  3  1 A   12 ,  4 ,  8 
 A   9 1 4  7 7 7 
 

17
FTVETI
APPLIED MATHEMATICS I MODULE

 
   
Proj B A =  A.B   6  3  1  B   64 ,  96 , 16 
 B 
2   1   53 53 53 
 B  49 
 4

Activity 1.9
One application of projections of vector arises in the definition of the work done by a force on
a moving body. Find another application.

Given a non-zero vector u  (u1 , u 2 , u3 ) in  3 . The direction cosines of the vector u are:

u1 u u
Cos  , Cos  2 , Cos  3
u u u

Where the direction angles,  ,  , and  are the angles that the vector makes with the
positive x, y, and z-axes respectively.

Remark: Cos 2  Cos 2   Cos 2  1

Example 7: Let u = (1, -2, 3). Find the direction cosines of u.

Solution: Since u  u.u  (1,2,3).(1,2,3)  1  4  9  14

Thus the direction cosines are: Cos  u1  1 , Cos  u 2   2 , and Cos  u 3  3


u 14 u 14 u 14

Activity 1.10

Is ?

18
FTVETI
APPLIED MATHEMATICS I MODULE

1.4 The Vector product


The second type of product of two vectors is the cross product. Unlike the dot product, the
cross product of two vectors is a vector.
Definition 1.4.1 the cross product (or vector product) 𝐴 𝑥 𝐵 of two vectors
𝐴 = (𝑎1 , 𝑎2 , 𝑎3 ) and 𝐵 = (𝑏1 , 𝑏2 , 𝑏3 ) is defined by
𝑨 𝒙 𝑩 = (𝑎2 𝑏3 − 𝑎3 𝑏2 , 𝑎3 𝑏1 − 𝑎1 𝑏3 , 𝑎1 𝑏2 − 𝑎2 𝑏1

Note that the cross product is defined in 


3

Example 1: Let A = (2, -1, 3) and B = (-1, -2, 4)


A x B = (-4 + 6, -3 – 8, -4 –1) = (2, -11, -5)

Question 1.1: Find B x A.

The following are some of the basic properties of cross product


Theorem 1.4: for vectors A, B and C,
1) 𝐴 𝑥 𝐵 = −(𝐵 𝑥 𝐴)
2) 𝐴 𝑥 𝐴 = 𝑂
3) 𝑡𝐴 𝑥 𝐵 = 𝑡(𝐴 𝑥 𝐵) = 𝐴 𝑥 (𝑡𝐵), 𝑡 

 A B  A.B
2 2 2 2
4) AxB

5) 𝐶. (𝐴 𝑥 𝐵) = 𝐵. (𝐶 𝑥 𝐴) = 𝐴. (𝐵 𝑥 𝐶)
6) (𝐴 + 𝐵) 𝑥 𝐶 = (𝐴 𝑥 𝐶) + (𝐵 𝑥 𝐶)
7) 𝐶 𝑥 (𝐴 + 𝐵) = (𝐶 𝑥 𝐴) + (𝐶 𝑥 𝐵)
8) 𝐴. (𝐴 𝑥 𝐵) = 0 and 𝐵. (𝐴 𝑥 𝐵) = 0 (that is, 𝐴𝑥𝐵 is perpendicular to
both A and B.)
9) (𝐴𝑥𝐵) 𝑥 𝐶 = (𝐴. 𝐶) 𝐵 – (𝐵. 𝐶)𝐴
Proof: The following is the proof for 1, 2 and 8. The rest are left as an exercise
1) From the definition of cross product,
𝐴 𝑥 𝐵 = (𝑎2 𝑏3 − 𝑎3 𝑏2 , 𝑎3 𝑏1 − 𝑎1 𝑏3 , 𝑎1 𝑏2 − 𝑎2 𝑏1 )
For 𝐵 𝑥 𝐴, interchange A and B to obtain
𝐵 × 𝐴 = (𝑏2 𝑎3 − 𝑏3 𝑎2 , 𝑏3 𝑎1 − 𝑏1 𝑎3 , 𝑏1 𝑎2 − 𝑏2 𝑎1 )

19
FTVETI
APPLIED MATHEMATICS I MODULE

= (𝑎3 𝑏2 − 𝑎2 𝑏3 , 𝑎1 𝑏3 − 𝑎3 𝑏1 , 𝑎2 𝑏1 − 𝑎1 𝑏2 )
= −(𝐴 × 𝐵)
2) 𝐴 𝑥 𝐴 = (𝑎2 𝑎3 – 𝑎3 𝑎2 , 𝑎3 𝑎1 − 𝑎1 𝑎3 , 𝑎1 𝑎2 − 𝑎2 𝑎1 )
= (0, 0, 0)
8) Setting 𝐶 = 𝐴 in 5) yields
𝐴 . (𝐴 𝑥 𝐵) = 𝐵 . (𝐴 𝑥 𝐴)
= 𝐵 .0 (why?)
= 0
By setting 𝐶 = 𝐵 in 5),
𝐵 . (𝐴 𝑥 𝐵) = 𝐴 . (𝐵 𝑥 𝐵)
= 𝐴 .0 = 0
This shows that for non-zero vectors A and B, the cross product A x B is orthogonal to
both A and B.
A x B = (-4 + 6, -3 – 8, -4 –1) = (2, -11, -5)

Activity 1.11

1. Are the usual commutative and associative laws valid? i.e. for any vectors A, B and C

in , is A x B = B x A?
Is A x (B x C) = (A x B) x C?

2. Let A = (2,1,0), B = (2,-1,1) and C = (0,1,1). Find


a. AxB b. (AxB)xC c. (A.C)B – (B.C)A d. BxC e. Ax(BxC)

From 4) of theorem 1.4.1, we derive an important formula for the norm of the cross product.

 A B  A.B
2 2 2 2
AxB

2 2 2 2
 A B  A B cos 2  (  Is the angle between A and B)

 A
2

B 1  cos 2 
2

 A B sin 2 
2 2

20
FTVETI
APPLIED MATHEMATICS I MODULE

 AxB  A B sin  (For 0    , sin is non- negative)

A x B = (-4 + 6, -3 – 8, -4 –1) = (2, -11, -5)

Activity 1.12

1. For the unit vectors and , find and . What is

2. If A and B are parallel, what is A B?

3. If A and B are orthogonal, What is

4. Find a unit vector perpendicular to both A = (2,-3,1) and B = (1,2,-4).


5. Prove that (A – B)x(A + B) = 2(AxB).

1.5. Application on area and volume

Let u and v be vectors and consider the parallelogram that the two vectors make. Then

u  v = Area of the Parallelogram

The direction of uxv is a right angle to the parallelogram that follows the right hand rule.

To find the volume of the parallelepiped spanned by three vectors u, v, and w, we find the
triple product:

u.(vxw) = Volume

This can be found by computing the determinant of the three vectors:

u1 v1 w1
u2 v2 w2 = u1 (v2 w3  v3 w2 )  v1 (u 2 w3  u3 w2 )  w1 (u 2 v3  u3 v2 )
u3 v3 w3

21
FTVETI
APPLIED MATHEMATICS I MODULE

Example 2: 1. Find the area of the parallelogram which is formed by the two vectors u=
(1, 3, 2) and v= (-2, 1, 3).

2. Find the volume of the parallelepiped spanned by the vectors

u = (3, -2, -1), v = (1, 3, 2), and w = (-2, 1, 3).

Solution: 1. The area of the parallelogram is given by:

u  v  (1,3,2)  (2,1,3)  (7,7,7)  147

2. The volume of the parallelepiped spanned by the three vectors is:

u.(vxw)  (3,2,1).(1,3,2)  (2,1,3)  (3,2,1).(7,7,7)  28

Activity 1.13

Find the area of the triangle having vertices at u = (3, -2, -1),

v = (1, 3, 2), and w = (-2, 1, 3).

1.6 Lines and planes


Question 1.2: Given two vectors A and B with B  0, consider vectors of the form A +tB,
where t varies over all real numbers. The vector A + 2B, for example is as
1 3
shown in the figure. Show the vectors A + tB for t = 0, 1, 3, , -1, -2,
2 2
, what kind of figure is generated by these vectors? i.e. what is the collection
of all points which are terminal points of A + tB?

22
FTVETI
APPLIED MATHEMATICS I MODULE

Definition 1.6.1: A line  is any set of the form {p : p  A  tB, t } where B is


assumed to be a non-zero vector and A is a fixed point on the line.

Note that if (x, y, z) is on line  and if A = (a1, a2, a3) and B = (b1, b2, b3 then
(x, y, z) = (a1, a2, a3) + t(b1, b2, b3) for some real number t.

Activity 1.14
Is point A on ? Is B parallel to
a vector formed by any two points of ?

P = A + tB is a vector equation of a line through A

Example 1: Find equation of a line through 𝑝1 = (0, 1, 2) and 𝑝2 = (−1, 1, 1).


Solution: We need a point A on the line and a vector B parallel to the vector formed by
two point of the line.
Take 𝐴 = 𝑝1 and 𝐵 = 𝑝2 − 𝑝1 . Then
𝐴 + 𝑡 𝐵 = (0, 1, 2) + 𝑡 (−1, 0, −1)
(𝑥, 𝑦, 𝑧) = (0, 1, 2) + 𝑡 (−1, 0, −1)

23
FTVETI
APPLIED MATHEMATICS I MODULE

is equation of the line. By giving distinct values for 𝑡 we will obtain distinct points on the
line. Find some of the points.
Note: The equation of a line passing through points A and B is given by:
𝑃 = 𝐴 + 𝑡(𝐵 – 𝐴) or 𝑃 = (1 – 𝑡)𝐴 + 𝐵, t  
Question 1.3: Let the line L1 passes through the points (5,1,7) and (6,0,8) and the line L2
passes through the points (3,1,3) and (1,3,  ) . Find the value of  for which the two lines
intersect.
Suppose 𝑃 = (𝑥, 𝑦, 𝑧) is a point on line  through 𝐴 = (𝑎1, 𝑎2 , 𝑎3 ) in the direction of
𝐵 = (𝑏1 , 𝑏2 , 𝑏3 ). Then
𝑝 = 𝐴 + 𝑡𝐵  (𝑥, 𝑦, 𝑧) = (𝑎1 , 𝑎2 , 𝑎3 ) + 𝑡(𝑏1 , 𝑏2 , 𝑏3 ) or equivalently
𝑥 = 𝑎1 + 𝑏1 𝑡
𝑦 = 𝑎2 + 𝑏2 𝑡
𝑧 = 𝑎3 + 𝑏3 𝑡
These equations are parametric equation of a line and t is called a parameter.

Activity 1.15

1) Find the parametric equation of a line that contains (2, -1, 1) and is parallel to the vector (3, , 0).

2) From the parametric equation of a line in , derive the equation

It is called standard form of equation of a line.

If the line is on a plane show that the standard form reduces to an equation of the form
𝑦 = 𝑚𝑥 + 𝑐.
Two lines  and m given by 𝐴1 + 𝑡𝐵1 and 𝐴2 + 𝑡𝐵2 are said to be parallel if 𝐵1 and 𝐵2
are parallel. That is the vectors 𝑃1 − 𝑄1 and𝑃2 − 𝑄2 are parallel for any two points 𝑃1 , 𝑄1of
 and 𝑃2 , 𝑄2 of m.

24
FTVETI
APPLIED MATHEMATICS I MODULE

Let  be a line through A in the direction of B ( B  0) . Consider the distance between 


and the origin. This distance is the minimum of the lengths of all vectors with initial point
the origin and terminal point on  . That is, minimum of A  tB for any real number t.

Now put f (t )  A  tB
2

This is a quadratic function whose graph opens upward: f (t )  A  2t ( A.B)  t 2 B


2 2

 2 A.B  A.B
So it has minimum at: t  2
 2
2B B

Let Po be a point and N be a non-zero


vector. We define the plane passing
through Po to perpendicular to N to be the
collection of all points P such that the

vector p o p is perpendicular N.
According to our definition, if P is any point on the plane through P0 and perpendicular to
N, then N . p o p  0 or N . ( p  p0 )  0

Question 1.4: Starting from the equation N . p o p  0 , show that equation of a plane
through point 𝑃0 = (𝑎0 , 𝑏0 , 𝑐0 ) perpendicular to 𝑁 = (𝑎, 𝑏, 𝑐) is

𝒂𝒙 + 𝒃𝒚 + 𝒄𝒛 = 𝒅 where 𝑑 = 𝑎𝑥0 + 𝑏𝑦0 + 𝑐𝑧0


This equation can be written as N . P  d . The vector N is said to be normal to the plane.
Hence a plane is any set of the form {P: N.P = d}. Where N is a given non-zero vector and
d is a given number.
Example 2: Find an Equation of the plane that contains point (-2, 4, 5) and that is normal
to (7, 0, -6).
Solution: The equation of the plane is given by 7(x+2)+0(y-4)-6(z-5) = 0 or 7x- 6z = -44
Question 1.5: Does this plane intersect the y-axis?

25
FTVETI
APPLIED MATHEMATICS I MODULE

Two planes in 3 spaces are said to be parallel if their normal vectors are parallel. They are
said to be perpendicular if their normal vectors are perpendicular. The angle between two
planes is defined to be the angle between their normal vectors.
These equations are parametric equation of a line and t is called a parameter.

Activity 1.16
1. A plane passes through (-1, 2, 3) and is perpendicular to the y-axis. What is the
equation?
2. Consider the planes x + 2y - 3z = 2 and 15x - 9y – z = 2. Are they parallel or
perpendicular planes? Or neither parallel nor perpendicular?
3. Find the equation of the plane passing through the three points
P1 = (2, 1,1), P2 = (3,-1,1), P3 = (4,1,-1).

Let Q be a point outside a plane normal to N. We define the distance from point Q to the
plane as follows. Let Po be the point of intersection of the line through Q, in the direction
of N, and the plane through P. The distance d from Q to the plane is the distance between
Q and Po.

Now we find a formula for this distance. Clearly d  Pr ojQP QP


= Pr oj QP
N

 N.QP 
However, Pr ojN QP   N
 N2 
 
 N.QP  N.QP N.QP
Hence d   N  N =
 N2  N
2
N
 
Therefore the distance d of a point Q from a plane through P which is normal to N is given
N.QP
by: d 
N

26
FTVETI
APPLIED MATHEMATICS I MODULE

Self-Test Exercises

1. Let A = (0, 1, 5),B = . Find the angle between A and B.

2. Which of the following vectors are parallel or perpendicular to (1, 1, -1)?

a) (2, 2, -2) d) (1, 0, 1)

b) (2, -2, 0) e)

c) (-2, 2, 2) f)
3. a) Find all vectors that are orthogonal to E1 = (1, 0, 0)

b) Find all vectors that are orthogonal to both E1 and E3 = (0, 0, 1)

c) Find all vectors that are orthogonal to E1, E2 and E3 = (0, 0, 1)

4. Find a non-zero vector orthogonal to (1, 2, -1)

5. Find a unit vector in the direction of (3, -1, 2, 4)

6. Let ,

a) Show that each u1, u2, u3 is orthogonal to the other two and that each is a unit vector
b) Find the projection of E1 on each of u1, u2, u3

27
APPLIED MATHEMATICS I MODULE

7. In the following cases compute (A x B).C

a) A = (1, 2, 0) B = (-3, 1, 0), C = (4, 9, -3)

b) A = (-3, 1, -2) B = (2, 0, 4), C = (1, 1, 1)

8. Prove that two non-zero vectors A and B are perpendicular if and only if for every

number t.
9. If A + B + C = 0. Show that A x B = B x C = C x A
10. Find a formula for the area of a parallelogram whose vertices, in order, are
P, Q, R & S.

11. Show that

12. Find parametric equations of lines through


a) (-5, -6, 8) and (1, 3, 7) b. (10, 3, 1) and (6, -2, -3
13. Find equation of a plane through points (0, 1, 0), (0, -1, -1) and (1, 2, 1)
14. Find a point of intersection of the lines {p:p = (1, -5, 2) + t(-1, 1, 0)} and {p:p = (3, -3, 1) + t(4, 0, -1)}
15. Prove that the line {p:p = (1, 3, -1) + t(0, 3, 5)} lies entirely in the plane
{(x, y, z): 2x – 5y + 3z = -16}

16. Find the intersection point of the lines {p:p = (-2, 4, 6) + t(1, 2, 3)} and {p:p = (-2, 1, 6) + t(3, 2,
1)}. Find equation of the plane containing the two lines.
17. Find all points of intersection of the line {p:p = t(1, -3, 6)} and the plane {p:x + 3y + z = 2}
18. Prove that if B.N = 0 and A is on the plane {p:(p – po).N = 0} then the entire line {p:p = A + tB} lies
in the plane.
19. Find a line through the point po = (-5, 2, 1) and normal to the plane
{(x, y, z): x = y}

20. Find a line through (xo, yo, zo) and normal to the plane {(x, y, z): ax + by + cz = d}
21. Let be the line x = 1 + 2t, y = -1 + 3t, z = -5 + 7t. Find the two points on at a distance 3 units
from the plane 2(x-1) + 2(y+3) –z = 0
22. The set of all points equidistant from (0, 1, 5) and 5, -1, 3) is a plane. Find the equation

28
APPLIED MATHEMATICS I MODULE

CHAPTER-TWO
MATRICES AND DETERMINANTS
Introduction
In working with a system of linear equations such as
x1  2 x2  3x3  x4  2
2 x1  3x2  7 x3  5x4  1
 x1  10 x2  x3  2 x4  1
8x1  4 x2  11x3  8x4  0

Only the coefficients and their respective positions are important. Thus these coefficients can be
efficiently arranged in a rectangular array called a ''matrix''.
Moreover, certain abstract objects in higher mathematics can also be represented by matrices.
Thus, matrices can be used to solve systems of linear equations. They have also numerous
applications.

Objectives:
At the end of this unit each student should able to:
 Learn a matrix and know properties of matrices
 Know properties of matrix operations
 Realize inverses of a matrices
 Know types of matrix
 work with elementary row/column operations
 Understand the determinant of a matrix
 Understand how to solve system of linear equations by using Cramer’s rule.
 Work with eigenvalues and eigenvectors of a matrix

29
APPLIED MATHEMATICS I MODULE

2.1. Definition of Matrix and basic operations


2.1.1. The concept of matrix
Definition 2.1: Definition of Matrix
A rectangular array of numbers is called a matrix.
We shall mostly be concerned with matrices having real numbers as entries.
The horizontal arrays of a matrix are called its rows and the vertical arrays are called its columns.
A matrix having m rows and n columns is said to have the order of 𝒎 × 𝒏.
A matrix A of order 𝒎 × 𝒏 can be represented in the following form:
𝒏
𝒄𝒐𝒍𝒖𝒎𝒏𝒔

a11 a1 2  a1 n 
 
a 21 a 2 2  a 2 n 
' ' ' 
m rows   
' ' ' 
' ' ' 
 
a m1 a m 2 a m n 

Where 𝒂𝒊𝒋 is the entry at the intersection of the 𝒊th row and 𝒋th column.
Remarks
1. The numbers in the above matrix are called the matrix entries and are denoted by
ai j ,Where i, j are indices (which are natural numbers) with 1  i  m and 1  j  n.
The index 𝑖 is called the row index or row subscript because it gives the position in the horizontal
lines, and j is the column index or column subscript because it gives the position in the vertical
lines. So aij is the entry which appears in the ith row and jth column of the matrix.

2. The plural of matrix is matrices. If each entry of a matrix is a real number / complex number, then
the matrix is called a real/ complex matrix respectively.
3. An 𝑚𝑥𝑛 matrix is said to be of order 𝑚𝑥𝑛. If 𝑚 = 𝑛, then the matrix is square of order n. For a

square matrix, the entries a11 , a2 2 , a3 3 ,   are called the main diagonal entries.

30
APPLIED MATHEMATICS I MODULE

4. We can abbreviate the above matrix by writing A  (ai j )mxn.

Example-1: Consider the matrix


0 1 2 3
a. 𝐴 = [𝑒 √2 −1 −15]
1
−3 0 10
2

Since A has three rows and four columns, we say that A is a 3𝑥4 matrix or A is a matrix of
Order 3𝑥4 .Moreover,𝑎11 = 0, 𝑎12 = 1, 𝑎13 = 2, 𝑎14 = 3
𝑎21 = 𝑒, 𝑎22 = √2, 𝑎23 = −1, 𝑎24 = −15
1
𝑎31 = , 𝑎32 = −3, 𝑎33 = 0, 𝑎34 = 10
2

1 3 7
b. 𝐵 = [ ] has two rows and three columns, we say that B is a matrix of order
−2 6 −1
2 × 3. Moreover,
𝑎11 = 1, 𝑎12 = 3, 𝑎13 = 7
𝑎21 = −2, 𝑎22 = 6, 𝑎23 = −1
A matrix having only one column is called a column vector and a matrix with only one
row is called a row vector
Example-2: The following matrices have the indicated orders.
(a) Order: 1x1 (c) Order: 3x3

0 0 0 
0 0 0 
2  
0 0 0

(b) Order: 1x4 (d) Order: 3x2


𝑒 1
0 2  1 3 [ 2 −1]
−8 1
Equality of matrix
Definition 2.2: Definition of Equality of Matrices
   
Two Matrices A= ai j and B= bi j are equal, denoted by A=B if and only if

I) they have the same order (𝑚 × 𝑛) and


II) 𝑎𝑖𝑗 = 𝑏𝑖𝑗 , ∀𝑖, 𝑗 Such that 1  i  m and 1  j  n.

31
APPLIED MATHEMATICS I MODULE

That is, two matrices of the same order are said to be equal if their corresponding entries are equal.
Example-3: Equality of Matrices
Solve for 𝑥, 𝑦, 𝑧 𝑎𝑛𝑑 𝑤 in the following matrix equation
x  1 0  2 y 
5 
 z  1 
 w  2
Solution:
Because two matrices are equal only if the corresponding entries are equal, we conclude that:
x  1  2, y  0, w  5 and z  1  2
 x  1, y  0, w  5, z  1.

2.1.2. Operation on matrix


I. Addition and subtraction of matrices
Definition 2.3: Definition of matrix Addition
 
Let A= ai j and B  bi j   be matrices of order 𝑚𝑥𝑛, then their sum is an mxn matrix given by
A  B  a  b 
ij ij That is, we add the two matrices of the same order by adding their

corresponding entries and the sum of two matrices of different orders is undefined.
𝑎11 𝑎12 … 𝑎1𝑛 𝑏11 𝑏12 … 𝑏1𝑛
𝑎 𝑎22 … 𝑎2𝑛 𝑏 𝑏22 … 𝑏2𝑛
That is, let 𝐴 = [ 21 ] and 𝐵 = [ 21 ] then
⋮ ⋮ ⋮ ⋮ ⋮ ⋮
⋱ ⋱
𝑎𝑚1 𝑎𝑚2 𝑎𝑚𝑛 𝑏𝑚1 𝑏𝑚2 𝑏𝑚𝑛
𝑎11 + 𝑏11 𝑎12 + 𝑏12 … 𝑎1𝑛 + 𝑏1𝑛
𝑎 + 𝑏21 𝑎22 + 𝑏22 … 𝑎2𝑛 + 𝑏2𝑛
𝐴 + 𝐵 = [ 21 ] = (𝑎𝑖𝑗 + 𝑏𝑖𝑗 )𝑚×𝑛
⋮ ⋮ ⋮

𝑎𝑚1 + 𝑏𝑚1 𝑎𝑚2 + 𝑏𝑚2 𝑎𝑚𝑛 + 𝑏𝑚𝑛

Example-4: Addition of Matrices

2 1 0  0 1  1 0 0
    0 1 0 
Let A = 4 0  1  , B =  1 3 , C =  
3  2 2 2 4 0 0 1

Then find a) A+B b) A+C.

32
APPLIED MATHEMATICS I MODULE

Solution:
a) Since A and B have different orders, we have A+B is undefined.

2 1 0  1 0 0 2+1 1+0 0+0 3 1 0


4 0  1  0 1 0 
b) A+C =   +   = [ 4 + 0 0 + 1 −1 + 0] = [ 4 1 −1]
3  2 2 0 0 1 3 + 0 −2 + 0 2 + 1 3 −2 3

Definition 2.4: Definition of Matrix Subtraction


Let A and B be two Matrices of the same order then the difference of A and B, denoted by A-B
, is defined to be the matrix obtained by subtracting each element of B from the corresponding
elements of A. That is if𝐴 = (𝑎𝑖𝑗 ) 𝑚 × 𝑛 &𝐵 = (𝑏𝑖𝑗 ) 𝑚 × 𝑛 then −𝐵 = (𝑎𝑖𝑗 − 𝑏𝑖𝑗 ) 𝑚 × 𝑛 .
𝑎11 𝑎12 … 𝑎1𝑛 𝑏11 𝑏12 … 𝑏1𝑛
𝑎 𝑎22 … 𝑎2𝑛 𝑏21 𝑏22 … 𝑏2𝑛
That is, let 𝐴 = [ 21 ⋮ ] and 𝐵 = [ ⋮ ] then
⋮ ⋮ ⋮ ⋮
⋱ ⋱
𝑎𝑚1 𝑎𝑚2 𝑎𝑚𝑛 𝑏𝑚1 𝑏𝑚2 𝑏𝑚𝑛
𝑎11 − 𝑏11 𝑎12 − 𝑏12 … 𝑎1𝑛 − 𝑏1𝑛
𝑎 − 𝑏21 𝑎22 − 𝑏22 … 𝑎2𝑛 − 𝑏2𝑛
𝐴 − 𝐵 = [ 21 ] = (𝑎𝑖𝑗 − 𝑏𝑖𝑗 )𝑚×𝑛
⋮ ⋮ ⋮

𝑎𝑚1 − 𝑏𝑚1 𝑎𝑚2 − 𝑏𝑚2 𝑎𝑚𝑛 − 𝑏𝑚𝑛

Example-5: Subtraction of Matrices


1 2 3  2 3 0 
Let A=  , B    then
 0 1 4   1 2 5

1 2 3   2 3 0  1  2 23 3  0   1  1 3 
A  B       =  
 0 1 4   1 2 5 0  (1) 1  2 4  5  1  1  3
Note that: Two matrices of the same order are said to be conformable for addition and subtraction.
Only conformable matrices can be added or subtracted.

II. Multiplication of a matrix by scalar:


Definition 2.5: Definition of scalar multiplication
Let 
A  ai j mxn be a matrix and k be scalar, and then a scalar multiple of A by k is an
mxn matrix given by:kA = kaij That is, kA is an mxn matrix obtained from A by multiplying each
of its elements by k.

33
APPLIED MATHEMATICS I MODULE

 ka11 ka12  ka1n 


 
 kaij mxn
 ka ka22  ka2 n 
kA   21
    
 
 ka  kamn 
 m1 kam 2
Example-6: Scalar multiplication

 1 2 4  2 0 0
   
Let A=  3 0  1 and B   1  4 3  then find
 2 1 2  1 3 2 

a) 3𝐴 b) −𝐵 c) 3𝐴 − 𝐵

 1 2 4 3(1) 3(2) 3(4)   3 6 12 


   3(2) 3(0) 3(1)   9 0  3 
Solution: a) 3A  3 3 0  1
 2 1 2  3(2) 3(1) 3(2)   6 3 6

2 0 0 (1)2 (1)0 (1)0   2 0 0 


   (1)  4 (1)3    1 4  3 

b)  B   1B  (1)  1  4 3   (1)1
 1 3 2 (1)  1 (1)3 (1)2  1  3  2

 3 6 12   2 0 0   1 6 12 
     
c) 3 A  B   9 0  3   1  4 3    10 4  6
 6 3 6  1 3 2  7 0 4
Note:

(1) If A is any matrix and k is any scalar then 𝑘𝐴 = 𝐴𝑘


(2) −𝐴 represent the scalar product (−1) 𝐴.
2.1.3. Properties of matrix addition and scalar multiplication
Suppose 𝐴, 𝐵 𝑎𝑛𝑑 𝐶 are 𝑚𝑥𝑛 matrices and  and  are scalars .Then
𝑖) 𝐴 + 𝐵 = 𝐵 + 𝐴 ………….(matrix addition is commutative)

𝑖𝑖) (𝐴 + 𝐵) + 𝐶 = 𝐴 + (𝐵 + 𝐶)……………..( matrix addition is associative)

𝑖𝑖𝑖) (𝛼 + 𝛽)𝐴 = 𝛼𝐴 + 𝛽𝐴

𝑖𝑣) 𝛼(𝐴 + 𝐵) = 𝛼𝐴 + 𝛼𝐵

𝑣) 1𝐴 = 𝐴

34
APPLIED MATHEMATICS I MODULE

Proof:
i. Let 𝐴 = (𝑎𝑖𝑗 )𝑚×𝑛 and 𝐵 = (𝑏)𝑚×𝑛 then
𝐴 + 𝐵 = (𝑎𝑖𝑗 )𝑚×𝑛 + (𝑏𝑖𝑗 )𝑚×𝑛 = (𝑎𝑖𝑗 + 𝑏𝑖𝑗 )𝑚×𝑛 = (𝑏𝑖𝑗 + 𝑎𝑖𝑗 )𝑚×𝑛
= (𝑏𝑖𝑗 )𝑚×𝑛 + (𝑎𝑖𝑗 )𝑚×𝑛 = 𝐵 + 𝐴
Note: Each of the remaining properties can be proved in similarly way, and use the above
definition.

Activity 2.1

1. Determine the order of each of the following matrices?

(a) (b) (c) (d)

2. What are the entries the matrix

a. A= b. A =

3. Construct a 4x3 matrix B= ( b ) where c)


b) d)

4 . Fin

2.2. Product of matrices and some algebraic properties; transpose of a matrix


2.2.1. Matrix multiplication
It is a basic matrix operation. At first glance the following definition may seem unusual. You will
see later, however, that this definition of the product of two matrices has many practical
applications.

35
APPLIED MATHEMATICS I MODULE

Definition 2.6: Definition of Matrix multiplication


Let   
A  ai j be an mxn matrix and B  bi j is an nxp matrix, then the product AB is an mxp

matrix given by AB  ci j  where ci j n


  ai k bk j
k 1

 ai 1 b1 j  ai 2 b2 j  ai 3 b3 j      ai n bn j
th
That is, the ij element of AB is obtained by multiplying each element of the ith row of A (denoted
by Ai) in to the corresponding element of the jth column of B (denoted by Bj ) and adding the
products or equivalently, the ijth element of AB is the dot product of the ith row vector of A by
 
the jth column vector of B and hence AB  ci j where ci j  Ai .B j

Example-1: Finding the Product of the following Matrices

 1 3 
   3 2 
a. Let A   4  2  and B  
 4 1
find AB
 5 0 

Solution:
First note that the product AB is defined because A has order 3x2, and will take the form.

 1 3  c11 c12 
 4  2    3 2   c 
 
 4 1   21 c 22 
 5 0     
c31 c32 
To find c11 (the entry in the first row and first column of the product) we take the dot product of
the first row of A and the first column of B That is,
−3
𝑐11 = 𝐴1 . 𝐵1 = [−1 3] [ ] = (−1)(−3) + 3(−4) = 3 − 12 = −9
−4
Similarly, to find c1 2 , we tare the dot product of the first row of A and the second column of B

and hence c1 2  A1 B
2

 2
 (1 3).  
 2
 (1) (2)  (3)(1)  2  3  1

Continuing this pattern produces the following results.

36
APPLIED MATHEMATICS I MODULE

 3
c21  A2 B1  (4  2)  .  .(4) (3)  (2)(4)  4
  4
 2
c22  A2 B 2  (4  2)  .  .(4) (2)  (2)(1)  6
1 
 3
c31  A3 B1  (5 0).   (5) (3)  (0)(4)  15
  4
 2
c32  A3 B 2  (5 0).   (5) (2)  (0)(1)  10
1 
Thus the product is

 1 3   9 1 
 3 2 
AB   4  2  
 4 1   4 6 

 5 0    15 10

1 2
 3 4 2 1  
  3 4
b. If A   1 2 3 1  and B .
 0 1 2 3 1 0
   
 1 1 

 3  12  2  1 6  16  0  1 16 23 
   
Then AB   1  6  3  1 2  8  0  1    9 11 
 03 23 0 4 03   2 7 
   
1 2 1
1 2 3
c. if 𝐴 = [ ] and 𝐵 = [0 0 3] then
2 4 1
1 0 4
1+0+3 2 + 0 + 0 1 + 6 + 12 4 2 19
𝐴𝐵 = [ ]=[ ]
2+0+1 4 + 0 + 0 2 + 12 + 4 3 4 18
Note that in this example, while AB is defined, the product BA is not defined. However, for square
matrices A and B of the same order, both the product AB and BA are defined.
Remarks
1. The product of two matrices A and B is defined if the number of columns of A is equal to the
number of rows of B, say if A is an 𝑚𝑥𝑛 matrix and B is an nxp matrix. In this case, the product
AB is an mxp matrix.
2. Let 𝐴 = (𝑎𝑖𝑗 )𝑚×𝑛 and 𝐵 = (𝑏𝑗𝑘 )𝑛×𝑝 then

37
APPLIED MATHEMATICS I MODULE

 A1 B1 A1 .B 2 A1 B 3    A1 .B p 
 
1
AB=  A2 .B A2 B
2
A3 B 3    A2 B p 
 A B1 A B 2 A B 3    A B p 
 m m m m 
j
Where Ai. B is the dot product of the ith row of A and jth column of B

i  1, 2,  , m and j  1, 2,  , p


3. If the product that AB is defined, then it is not necessary that BA must also be defined. For instance,
if A is of order 3x3 and B is of order 3x2, then clearly AB is defined but BA is not defined, as the
number of columns of B is not equal to the number of rows of A.
2.2.2. Properties of Matrix operation
Theorem 2.1: Properties of Matrix Multiplication
In defining the properties of matrix multiplication below, the matrix 𝐴, 𝐵 𝑎𝑛𝑑 𝑪 are assumed to be
of compatible dimensions for the operations in which they appear.
Property I: Matrix multiplication is, in general, not commutative. That is 𝑨𝑩  𝑩𝑨. Observe that
in Example 1(c) of the above section 𝑩𝑨 is not even define because the first matrix in this case 𝐵
does not have the same numbers of columns as the number of rows of the second matrix A.
Property II: From AB=0, it does not follow that either A=0 or B=0.Here O’S are null matrix of
appropriate order.
Example 2: For matrices A and B are given by
1 0   0 0
A=   and B=  
1 0  1 1
 0 0
We have, AB=   is a null matrix even though A or B is not a null matrix.
 0 0
Property III: The relation AB=AC or BA=CA does not imply that B=C.The cancelation law
does not hold in general as in a real numbers.
Example 3 For the matrices

 1 2 3 1 2 3   2 3 4
     
A   1 1 2 B   1 1  1, C   2 2 0
 1 4 3  2 2 2  1 1 1
     
We have, by direct multiplication

38
APPLIED MATHEMATICS I MODULE

 9 10 7 
 
AB   6 7 6   AC , although B  C
 9 8  1
 
Property IV: Matrix multiplication is associative That is
A (BC) = (AB) C
Property V: The multiplication of matrices is distributive with respect to addition i.e
A(B+C)=AB+AC (B+C)A=BA+CA
Example 4:
1 2  2 1 1 0
A=   B=   and C=  
3 4   3 2  2 1
Verify that A (BC) = (AB) C and A (B+C) =AB+AC
Solution:

 1 2  4 1   6 5 
A(BC )       
 3 4  1 2   16 11 
  4 5  1 0   6 5 
( AB )C       
  6 11 2 1  16 11
Thus
6 5
A( BC )     ( AB )C
16 11
 1 2  3 1   1 7 
A( B  C )       
 3 4   1 3   5 15 
  4 5   5 2 1 7 
AB  AC         
  6 11 11 4   5 15 
Therefore
A(B+C)=AB+AC.
Notation: Since A (BC)=(AB)C, one may simply omit the parentheses and write ABC.
The same is true for a product of or more matrices .In the case where an nxn matrix is multiplied
by itself a number of times, it is convenient to use exponential notation. Thus, if k is a positive,
then

39
APPLIED MATHEMATICS I MODULE

Ak  
AA
 
A
k times

1 1
Example 5: if A   
 1 1 
1 11 1  2 2 
Then A2       
1 11 1  2 2 
1 1 2 2   4 4 
A3  AAA  AA 2       
1 1 2 2   4 4 
And in general
 2 n1 2 n1 
An   n1 
2 2 n1 

Example 6: Simplify the following matrix expression


A(A+2B)+3B(2A-B)-A2+7B2-5AB
Solution: Using the properties of matrix we get
A( A  2 B)  3B(2 A  B)  A2  7 B 2  5 AB  A2  2 AB  6 BA  3B 2  A2  7 B 2  5 AB
 3 AB  6 BA  4 B 2

Proof: Exercise
2.2.3. Transpose of a matrix and its properties
Definition 2.7: The Transpose of a Matrix

Let 𝐴 = [𝑎𝑖𝑗 ]be an 𝑚 × 𝑛 matrix. Then the transpose of 𝐴, denoted by At (and sometimes by 𝐴′),
𝑡
is the matrix obtained interchanging rows and columns to produce the 𝑛 × 𝑚 matrix 𝐴𝑡 = [𝑎𝑖𝑗 ] =
[𝑎𝑗𝑖 ].

Example-7: The Transpose of a matrix

3 2 1
 3
(a) A    (b) B  4 1 0
  4 0 0 1

40
APPLIED MATHEMATICS I MODULE

a11 a12 a13  a1n 


 
a 21 a 22 a 23  a 2 n 
(c) C  a31 a32 a33  a3n 
 
    
a a a  a 
 m1 m 2 m 3 mn 

3 4 0 
 
Solution: (a) A  3  4 (b) B  2 1 0
t t

1 0 1

a11 a 21 a31  a m1 
a a a32  a m 2 
 12 22
(c) C  a13 a 23
t
a33  a m3 
 
    
a1n a 2 n a3n  a mn 

Theorem 2.2: Properties of Transposes
Let A and B be matrices with orders such that the given matrix operations are defined and c is a
scalar, then the following properties are true

a) (𝐴𝑡 )𝑡 = 𝐴
(b) (𝐴 + 𝐵)𝑡 = 𝐴2 + 𝐵2
(c) (𝑐𝐴)𝑡 = 𝑐(𝐴𝑡 )
(d) (𝐴𝐵)𝑡 = 𝐵𝑡 𝐴𝑡
(e) (I n )t  I n
Proof: Exercise

Definition 2.8: A matrix A is said to be symmetric if it is equal to its transpose. That is, if
𝐴𝑡 = 𝐴 then A is called symmetric matrix.
Note: A symmetric matrix is necessarily a square matrix
 1 1 2 
 
Example-8: Let A   1 0 3 then since A=At we have A is symmetric
 2 3 7
Definition 2.9: A square matrix a is said to be skew-symmetric if At = -A

 2 0
Example-9: Let A   , then A   A and hence A is skew symmetric.
t

  2 0 

41
APPLIED MATHEMATICS I MODULE

Activity 2.2

1. Let then show that A(BC) =(AB)C

2. Consider then show that

3. Let and B= and conclude that matrix

multiplication is not commutative.

4. Let A=

2.3. Types of Matrices: Square, identity, scalar, diagonal, triangular,


symmetric, and Skew symmetric matrices
1) Row Matrix: A matrix which has only one row. It is a 1xn matrix, where n is arbitrary natural
number. It is also called a row vector
Example-1: (-1 0 0 4) is a row matrix with order 1x4.
(1 -2 -3) is a 1x3 row matrix.
2) Column Matrix: A matrix which has one column. It is also called a column vector.

2
1 
Example-2:   is a 3 by 1 column matrix.
5 

3) Zero matrix: is a matrix of arbitrary size in which all the entries are zero. It is also called null
matrix and denoted by O.
Example-3: The following are null matrices

42
APPLIED MATHEMATICS I MODULE

0 0 
0 0  0 0 
0 0 0 0   
  0 0

 order 1x2  order 2x2  order 3x2


Theorem 2.3: Properties of zero Matrices
Let A be an mxn matrix and c be a scalar then
a) A  Om n  A

b) A  ( A)  Om n

c) cA Om n  c  o or A  Om n

Proof: Left as an exercise.


Remarks:

1. Property b) above can be described by saying that the matrix-A is the additive inverse of A
The algebra of real numbers (scalars) and the algebra of matrices have many similarities.
4) A square matrix: Is a matrix where the number of rows is equal to the number of columns. It is
an 𝑛 × 𝑛 matrix; where n  N .

1 0  3 
2 1  4 1 3 
Example 4:    
3 8 6 7 1 2 

Square matrix of square matrix of


Order 2x2 order 3x3
 
Let A  ai j be a square matrix of order n, then

a) The entries ai i ; 1  i  n are called main diagonal entries of A.

b) The sum of the main diagonal entries of A is called the trace of A and it is denoted by
n
Tr ( A) That is, Tr ( A)  a11  a 22  a33      a nn   aii
i 1

43
APPLIED MATHEMATICS I MODULE

Example-5: Find the trace of the following matrix

1 2 3  1 0 0 
(a) A  0  2 4  (b) B  0 1 0
 
3 1 3 0 0 1
3
Solution: (a) T r ( A)   aii  1  2  3  2
i 1

3
(b) Tr ( B)   bii  b11  b22  b33  1  1  1  3
i 1

5) Diagonal Matrix: Is a square matrix in which all entries that are not on the main diagonal are zero.

 d11 0  0 
 
 0 d 22  0 
D
    
 
0  d nn 
 0
Example-6: The following are Diagonal matrices

 1 0 0  2 0 0 
  (b) 0  3 0 
(a)  0 2 0
 0 0 2 0 0 0

a11 0 0 0 
0 a 22 0 0 

(c) 0 0 a33  0 
 
    
0 0 0 a nn 

Note: If A  (ai j )nxn is a diagonal matrix then ai j  O i  j while aii can be zero or different

from zero.
6) Scalar Matrix: Is a diagonal matrix whose main diagonal elements are equal. That is, if A  (ai j )

is a square matrix such that


0 , if i  j
ai j   Where c is a scalar, then A is called scalar matrix.
c , if i  j

44
APPLIED MATHEMATICS I MODULE

c 0  0
 
0 c  0
Sn  
   
 
0 0  c 

3 0 0 
 
Example-7: D  0 3 0 
0 0 3

7) Identity Matrix: Is a diagonal matrix in which all the main diagonal elements are equal to 1. It is
also called a unit matrix.
1 0 0 0  0
0 1 0 0  0

In  0 0 1 0  0
 
    
0 0 0 0 1

1 0 0 
1 0  
Example-8: 1  , 0 1 0 are unit matrices of order 1,2 and 3 respectively
0 1 0 0 1
 
Note

1. The 𝑛 × 𝑛 identity matrix is denoted by I n

2. The identity matrix behaves like 1 in multiplication of numbers: If A is a 𝑚 × 𝑛 matrix,

then Im A = A and A I n  A
Theorem 2.4: Properties of the identity Matrix
If A is a matrix of order mxn, then

a) A I n  A

b) I m A  A

c) If m=n then A I n  I n A  A

45
APPLIED MATHEMATICS I MODULE

Example-9:
 2 
2 3  
Let A   , B   1  then
5 6  4
 
2 3  1 0   2 3 
a) AI 2          A  I 2 A
5 6   0 1  5 6 

1 0 0    2   2 
     
b) I 3 B   0 1 0   1    1 
 0 0 1  4   4
     
Note: For repeated multiplication of square matrices, we use the same exponential notation used
with real numbers.

That is, A1  A

A2  AA and for k  N
We define Ak  AA    A  K factors

It is convenient also to define A0= I n , where A is a square matrix of order n


As a result one can observe that the following properties.
a) A j Ak  A j k

b) ( A )  A
j k jk

8) Upper triangular Matrix: Is a square matrix in which all the elements below the main diagonal
are zero. That is, if A  (ai j )nxn is a square matrix such that ai j  0 for all i  j then A is

said to be upper triangular matrix.


Example-10:
1 0 0 0
1 2 3  0 1
  0 0
A  0 2 0  , B   are upper triangular matrices of order 3 and 4
0 0 1 0
0 0 8  
0 0 0 1
respectively.
9) Lower triangular matrix: Is a square matrix whose entries above the main diagonal are 0.

46
APPLIED MATHEMATICS I MODULE

1 0 0 0
  1 0 0 0 1 0 0 
Example-11:  1 0 0,   are lower triangular matrices of order 3
0 0 1 0 
 2 3 1   
0 0 0 1
and 4 respectively.
Note:

1) We often indicate that a whole region in a matrix consists of zeros by leaving it blank or by putting
in a single O.
2) We use * to indicate an arbitrary undetermined entry of a matrix.

Activity 2.3

Identify then type of the following matrix


−1
1 0 0 0
a. 𝐴 = [3 1 0] c. 𝐵 = 0 e. 𝐸 = [0 9 12 5 −2]
2 3 1 3
4
5 0 0 0 −2 6 4
0 8
b. 𝐵 = [0 5 0 0] d. 𝐷 = [ ] f. 𝐹 = [ 0 0 0]
0 0 5 0 0 0
7 8 9
0 0 0 5

2.4. Elementary operations and its properties

2.4.1. Elementary row operations


1 2 3  4 5 6  1  2 1 
Consider the matrices A  4 5 6  and B  1 2 3, C  2 1  2,
 
0 0 1 0 0 1 3 1  1

  2 4  2 1  2 1  1  2 1 
D   2 1  2 E  3 1  1  and F  0 7  4
 3 1  1  2 1  2 2 1  2

47
APPLIED MATHEMATICS I MODULE

Observe that:
I) B is obtained from A by interchanging the first and second rows of A
II) D is obtained from C by multiplying the first row of C by -2
III) F is obtained from matrix E by replacing the second row of E by a new row
This is made up by multiplying the first row by -3 and adding to the second row of E.
Such operations on rows of a matrix as described above in (i),(ii) and (iii) are called elementary
row operations. These operations will be very useful in finding inverse of a matrix and solving
systems of linear equations.
Similarly, elementary column operations can be defined.
Transformations called elementary transformations can be used to change a system of linear
equation into another system of linear equations that has the same solution. These
transformations are used to solve systems of linear equations by eliminating variables. In practice
it is simpler to work in terms of matrices using equivalent transformations called elementary row
operation. Hence we have the following definition
Definition 2.10: (Elementary Transformations)
1. Interchanging two equations
2. Multiplying both sides of an equation by a nonzero constant
3. Add a multiple of one equation on to another equation.
Definition 2.11: An elementary operation on a matrix is either elementary row operation or
elementary column operation and is of the following three types.

Type I: The interchange of any two rows or (columns)


Type II: The multiplication of any row or (column) by a non-zero number
Type III: The addition of a multiple of one row (or column) to another row (or column)
Notations: We shall use the following notations for the three types of elementary operations.

1. The interchange of 𝑖 th and 𝑗th rows (columns) is denoted by


Ri  R j (Ci  C j )

2. The multiplication of the 𝑖 th row (column) by a none zero number k is denoted by


Ri  kRi (Ci  kCi )
3. The addition of k times the 𝑗th row (column) to 𝑖 th row (column) is denoted by

48
APPLIED MATHEMATICS I MODULE

Ri  Ri  KR j (C  Ci  kC j )

Note:

a. Ri  kRi means replace the 𝑖 th row by k times the 𝑖 th row

b. Ri  Ri  kR j Means replace the 𝑖 th row by the sum of itself and k times the 𝑗th

row.
c. Although elementary operations are simple to perform, they involve a lot of
arithmetic Because it is easy to make a mistake, we suggest that you get in the
habit of noting the elementary operation performed in each step so that you can
go back to check your work
Example 1: Solve the following system of linear equations.

 x1  x2  x3  2

 2 x1  3x2  x3  3
 x  x  2 x  6
 1 2 3

Solution:
a)
I. Elimination Methods
x1  x2  x3  2
Initial system 2 x1  3x2  x3  3
x1  x2  2 x3  6
Eliminate 𝑥1 from the 2nd and 3rd equations
x1  x2  x3  6
eq (2)  2eq (1)  eq (2)
 x2  x3  1
eq (3)  eq (3)  eq (1)
 2 x2  3x3  6
Eliminate 𝑥2 from the 1stand 3rdequations
x1  2 x3  3
eq (1)  eq (1)  eq (2)
 x2  x3  1
eq (3)  eq (3)  2eq (2)
 5 x3  10
Make coefficient of x3 in 3rd
x1  2 x3  3
 x2  x3  1 1
eq (3)  eq (3)
5
x3  2
st nd
Eliminate x3 from 1 and 2 equations

49
APPLIED MATHEMATICS I MODULE

x1  1
 x2  1
x3  2
The solution is x1=-1,x2 = 1,x3 =2
II. Matrix Method
1 1 1 2
 
Augmented matrix 2 3 1 3
 1 1  2  6 
 

We refer to the first row as the pivot row, and then we have:
1 1 1 2
 
  0 1  1  1  , R2  R2  2 R1 , R3  R3  R1
 0  2  3  8
 

b)

Create appropriate zeros in column 2


1 0 2 3 
 
  0 1 1 1  R1  R1  R2 , R3  R3  2 R2
 0 0  5  10 
 

c) Make the (3,3) element 1


1 0 2 3 
  1
  0 1  1  1, R3  R3
0 0 1 2  5
 

d) Create zeros in column 3


 1 0 0  1
 
  0 1 0 1 , R1  R1  2 R3 , R2  R2  R3
0 0 1 2 
 

e) Matrix corresponding to the system


x1  1
x2  1
x3  2
Thus the solution is 𝒙𝟏 = −𝟏, 𝒙𝟐 = 𝟏, 𝒙𝟑 = 𝟐

50
APPLIED MATHEMATICS I MODULE

Definition 2.12: Row Equivalent and column Equivalent Matrices


Two matrices are said to be row-equivalent or (column equivalent) if one can be obtained from the
other by a finite sequence of elementary row or (column) operations. That is,
row
a. A  (ai j )mxn is row equivalent to B  (bi j )mxn, denoted by A  B, if B is attainable from
A by successive operations of finitely many elementary row operations on A
column
b. A  (ai j )mxn is column equivalent to B  (bi j )mxn, denoted by A  B, if B is attainable
from A by successive operations of finitely many elementary column operations on A.
Example-2:
 1 2 0 1 3 3 
Let A   3  1 2 , B  1 4 3  then show that
 2 3  2 1 3 4

1 2 0
(a) A  0  7 2
row row
(b) B  I 3
0 0 0 

Solution:
(a)  1 2 0  1 2 0 
  R2 3 R  R  
A   3 1 2     1
2  
 0 7 2
  3 3  2    2 3  2 

1 2 0 1 2 0 
3 
2 R1  R 3   R3 R2  R3  
 R 
 0 7 2      0  7 2 

0 7  2   0 0 0 

1 3 3  1 3 3  1 3 3 
(b) B  1 4 3  0 1 0  R
  R2  R2  R3 3  R3  R1
 0 1 0
 
1 3 4 1 3 4 0 0 1

1 0 3  1 0 0 
  0 1 0   0 1 0 
 
R1  R1 3 R2 R1  R1 3 R3

0 0 1 0 0 1

Example 3: the following foure matrices are row equivalent

51
APPLIED MATHEMATICS I MODULE

0 0 1 2  3 3 6  9 0 0 1 2  0 0 1 2 
       
A   2 3 0  2  B   2 3 0  2  C   2 3 0  2  D   2 3 0  2
3 3 6  9 0 0 1 2   1 1 2  3  1  3 6  5 
       
Because of

 B is obtained from A by interchanging row 1and row 3 .(i.e R1  R3 )

 C is obtained from A multiply the third row of A by 1 . (i.e R3  1 R3 )


3 3
 D is also obtained from A multiply the second of A by -2. (i.e R2  2R2 )
Elementary Matrices
Definition 2.13: Definition of Elementary Matrix
Let A be an nxn matrix such that A can be Obtained from In by a single elementary
row /column operation then A is called an elementary matrix.
Example-2: Elementary Matrices and Non elementary Matrices
Which of the following matrices are elementary? For those that are describe the corresponding
elementary row operation.
1 0 0 0 
 1 0 0 0 1 0 0 
(a) A  I n (b) B   0  4 0 (c ) C   
0 0 1 0 
 0 0 1  
0 0 0 0
1 0 0  1 0 0 
1 0 0 1 0 
(d) D    (e) E  0 0 1  ( f ) F  0 2 0  ( g ) G  
 
0 1 0   4 1
0 1 0 0 0  1

Solution:
(a) Since A  I n can be obtained from itself by multiplying any one of its rows by 1.
1 1R1
(i.e. I n R  A) we' ve I n is elementary matrix.

(b) Since B can be obtained from I 3 by multiplying the second row of


2 4 R2
I 3 by  4 we have I 3 R  B and hence B is an elementary matrix.

(c) Since C is obtained by multiplying the fourth row of I 4 by 0 we have C is not

elementary matrix. Note that, row multiplication must be anon zero constant.

52
APPLIED MATHEMATICS I MODULE

(d) D is not elementary because it is not square matrix.


2  R3
(e) Since I 3 R  E we've E is elementary.

(f) This matrix is not elementary because two elementary row operations are required to
obtain it from I 3 .
2 4 R1  R2
(g) Since I 2 R   G we have G is elementary.
Note: Elementary Matrices are useful because they enable us to use matrix multiplication to
perform elementary row/ column operations.

Example-4: Elementary Matrices and Elementary Row Operations


0 1 0  1 0 0  1 0 0 
Let E1  1 0 0, E 2  0 1 2 0 and E3  2 1 0 
   
0 0 1  0 0 1 0 0 1

0 2 1  1 0  4 1   1 0 1 
Let A1  1  3 6, A2  0 2 6  4 and A3   2  2 3
   
3 2  1 0 1 3 1   0 4 5 

Then
1  3 6 
(a) E1 A1  0 2 1 , this product matrix is a matrix obtained by interchanging the first two rows
3 2  1

of A. And E1 is an elementary matrix in which the first two rows of I 3 has been interchanged.

1 0  4 1 
(b) E 2 A2 0 1 3  2 
0 1 3 1 

Here, E 2 is an elementary matrix in which the second row of I 3 has been multiplied by 1
2 and
1 R
A   2  E2 A2
2 R
2

1 0  1 2 2 R1  R2
I 3 R   E3 and
(c) E3 A3  0  2 1 Here,
2  2 R1  R2
A3 R   E3 A3
0 4 5 

53
APPLIED MATHEMATICS I MODULE

Definition 2.14: Let A, B be an mxn matrices. We say that A is equivalent to B, denoted by

A  B , if B can be attainable from A by successive application of finitely many elementary row


or column operations.
Example-5:
0 1 0   1 0 0 1 0 0 
 
Let E1  1 0 0 ,  
E 2   0 1 0 and E3  0 1 0  then
0 0 1  2 0 1 0 0 12 

𝐸1 ≅ 𝐸2 ≅ 𝐸3 which is
0 1 0 𝑅1 ↔𝑅2 1 0 0 𝑅3 →−2𝑅1 +𝑅3 1 0 0
𝐸1 = [1 0 0] → [0 1 0] → [ 0 1 0]= 𝐸2
0 0 1 0 0 1 −2 0 1
0 1 0 𝑅1↔𝑅2 1 0 0 𝑅3 →1𝑅3 1 0 0
2
𝐸1 = [1 0 0] → [0 1 0 ] → [0 1 0]= 𝐸3
0 0 1 0 0 1 −2 0 1

2.4.2 Row reduced echelon form of a matrix

1. Row –Echelon Matrix


Definition2.15: Definition of Row-Echelon Form of a Matrix

A matrix is said to be in row-echelon form if

1. All the non-zero rows, if any, precede all zero rows, if any. That is, all rows consisting
entirely of zeros occur at the bottom of the matrix.
2. In any non-zero row after the first row, the number of zeros preceding the first non- zero
elements is greater than the number of such zeros in the preceding row. That is, the first
non-zero entry of row i+1 is to the right of the first non-zero entry of row 𝑖
3. The first non-zero entry in each non-zero row is 1. This entry is called a pivot or a leading
1.
Example-1: Determine whether the given matrix is in row-echelon form or not.
 1 2 1 4  0 1 0 5 
(a)  0 1 0 3 (b) 0 0 1 3 
 
 0 0 1  2 0 0 0 0

54
APPLIED MATHEMATICS I MODULE

1 2  3 4  1 2  1 2 
(c) 0 2 1  1  (d ) 0 0 0 0 
 
0 0 1  3 0 1 1  4

Solution: The matrices shown in parts (a) and (b) are in row-echelon form, while the matrices
shown in parts (c) and (d) are not in row-echelon form.
Note: One can show that every matrix is row equivalent to a matrix in row-echelon form.

Definition 2.16: Definition of Reduced Row-Echelon Form of a matrix.


A matrix M in row-echelon form is in reduced row-echelon form if the first non-zero element in
each non-zero row is the only non-zero element in its column.
That is, M is in reduced row-echelon form if M is in row-echelon form and if every column that
has a leading 1 has zeros in every position above and below its leading 1
Note that: It can be shown that the reduced row echelon matrix obtained from a given matrix A
by elementary operations is unique. That is, that it does not depend on the particular sequence of
operations used.

Example-2: The following matrices are in reduced row-echelon

1 0 0  1
0 0 1 0 5
1 0 2  0 0 1 3
(a)   
0 0 1 3
  0 0 0 0
0 0 0 0
Example 3: Determine which matrix is row equivalent which is reduced row equivalent matrix
1 3 0 0 2 1 0 0 3 0
   
0 0 1 0 0 0 0 1 0 0
a. 0 0 0 1 0 c. 0 0 0 2 0
   
0 0 0 0 0 0 0 0 0 1
0 0 0 0 0  0 0 0 0 0 
 
1 2 3 4 1 0 0 0
   
0 1 9 6 0 1 0 0
b. 0 d.
0 1 9 0 0 1 0
   
0 0 0 0  0 0 0 0 
 

55
APPLIED MATHEMATICS I MODULE

Solution: Matrix in a nad d are reduced row echelon form and matrix in b is row echelon form but
matrix in c is not row echelon form.

Theorem 2.5: Let A be an mxn matrix, then A is row equivalent to an mxn reduced row-echelon
form of a matrix.

Proof: Exercise
 0 2 1 4 
Example- 4: Let A   3 2 0 2  then find the unique reduced row-echelon matrix that is row
 3 3 3 4

equivalent to the matrix A.

3 2 0 2  1 2 3 0 2 3 
 
 0 2  1 4 
1  R2
R1  3 R1
1
A R 0 2  1 4
3 3 3 4 3 3 3 4
 

1 2 3 0 2 3  1 2 3 0 2 3 
 0 2  1 4   0 1 1 2 2 
3  3 R1  R3
R2  2 R2
1
R 
0 1 3 2 0 1 3 2

1 0 13 2 3 
1 0 13 2 3 
   0 1 21 2 
2
R1  R2  R1

 0 1 21 2 
3
R3  7 R3
2
 
0 0 7 2 0
0 0 1 0
R3   R2  R3

1 0 0  2 3 
 0 1 0 2
R1   13
R3  R1
 
0 0 1 0 
R2  12 R3  R2

1 0 0  2 3 
Thus the reduced row-echelon matrix which is equivalent to A is 0 1 0 2 .
0 0 1 0

56
APPLIED MATHEMATICS I MODULE

Activity 2.4

Reduce the following matrices to row echelon and reduced row echelon forms.

2.5. Rank of a Matrix


Defination 2.17: Let A be mxn matrix and U be an echelon or the reduced echelon form of A.The
rank of A is denoted by Rank(A) and is define as the number of non-zero rows of U.
i.e if a matrix A is carried to a row-echelon matrix U by elementary row operations, then the
number of leading 1s in U is called the rank of A.

Example 1: Find the rank of each of the following matrices.

 1 1  2  3  2 1  2
   
A   3 1 1  B   1 1 3 5 
 1 3 4   1 1 1 1 
  
Solution : We transform the matrix A in to row –echelon form by using elementary row operations.

 1 1  2 1 1  2
   
A   3  1 1    0  4 5  , R2  3R1  R2 and R3  R1  R3 ,
 1 3 4   0 4 2 

1 1  2
 
  0  4  5  , R3  R2  R3
 0 0  3
 
1 1  2
 4  1 1
 0 1  , R2   R2 and R3   R3
 5 

4 3
0 1 1 

57
APPLIED MATHEMATICS I MODULE

is row echelon form.Therefore Rank(A)=3


 3  2 1  2  3  2 1  2
   
B   1  1 3 5    0  5 10 13  , R2  3R2  R1 and R3  3R3  R1
 1 1 1 1   0 5 2 1 
   

  3  2 1  2
 
  0  5 10 13 , R3  R2  R3
 0 0 12 12 

 2 12 
1 
 3 3 3 
 13  1
 0
1 1
1 2 R1  R1 , R2   R2 and R3   R3
 5  3 5 12
0 0 1 1 
 
 
 2 1 2 
1 
 3 3 3 
 13 
 0 1 2 , R3  2 R3  R2
 5 
 3 
0 0 0 
 5 

is row of echelon form and also the number of leading 1’s is 3.Therefore Rank(B)=3

Activity 2.5
Find the rank of the following matrix
3 2 1 1 1 1 0 1
2 3 0 1 1 1 2 1 1 1 1 2 2
𝐴 = −1 1 2 1 0 𝐵 = [1 3 0 1 1] 𝐶= 2 1 1
1 1 0 1 1 1 2 1 1 1 −1 0 −1
0 1 1 2 −1 1 1 2

58
APPLIED MATHEMATICS I MODULE

2.6: Inverse of a matrix and its properties


Definition 2.18: Definition of an inverse of a matrix
An 𝑛 × 𝑛 matrix A is invertible (or nonsingular) if there exists an nxn matrix B such that

AB  I n  BA; where I n is the identity matrix of order n. The matrix B is called the
(multiplicative) inverse of A. A matrix that doesn't have an inverse is called noninvertible (or
singular). The inverse of a matrix A is donated by A 1 .
Example-1:
Find the inverse of the following matrices
1 1  2 1 
(a) A  (b) B   
0 1 5 3
Solution:
a. To find the inverse of A. we try to solve the matrix equation 𝐴𝑋 = 𝐼 where X is a matrix with
the same order in A and I is an identity matrix
1 1   x11 x12  1 0
𝐴𝑋 = 𝐼    x x  
0 1  21 22  0 1

 x11  x21 x12  x22  1 0


 
 x21 x22  0 1

Now, by equation the corresponding entries, we obtain the following two systems of linear
equations.
 x11  x 21  1 and  x12  x 22  0
 
 x 21 0  x 22 1
 x11  1, x21  0, x12  1 and x22  1

 x11 x12  1  1 
x   
 x 21 x 22  0 1

1  1 
Therefore, the inverse of A is A 1  x  
0 1

b11 b12 
(b) Suppose B 1    be the inverse of B then B 1B  I 2  B B 1 But,
b22 b22 

B B 1  I 2

59
APPLIED MATHEMATICS I MODULE

2 1  b11 b12  1 0 
  b b   0 1
5 3  21 22   

2b11  b21 2b12  b22  1 0 


   
5b11  3b21 5b12  3b22  0 1

2b11  b21  1 and  2b12  b22  0


 
5b11  3b21  0  5b12  3b22  1
 b11  3, b21  5, b12  1 and b22  2

b11 b12   3  1 
 B 1    =   5 2
 21 22  
b b 
1
In fact, one can check that B B  I 2  B 1 B
Remarks

1. Non square matrices do not have inverses. To see this, note that if A is of order mxn and B is of
order nxm (where m  n) , then the products AB and BA are of different orders and therefore could
not be equal to each other.
1 2 
2. Not all square matrices possess inverse. For example, the matrix   doesn't have inverse
1 2
3. We will see later that square matrix A is invertible if there is a square matrix B such that either
one of the two relations AB  I or BA  I holds, or then B is the inverse of A.
Definition 2.19: An n×n matrix is said to be singular if it does not have a multiplicative inverse.
Notation: Let A be an invertible matrix. We denote its inverse by A-1.
Theorem 2.6: Uniqueness of an Inverses Matrix
If A is an invertible matrix, then its inverse is unique
Proof:
Since A is invertible, we know that it has at least one inverse B such that AB  I  BA. Suppose
that A has another inverse C such that AC I  CA, then
B  I B  (CA) B
 C ( AB); Why ?

60
APPLIED MATHEMATICS I MODULE

 CI j why ?

=C
 B  C And it follows that the inverse of a matrix is unique
Theorem 2.7: Properties of Inverse Matrices
Let A be an invertible matrix, K  N and C is a scalar then the following are true
1 1
(a) ( A )  A
(b) ( Ak ) 1  A1 A1    A1 K factors

(c) (cA) 1  c 1 A1 ; c  0

(d) ( At ) 1  ( A1 ) t

Proof:
(a) Suppose A 1 is the inverse of A
 A1 A  I  A A1

 the inverse of A1 is A ; by def int ion of an inverse

 ( A1 ) 1  A

(c) Since (cA) (c 1 A1 )  (cc 1 ) AA 1 ; by property of scalar multiplication.


 (1) I
I

       
And c 1 A1 cA  c 1c A1 A  1 I  I we have cA c 1 A1  I  c 1 A1 cA

And if follows that c 1 A 1 is the inverse of CA.


 cA  c 1 A1 .
1

As an exercise show property b& d.


Note: For nonsingular matrices, the exponential notion used for repeated multiplication of square
matrices can be extended to include exponents that are negative integers. This may be done by
defying A k to be

A k  A1 A1    A1  k factors 


With this convention we can show that the following properties.

61
APPLIED MATHEMATICS I MODULE

(a) A j Ak  A j k
(b) ( A j ) k  A j k For any integers j and K.
Theorem 2.8: The inverse of a product
Let A and B be invertible matrices of order n, then AB is invertible and
(AB) 1  B 1 A1
Proof:
 
Since ( AB) B 1 A1  A( BB 1 ) A1

 A( I ) A1

 ( AI ) A 1

 AA 1
I

And in a similar way we can show that


( B 1 A1 ) ( AB)  I we have ( AB) CB 1 A1 )  I  ( B 1 A1 ) and hence we get B 1 A1
is the inverse of AB.
 B 1 A1  ( AB) 1 and AB is invertibe.
Remarks
1. The above Theorem says that the inverse of a product of two invertible matrices is the product of
their inverses taken in the reverse order. This can be generalized to include the product of several
invertible matrices.
( A1 A2    An ) 1  An1 An11    A31 A21 A11

2. The inverse of AB is usually not equal to A1 B 1


Theorem 2.9: Cancellation Properties
Let C be an invertible matrix, then the following properties hold.
a. If AC  BC , then A  B ------------- Right cancellation property
b. If
c. CA  CB, then A  B ------------- Left cancellation property
Proof:
(a) Suppose AC  BC , then sin ce C is invertible we have ( AC ) C 1  ( BC ) C 1

62
APPLIED MATHEMATICS I MODULE

 A(C C 1 )  B(C C 1 )
AI BI
 A B
(b) Similar to (a)
Note If C is not invertible then cancellation is not usually valid and hence theorem 4 can be
applied only if C is an invertible Matrix.

Remark: Finding the Inverse of a Matrix by Gauss-Jordan Elimination

Let A be a square matrix of order n


1. Write the 𝑛 × 2𝑛 matrix that consists of the given matrix A on the left and the nxn identity matrix
I on the right to obtain A / I . Note that we separate the matrices A and I by a line. We call this
process adjoining the matrices A and I
2. If possible, row reduce A to I using elementary row operations on the entire matrix A / I . The

 
result will be the matrix I / A 1 . If t his is not possible, then A is not invertible

3. Check your work by multiplying to see that A A1  I  A1 A .


Example-5: Finding the Inverse of a matrix
 1 1 0 
A   1 0  3  then find its inverse.
 6 2 3

Solution:
We begin by adjoining the identity matrix to A to form the matrix

63
APPLIED MATHEMATICS I MODULE

 2  3  1
Therefore, A is invertible and its inverse is A   3  3  1 
1

 2  4  1

Activity 2.6

1. Find the inverse of the following matrix by using Gauss-Jordean elimination method

2. show that the following matrices have an inverse

Note: - Using Gauss-Jordan elimination to find the inverse of a matrix works well (even as a
computer technique) for matrices of order 3x3 or greater. For 2x2 matrices, however, many
people prefer to use a formula for the inverse, rather than find the inverse by Gauss-Jordan
elimination. The formula is given by:

 d b  a b 
A 1  1
abbc  c a  where A  c d  with ad  bc  0.
   
Thus A is invertible if and only if ad  bc  0. . Why?

64
APPLIED MATHEMATICS I MODULE

2.7. Determinant of a matrix and its properties


2.7.1. Determinant of a matrix
Consider a system of linear equations
ax  by  e
cx  dy  f
You recall that, one of the methods available to us to solve such a system was the elimination
method. In this unit, using determinants, we shall exhibit a very efficient computational method
to solve system of linear equations. We first define determinant and study its properties.
Definition 2.20: To every square matrix A with elements from the set of real numbers there is
assigned a specific real number called the determinant of A. It is usually denoted by
de t ( A) or A .

a11 a12    a1n  a11 a12    a1n


a a    a  a a    a2n
Let A   21 22 2n 
then det er min ant of A is denoted by 21 22
  
 
a n1 a n 2    a nn  a n1 a n 2    a nn

Remarks:

(i). Observe that determinant is a function from the set of all square matrices to the set of real numbers,
and it is defined only for square matrix. In fact, if A  (ai j ) is a square matrix over a field F

a ij  F  then A  F . In particular for F  The set of complex numbers then det A  . From

now on wards unless we have stated we consider matrices over R.


(ii).The determinant of an 𝑛 × 𝑛 matrix is called determinant of order n. Before we go to the definition
of determinant of order n(n  N ) we shall consider determinants of order 1,2 & 3.
You recall that a single number a is considered as a 1x1 matrix. Hence
If 𝐴 = [𝑎11 ] then the determinant of A is defined to be the number itself. That is,
det( A)  a11  a11. By definition, if A is invertible, then a11  0 and so det A≠0. Also, conversely

if det (A)≠0, then a11  0 and so, A is invertible.


1) Determinant of order two

65
APPLIED MATHEMATICS I MODULE

a b 
Definition 2.21: The determinant of the 2x2 matrix A    is defined to be the number 𝑎𝑑 −
c d 
a b
𝑏𝑐. That is, det A=  ad  bc
c d

A convenient method for remembering the formula is to note that ad  bc is just the difference of
the products of diagonally opposite entries.
a b
That is, A   ad  bc
c d
Example-1: Find the determinants of the following matrices
2  3 3 1  1 2
(a) A    (b) B    (c ) C   
1 2  6 2 6 4
 1 
  6
d .. 9 
 1 3 
 
 3 

Solution:

2 3
(a) A   (2) (2)  (3) (1)  4  3  7
1 2

3 1
(b) B   (3) (2)  (6) (1)  6  6  0
6 2

1 2
(c) C   (1) (4)  (6) (2)  4  12  8
6 4

 1 
  6 1 1 1 1 1
d. det (d)=det(  9 )  .3  6   
  1 3  9 3 3 2 6
 3 
Remark: Note that the determinant of a matrix A can be positive, zero, or negative.

Definition 2.22: Definition of Minors and cofactors of a matrix


 
Let A  ai j be a square matrix then

66
APPLIED MATHEMATICS I MODULE

th
1-The determinant of the matrix obtained by deleting or removing the i th row and j column of

A is called the minor of the element ai j and it is denoted by M i j .

That is, M i j  Ai j ; where Ai j is a matrix in which the i th row and j column of A are removed.
th

2- The product of the minor of the element ai j and the number (1)
i j
is called the cofactor of

ai j and it is denoted by Ci j , That is, Ci j  (1) i j M i j  (1) i j Ai j

 21 0 
 
Example-2: Find all the minors and cofactors of A   1 1 4 
 3 2 5
th
Solution: To find the minor M i j of ai j , we delete the i th row j column of A and evaluate the

determinant of the resulting matrix


14
Minor of a11  M 11  A11   (1( (5)  (2) (4)  3
25

1 4
Minor of a12  M 12  A12   (1( (5)  (4) (3)  17,
3 5

1 1
Minor of a13  M 13  A13   (1( (5) (1) (2)  (1) (3)  5,
3 2

1 0
Minor of a 21  M 21  A21   (1) (5)  0.2  5,
2 5

2 0
Minor of a 22  M 22  A22   (2) (5)  0(3)  10,
3 5

2 1
Minor of a 23  M 23  A23   (2) (2) (1) (3)  7,
3 2

1 0
Minor of a31  M 31  A31   (1) (4) (0) (1)  4,
1 4

2 0
Minor of a32  M 32  A32   (2) (4) (0) (1)  8,
1 4

2 1
Minor of a33  M 33  A33   (2) (1) (1) (1)  1,
1 1

67
APPLIED MATHEMATICS I MODULE

Now, to find the cofactor Ci j of a i j , we multiply M i j with (1) i  j

C11  (1)11 M 11  3 C21  (1) 21 M 21  5 C31  (1) 31 M 31  4

C12  (1)12 M 12  17 C22  (1) 22 M 22  10 C32  (1) 32 M 32  8

C13  (1)13 M 13  5 C23  (1) 23 M 23  7 C33  (1) 33 M 33  1


Remark:

(i). The minors and cofactors of a matrix differ at most in sign. To obtain the cofactors of a matrix,
first find the minors and then apply the following checker board pattern of +’s and –‘s.
Sign Pattern for Cofactors
     
        
    
    
              
   
         
     
           
 
     
3x3 marix 4 x 4 matrix nxn matrix
Note that odd positions (where 𝑖 + 𝑗 is odd) have negative signs, and even positions (where
𝑖 + 𝑗 is even) have positive signs.

ii) If A is an nxn matrix, then the order of Ai j is (n  1) x(n  1) )

2) Determinant of Order Three


Definition 2.23: The determinant of the 3x3 matrix

a11 a12 a13 


A  a 21 a 22 a 23  is defined to be the number A  a11 C11  a12 C12  a13 C13
a31 a32 a33 
11
= a11 (1) M 11  a12 (1)12 M 12  a13 (1)13 M 13

= a11 A11  a12 A12  a13 A13

a 22 a 23 a 21 a 23 a 21 a 22
= a11  a12  a13
a32 a33 a31 a33 a31 a32

= a11 a22 a33  a12 a23 a31  a13 a21 a32  a11 a23 a32  a12 a21 a33  a13 a22 a31

68
APPLIED MATHEMATICS I MODULE

Remarks: The formula used to compute determinants is known as expansion by a row (column).
Thus, if we consider the first row, then the determinant of A will be equal to the sum of the products
obtained by multiplying the elements of the first row by their respective cofactors.

0 2 1
Example-3: Let A  3  1 2 , then det er min e A
4 0 1

(a) By expanding along the first row


(b) By expanding along the second row
(c) By expanding along the first column
Solution:
1 2 3 2 31
(a) C11    1, C12    (5)  5, and C13   4
0 1 4 1 4 0

Thus, A  a11 C11  a12 C12  a13 C13 , First row exp ansion

= 0(1)  2(5)  1(4)  14

2 1 0 1
(b) Since C 21    2, C 22    4,
0 1 4 1

0 2
And C 23    (8)  8 we have
4 0

A  a21 C21  a22 C22  a23 C23 , sec ond row exp ansion

= 3(2)  (1) (4)  2(8)


=14
1 2 2 1 2 1
(c) Since C11    1, C 21    2 and C31    5 we have
01 0 1 1 2

A  a11 C11  a21C21  a31 C31 , First column exp ansion

= 0(1)  3(2)  4(5)  14


As an exercise try some other possibilities to see that the determinant of A can be evaluated by
expanding by any row or any column. This result is stated formally in the following theorem 4.1,
called Lap lace’s Expansion of a determinant, after the French Mathematician Pierre-Simon
Laplace (1749-1827)

69
APPLIED MATHEMATICS I MODULE

3) Determinant of order n
Definition 2.24: Definition of the Determinant of a Matrix
Let A be a square Matrix of order n where n>2, then the determinant of A is the sum of the entries
in the first row of A multiplied by their cofactors, That is,
n
A   a1 j C1 j  a11 c11  a12 c12      a1n c1n .
j 1

When this definition is used to evaluate a determinant, we say that we are expanding by cofactors.
Theorem 2.10: Expansion by cofactor
Let A be a square matrix of order n, then the determinant of A is given
n
A   ai j ci j  ai1 ci1  ai 2 ci 2      ain cin (i th row exp ansion) or
j 1

n
A   ai j ci j  a1 j c1 j  a2 j c2 j      anj cnj ( j th column exp ansion)
i 1

Proof: Exercise
Note: when expanding by cofactors we do not need to evaluate the cofactors of zero entries,
because a zero entry times its cofactor is zero. That is, ai j ci j  (0) ci j  0 . Thus the row (or

column) containing the most zeros is usually the best choice for expansion by cofactors.

 1 2 3 0
 1 1 0 2 
Example-4: Find the determinant of A  
 0 2 0 3
 
 3 4 0  2

Solution:
Inspecting this matrix, we see that in third column of A we have more number of zeros than the
other rows and columns. Thus we can eliminate some of the work in the expansion by using the
third column.
Therefore,
A  a13 C13  a23 C23  a33 C33  a43 C43

= 3 C13  0(C23 )  0(C33 )  0(C43 )

70
APPLIED MATHEMATICS I MODULE

= 3 C 13
But
1 1 2 1 1 2
1 3
C 13 (1) 0 2 3 0 2 3
3 4 2 3 4 2

Again, expanding by cofactors in the second row we get


1 2 1 2 1 1
C13  (0) (1) 3  (2) (1) 4  (3) (1) 5
4 2 3 2 3 4
= 0  2(1) (4)  3(1) (7) = 13.

Thus we obtain A  3(13)  39

 2 1 4 9
 
 3 2 7 3
Example 5:Caluculate det(  )
0 0 0 0
 
 4 3  2 5 

Solution:Choose row 3 ,since it has the most zeros.
 2 1 4 9
 
 3 2 7 3
det(  ) =0.det( )+ 0.det( )+0.det( )+0.det( )=0
0 0 0 0
 
 4 3  2 5 

This example illustrate es the fact that if amatrix has a row (or column ) containing all zeros the
determinant is zero.
Theorem 2.11: Determinant of a Triangular matrix
Let A  (ai j ) be a triangular matrix of order n, and then its determinant is the product of the

entries on the main diagonal.


Proof: We use mathematical induction to prove this theorem for the case in which A is an upper
triangular matrix. The case in which A is lower triangular can be proven similarly. If A has order
1, then A= a11  and the determinant is given by A  a11 .

Assuming that the theorem is true for any upper triangular matrix of order k-1, we now consider
an upper triangular matrix A of order k. Expanding by the k th row, we obtain
A  o ck1  o ck 2      o ckk 1  akk ckk

71
APPLIED MATHEMATICS I MODULE

= a kk ckk

 a kk (1) k  k M kk  a kk (1) 2 k M kk

 akk M kk

= akk Akk

 a kk a11 a 22 a33    a k 1k 1 , Since Akk is


order k  1 we can apply induction assumpton to it

= a11 a22 a33    akk


This completes the proof
 1 0 0 0 0
 0 3 0 0 0 

Example-6: (a) Let A   0 0 2 0 0 and B  I n
 
 0 0 0 4 0
 0 0 0 0  2

Then since a diagonal matrix is both upper and lower triangular we have
A  (1) (3) (2) (4) (2)  48 and B  (1) (1)    (1)  1  I n

2.7.2. Properties of determinant


It should be clear that calculating the determinant of a large matrix from the definition can be
quite long and cumbersome. For instance, the determinant of a 6x6 matrix involves six 5x5
cofactors. Each of these in turn involves five 4x4 cofactors each of which involves four 3x3
cofactors for a total of 120 x3x3 cofactors to be calculated. Consequently other methods are
often used to compute determinants. These methods depend on the properties of determinants
which h are stated as follows.
Property-1: Elementary Row operations and Determinants
Let A and B be square matrices
(a) If B is obtained from A by interchanging two rows (or columns) of A, then B   A

(b) If B is obtained from A by adding a multiple of a row (or a column)of A to another row (or
c column) A then the determinant is unchanged. That is, B  A .

72
APPLIED MATHEMATICS I MODULE

(c) If B is obtained from A by multiplying a row (or a column) of A by a non zero c constant c,
then B  c A .

Proof: Exercise
Example-1: Show that the determinant of an elementary matrix E of
a. First kind (add a multiple of one row to another) is 1
b. Second kind (row interchange) is -1
c. Third kind (multiply a row by anon zero constant c) is c
Solution: We know that I n  1. Thus, we have

a. E  I n , by property 1 (b)

=1
b. (i) E   I n , by property 1 (a)

= −1.
c. E  c I n , by property 1 (c)

=c.1=c
Example-2: Evaluating a determinant using elementary Row operations.
2  3 10 
Let A  1 2  2 then find det er min ant of A .
0 1  3 

Solution:

73
APPLIED MATHEMATICS I MODULE

2  3 10 1 2 2
A  1 2  2   2  3 10 , by R1  R 2
0 1 3 0 1 3
1 2 2
  0  7 14 , by R2  2 R 1  R2
0 1 3

Note: In the above solution we have changed the given matrix in to triangular matrix.

Property-2: Determinant of a transpose

If A is a square matrix, then A  At

Proof: Exercise (Hint: use mathematical induction and Laplace’s expansion of determinant)
3 1  2

Example-3: Show that A  A for the matrix A   2
t
0 0 
 4  1 5 

Solution: We expand by cofactors along the second row to find A

 3 2  4
A   1 0  1 We expand by cofactors down the second column
t
To find the determinant of
 2 0 5 
  
3 2 4
1 1
to get At  1 0  1  2   23  6
2 5
2 0 5

Thus, A  6  At  A  At

Notation

Let A is a square matrix of order n with columns 𝐴1 , 𝐴2 , 𝐴3 , … , 𝐴𝑗 , … 𝑎𝑛𝑑 𝐴𝑛


And rows 𝐴1 , 𝐴2 , 𝐴3 , … , 𝐴𝑗 , … 𝑎𝑛𝑑 𝐴𝑛 then we can also denote det (A) by
det( 𝐴1 , 𝐴2 , 𝐴3 , … , 𝐴𝑗 , … 𝐴𝑛 ) or det( 𝐴1 , 𝐴2 , 𝐴3 , … , 𝐴𝑗 , … 𝐴𝑛 )
Remarks: - property-1 can be expressed as
1. A. det( 𝐴1 , 𝐴2 , 𝐴3 , … , 𝑐𝐴𝑗 , … 𝐴𝑛 ) = c det( 𝐴1 , 𝐴2 , 𝐴3 , … , 𝐴𝑗 , … 𝐴𝑛 )

74
APPLIED MATHEMATICS I MODULE

B. det( 𝐴1 , 𝐴2 , 𝐴3 , … , 𝑐𝐴𝑗 , … 𝐴𝑛 ) = 𝑐 det( 𝐴1 , 𝐴2 , 𝐴3 , … , 𝐴𝑗 , … 𝐴𝑛 ) where c is a non zero


constant
2. A. det( 𝐴1 , 𝐴2 , 𝐴3 , … , 𝐴𝑗 , … 𝐴𝑛 ) = − det( 𝐴1 , 𝐴2 , 𝐴3 , … , 𝐴𝑗 , … 𝐴𝑛 )
B. det( 𝐴1 , 𝐴2 , 𝐴3 , … , 𝐴𝑗 , … 𝐴𝑛 ) = − det( 𝐴1 , 𝐴2 , 𝐴3 , … , 𝐴𝑗 , … 𝐴𝑛 )
3. A.det( 𝐴1 , 𝐴2 , 𝐴3 , … , 𝑐𝐴𝑗 + 𝐴𝑖 , … 𝐴𝑗 , … 𝐴𝑛 ) = det( 𝐴1 , 𝐴2 , 𝐴3 , … , 𝐴𝑗 , … 𝐴𝑛
B det( 𝐴1 , 𝐴2 , 𝐴3 , … , 𝑐𝐴𝑗 + 𝐴𝑖 , … 𝐴𝑗 , … 𝐴𝑛 ) = det( 𝐴1 , 𝐴2 , 𝐴3 , … , 𝐴𝑗 , … 𝐴𝑛 ) where c is
constant number.

Property-3: Conditions that yield a zero determinant

Let A is a square matrix of order n and any of the following conditions is true, then
A0

(a) An entire row(or an entire column ) consists of zero


(b) Two rows ( or columns) are equal.
(c) One row(or two columns) is a multiple of other row (column )
Proof: Let A be an 𝑛𝑥𝑛 matrix with rows 𝐴1 , 𝐴2 , 𝐴3 , … , 𝐴𝑗 , … 𝐴𝑛
(a) Without loss of generality suppose that the first row of A is a row of zeros then
det(𝐴) = 𝑑𝑒𝑡(0, 𝐴2 , 𝐴3 , … , 𝐴𝑗 , … 𝐴𝑛 )
= 𝑑𝑒𝑡(0, 𝐴2 , 𝐴3 , … , 𝐴𝑗 , … 𝐴𝑛 ) = 0
Hence, the proof.
(b) Without loss of generality assume that 𝐴1 = 𝐴2 then
𝑑𝑒𝑡 (𝐴) = 𝑑𝑒𝑡(𝐴1 , 𝐴2 , 𝐴3 , … , 𝐴𝑗 , … 𝐴𝑛 )
= 𝑑𝑒𝑡(𝐴1 − 𝐴2 , 𝐴2 , 𝐴3 , … , 𝐴𝑗 , … 𝐴𝑛 )
= 𝑑𝑒𝑡(0, 𝐴2 , 𝐴3 , … , 𝐴𝑗 , … 𝐴𝑛 )
=0
(c) Without loss of generality assume that 𝐴1 = 𝑐𝐴2 then
𝑑𝑒𝑡 (𝐴) = 𝑑𝑒𝑡(𝐴1 , 𝐴2 , 𝐴3 , … , 𝐴𝑗 , … 𝐴𝑛 )
= 𝑑𝑒𝑡(𝑐𝐴2 , 𝐴2 , 𝐴3 , … , 𝐴𝑗 , … 𝐴𝑛 )
= 𝑐𝑑𝑒𝑡(𝐴1 , 𝐴2 , 𝐴3 , … , 𝐴𝑗 , … 𝐴𝑛 )
= 0 by (b)

75
APPLIED MATHEMATICS I MODULE

2 4  5 1 2  4 1 2  3

Example- 4: Let A  0 0 0  B  0 1 2  and C   2  1  6 then
   
3  5 2  1 2  4  2 0 6 

(a) Since 𝐴2 is a row of zeros we have A  0

(b) Since𝐵1 = 𝐵3 we have B  0

(c) Since𝐶 3 = −3𝐶 1 we have C  0

Further Properties of determinant


Theorem 1.12.Let A and B be n × n matrices and c be a nonzero scalar.
a) Determinant of a scalar multiple : |cA|=cn|A|
b) Determinant of a product: |AB| = |A||B|
c) Determinant of a transpose: |At| = |A|
1
d) Determinant of an inverse: A 1  (assuming A-1 exists)
A

Proof a)

cA  det cA  det cA1 , cA 2 , cA3 ,..., cA n 
 
 c det A1 , cA 2 , cA3 ,..., cA n

 c det A , A , cA ,..., cA 
2 1 2 3 n

 c det A , A , A ,..., cA 
3 1 2 3 n



 c n det A1 , A 2 , A 3 ,..., A n 
 c det  A  c A
n n

b) Suppose A is invertible le then AA-1 = I and


A  0  AA 1  I

 A A 1  1

 A 1  1
A

Proof of b and c left as exercise

Example- 5: If A is a matrix of order 3 and At  5 then find determinant of 10A

76
APPLIED MATHEMATICS I MODULE

Solution: 10 A  10 3 A  10 3 At  10 3 5  5000

Example-6: uses a determinant to decide whether A is singular or non-singular. If A is invertible,


then find A 1 .

0 2  1  1 0 3
(a) A  3  2 1 (b) A  3  2 1
3 2  1 3 2 1

Solution:
(a) Because A  0 we conclude that A has no inverse

(b) Since A  4  0 we have A is invertible and A 1  1


A
 1
4

Activity 2.7.
 x 1  2 
1. Find all the values of x that make the following determinant zero.  
 x  2 x  1

 1 1  3 
 
2.If A   2 0  4  then |A| = –2. Use this information, together with the properties
 1 1 2 

of determinants, to compute the determinant of the following matrices.
 1 1  3   2 0  4  1 1  3 
     
a)  2 0  4 b) 1  1  3  c) 4  2  10 
 2 2 4   1 1 2   1 1 2 
  
3.If A and B are 3 × 3 matrices and |A|= – 3, |B| = 2, compute the following determinants.
a) AB b) AAT c) ( AB 1 )T

2 0 3 9
 
4 0 1 5
4. Let A   ,then find the following minors and cofactors of A.
5 8 2 5
 
0 3 6 7 

a. M12 and C12 b) M43 and C43.
5. Find the determinant of the following matrix using as little computation as
possible.

77
APPLIED MATHEMATICS I MODULE

 1 2 3 0
 
 4 0 5 0
B
7 3 8 4
 
3 9 3 0 

2.8. Determinant Method of Finding Inverse Matrices


We first introduce tools necessary for developing a formula for the inverse of nonsingular matrix.
Definition 2.25: Let A be an n × n matrix andC ij be the cofactor of aij.The matrix whose (i, j)th
element is Cij is called the matrix of cofactors. The transpose of this matrix is called the adjoint of
A and is denoted adj(A).
T
 C11 C12  C1n   C11 C12  C1n   C11 C21  Cn1 
     
 C21 C22  C2 n   C21 C22  C2 n   C12 C22  Cn 2 
  , then 
               
     
C C  C  C C  C  C C  C 
n1
n 2nn  n1 n2 nn  
1n
2 nnn
matrix of cofactors adjoint matrix

  2 1 3 
 
Example1: Find the matrix of cofactors and the adjoint matrix of the matrix A    4 5 2
  3 1 4
 
Solution:The cofactors of A are
5 2 4 2 4 5
C11   18 C12    10 C13   11
1 4 3 4 3 1
1 3 2 3  2 1
C 21  7 C 22  1 C 23  5
1 4 3 4 3 1
1 3 2 3  2 1
C31   17 C32   8 C33   14
5 2 4 2 4 5
Thus the matrix of cofactor of this matrix is
 18 10 11 
 
 7 1 5 
  17  8  14 
 
and the adjoint of A is the transpose of this matrix

78
APPLIED MATHEMATICS I MODULE

18 7  17 
 
adj ( A)  10 1  8 
 11 5  14 
 
Theorem 1.13: Let be a square matrix with det(A)  0. A is invertable with
1
A1  adj ( A)
A

Proof: as exersise
 a11 a12  a  a12 
Example 2: For a 2x2 matrix   ,the adjoint Adj ( A)   22 
 21 22 
a a  a 21 a11 

1  a22  a12 
If A is nonsingular ,then A1   
a11a22  a12a21  a21 a11 

Example 3: use the result of Theorem 1.25 to compute the inverse of the matrix
  2 1 3 
 
A    4 5 2
  3 1 4
 
Solution: A is computed and found to be -13.This matrix was discussed in example 10.There

we found that
18 7  17 
 
adj ( A)  10 1  8 
 11 5  14 
 

The formula for the inverse of a matrix gives


  18  7 17 
 
 13 13 13 
1  10 1 8 
1
A  adj ( A)  
13  13 13 13 
  11  5 14 
 
 13 13 13 

79
APPLIED MATHEMATICS I MODULE

Activity 2.8
Determine whether the following matrices have inverse. If a matrix has an inverse, find

the inverse using the formula for the inverse of a matrix.

a) b)

We now discuss the relationship between the existence and uniqueness of the solution to a system
of n linear equations in n variables and the determinant of the matrix of coefficient of the system.
2.9. Solving system of linear equations
Definition 2.26: Systems of Linear equations
A system of m linear equations in n variables is a set of m-equations, each of which is linear in the
same n-variables:
a11 x1  a12 x2  a13 x3      a1n xn  b1

a21 x1  a22 x2  a23 x3      a2n xn  b2

a31 x1  a32 x2  a33 x3      a3n xn  b3

   
am1 x1  am2 x2  am3 x3      amn xn  bm

Remark:

The double subscript notation indicates that ai j is the coefficient of x j in the i th equation.

Definition 2.27: A solution of the above system of linear equations is a sequence of numbers
t1 , t 2 , t 3 ,  , t n that is a solution of each of the linear equations in the system.
It can happen that a system of linear equations has exactly one solution, an infinite number of
solutions, or no solution. A system of linear equations is called consistent if it has at least one
solution and inconsistent if it has no solution.

80
APPLIED MATHEMATICS I MODULE

Note: Number of solutions of a system of linear equations

For a system of linear equations in n-variables, precisely one of the following is true.
(a) The system has exactly one solution (consistent system)
(b) The system has an infinite number of solutions (consistent system)
(c) The system has no solution (inconsistent system)
Theorem 2.14: Let AX = B be a system of n linear equations in n variables.
i) If A ≠ 0, then AX = B has a unique solution. The system has a trivial solution that

is X=0 if B = 0.
ii) If A = 0, and at least one of the Ais is nonzero the system has no solution. For, if A

= 0 and |A1| ≠ 0, then X A =|A1| leads to a contradiction. Such systems are

called inconsistent
iii) If A = 0 and Ai  0 , i =1,2,. . . ,n the system my ave an infinite number of

solutions or may not have a solution. A system having an infinite number of solutions is called
dependent.

Notation: Matrix Notation of system of linear equations

One very common use of matrices is to represent a system of linear equations. Matrix notation was
introduced in the nineteenth century to provide a short-hand way of writing linear equations. The
system of m linear equations in n variables
a11 x1  a12 x2      a1n xn  b1

a21 x1  a22 x2      a2n xn  b2

  
am1 x1  am2 x2      amn xn  bm

Can be written in matrix notation as AX  B


Where

81
APPLIED MATHEMATICS I MODULE

a11 a12  a1 n   x1 
  x 
 a 21 a 22  a 2 n 
A , x 
2
     
   
a m1 a m 2  a mn   xn 

b1 
b 
B 2,
 
 
bm 
And AX is the matrix product
Here, in matrix notation AX  B, we have the following definitions.
(1) A  ai j  mxn is called the coefficient matrix of the system.

(2) A and B are column vectors in R n and R m respectively. is unknown and B is a given column
vector.
(3) The matrix derived from the coefficients and constant terms of a system of linear m equations is
called the augmented matrix of the system. That is, the matrix A B  is called the augmented matrix

of the system.
Example-1:Use matrix notation to denote the following system of linear equations. Also find the
coefficient matrix and the augmented matrix of the system.
 x 2  2 x3  2
3x1  4 x2  6 x3  1
Solution:
 x 2  2 x3  2
3x1  4 x2  6 x3  1
0𝑥1 + −1𝑥2 + 2𝑥3 = 2
⇒{
3𝑥1 + 4𝑥2 − 6𝑥3 = 1
𝑥1
0 −1 2 𝑥 2
⟹[ ] [ 2] = [ ]
3 4 −6 𝑥 1
3

82
APPLIED MATHEMATICS I MODULE

is the matrix notation for the given system.

 (ii) A  0  1 2
3 4  6 
 
is the coefficient matrix of the given system and
0  1 2 2
 (iii) A B  
3 4  6 1
is the associated augmented matrix of the system.

2.9.1. Gaussian Elimination and Gauss-Jordan Elimination


I.Gausian Elimination
It is a procedure for solving a system of linear equations.
Rewriting a system of linear equations in row-echelon form usually involves a chain of equivalent
systems, each of which is obtained by using one of the three basic operations. This process is
called Gaussian elimination, after the German mathematician Carl Friedrich Gauss (1777-1855)
Theorem 2.15: Let AX  B be represent the system of linear equations and M  A B be the

block matrix. If 𝑀′ = [𝐴′ |𝐵 ′ ] is row equivalent to M, then the solution of 𝐴′ 𝑥 = 𝐵 ′ are the same
as those of 𝐴𝑥 = 𝐵
Proof: Exercise
II. Gausian Elimination with Back Substitution
The general procedure for using Gaussian elimination with back substitution to solve a system of
linear equations is summarized as follows:
Step-1 Write the augmented matrix of the system
Step- 2 Use elementary row operations to rewrite the augmented matrix in Row-echelon form
Step- 3 Write the system of linear equations corresponding to the matrix in
Row-echelon form, and use back-substitution to find the solution.
Example-2:
Use Gauss Ian elimination with back-substitution to solve the following system.
x2  x3  2 x4  3
x1  2 x2  x3  2
2 x1  4 x2  x3  3x4  2
x1  4 x2  7 x3  x4  19

83
APPLIED MATHEMATICS I MODULE

Solution: Step-1: The augment matrix for this system is M  A B where,

0 1 1 2 3
 
 
M  AB = 
1 2 1
2 4
0 2 
1 3 2
 
1  4  7  1  19

Step 2: using the elementary operations


R1  R2 , R3  2R1  R3 , R4   R1  R4 , R4  6R2  R4 , R3  13 R3 and R4  13
1
R4
respectively we get the row-echelon form of the matrix M,

Which is M '  A' B'

Step-3: The corresponding system of linear equations to the matrix M’ is

and using back substitution, we can determine that the solution is


x1  1, x2  2, x3  1, x4  3
III.Gauss-Jordan Elimination

With Gaussian elimination, we apply elementary row operations to a matrix to obtain a row
equivalent row-echelon form. A second method of elimination called Gauss Jordan elimination
after Carl Gauss and Wilhelm Jordan (1842-1899), Continues the reduction process until a reduced
row-echelon form is obtained.
Example-3: use Gauss-Jordan elimination method to solve the system
x  2 y  3z  9
 x  3 y  4
2 x  5 y  5 z  17

84
APPLIED MATHEMATICS I MODULE

Solution:
Step-1: The associated augmented matrix of the system is

Step-2: You can show that the reduced row-echelon form of the matrix M is given by

Step-3: The corresponding system of linear equations to the matrix M ' is


x  1, y  1 and z  2
Note:

(1) If in the elimination process, you obtain a row with zeros except for the last entry, it is unnecessary
to continue the elimination process. You can simply conclude that the system is inconsistent.
(2) If in the row-echelon form system the number of equations and the number of unknowns are equal
then the system will have a unique solution.
(3) Observe that if in the row-echelon form system the number of equations is less than the number of
unknown, then the system will have infinitely many solutions.

Definition 2.28: Homogeneous system of linear equations


A system of linear equations in which each linear equation is equal to zero is called a homogeneous
system and it is denoted by Ax  0.
It is easy to see that a homogeneous system must have at least one solution. Specifically, if all
variables in a homogeneous system have the value zero, then each of the equations must be
satisfied. Such a solution is called trivial or obvious.
2.9.2. Cramer’s rule
Cramer’s rule
Cramer’s Rule, named after Gabriel Cramer (17 04- 17 52), is a formula that uses determinants
to solve a system of n linear equations in n-variables. This rule can be applied only to systems
of linear equations that have unique solutions.

85
APPLIED MATHEMATICS I MODULE

Theorem 1.15: Cramer’s Rule


If a system of n linear equations in n variables has a coefficient matrix A with a non-zero
determinant A , then the solution to the system

det ( A1, A2 , A j 1, B, A j 1,  , A n )


Ax  B is given by x    1, 2,  , n.
j A j

Proof: Exercise
Example 1: Use Cramer’s Rule to solve the following system of linear equations
for x, y &z
− 𝑥 + 2𝑦 − 3𝑧 = 1
{ 2𝑥 + 𝑧 = 0
3𝑥 – 4𝑦 + 4𝑧 = 2

 1 2  3  x  1
Solution: Let A   2 0 1 , x   y  and 𝐵 = [0], then the system can be represented by
 
 3  4 4  z  2

𝐴𝑥 = 𝐵. And since
−1 2 −3
|𝐴| = | 2 0 1 | = 10 ≠ 0
3 −4 4
Then by Cramer’s rule the system Ax  B has a unique solution given by

( B, A 2 , A 3) (, A1, B, , A 2 ) (, A1, A 2 B)
x  det , y  det and z  det
A A A

1 2 3 1 1  3 1 2 1
0 0 1 2 0 1 2 0 0
2 4 4 3 2 4 3 4 2
 x , y , and z 
10 10 10
x 4
5 , y 3
2 and z  8
5

Thus the solution set for 𝐴𝑥 = 𝐵 is given by

86
APPLIED MATHEMATICS I MODULE

2.9.3 Inverse matrix Method


Thus far, we have studied three methods for solving linear systems: Gaussian elimination , Gauss–
Jordan elimination and Cramer’s rule. The following theorem provides a new method for solving
certain linear systems.
Theorem 1.16: If A is an invertible 𝑛 × 𝑛 matrix, then for each 𝑛 × 1 matrix b, the system of
equations 𝑨𝑥 = 𝒃 has exactly one solution, namely, 𝑥 = 𝐴−1 𝑏
Proof: since 𝐴(𝐴−1 𝑏) = 𝑏 it follows that 𝑥 = 𝐴−1 𝑏 is a solution of 𝐴𝑥 = 𝑏. To show that this is
the only solution. We will assume that 𝑥0 is an arbitrary solution and thaen show that 𝑥0 must be
the solution 𝐴−1 𝑏. If 𝑥0 is any solution then 𝐴𝑥0 = 𝑏. Multiplying both side by 𝐴−1 , we obtain
𝑥0 = 𝐴−1 𝑏.
Example1: solve the following system of linear equation using inverse method
In matrix form this system can be written as 𝐴𝑥 = 𝑏, where
1 2 3 𝑥1 5
𝐴 = [2 5 3], 𝑥 = [𝑥2 ] , 𝑏 = [ 3 ]
1 0 8 𝑥3 17
−40 16 9
A matrix A is invertible and 𝐴−1 = [ 13 −5 −3] (prove!)
5 −2 −1
−40 16 9 5 1
By theorem, 𝑋 = 𝐴−1 𝑏 = [ 13 −5 −3] [ 3 ] = [−1]
5 −2 −1 17 2
Or 𝑥1 = 1, 𝑥2 = −1 𝑎𝑛𝑑 𝑥3 = 2

Activity 2.9
Solve the following system of linear equations

87
APPLIED MATHEMATICS I MODULE

Self-Test Exercise

1. Find the determinant of the following matrices:

2. Let A and B be matrices such that

3. For each of the following compute

4. Use Cramer’s Rule to solve for each of the following systems

a) 𝑥2 + 3𝑥3 = 5 d) 2𝑥1 + 𝑥2 + 𝑥1 = 3
2𝑥1 + 2𝑥2 + 𝑥3 = 11 𝑥1 + 3𝑥2 − 𝑥3 = 7
3𝑥1 + 𝑥2 + 2𝑥3 = 13 𝑥1 + 𝑥2 + 𝑥3 = 1

5. Evaluate the following determinant

2 1 1 0 1
2 1 2 −1 1 3 −1 4
a. b. 0 0 1 2 0 𝑐. |2 5 1|
1 0 3 1 1 2 0 6
2 1 1 2 1

88
APPLIED MATHEMATICS I MODULE

CHAPTER THREE

Limit and Continuity

Introduction:

The idea of a limit lies at the logical foundation of calculus. Practically speaking, it is not necessary
to think about limits when using calculus to solve real-world problems; indeed the full formalism
of limits was not developed until after calculus had been practiced and applied successfully for
hundreds of years. Nevertheless, it is an inspired idea that brings clarity to many ideas that would
otherwise be fuzzy and vague. The quotation above is an excerpt from one of the early critiques of
calculus, when the subject was still done in an informal manner (complete with phrases like
“infinitely small” that had no precise meaning). Limits are a mathematical idealization.

In this chapter we develop the limit first intuitively and then formally. We use limits to describe
the way a function f varies. Some functions vary continuously; small changes in x produces only
small changes in f (x) . Other functions can have values that jump or vary erratically. The notion
of limit gives a precise way to distinguish between these behaviors. The geometric application of
using limits to define the tangent to a curve leads at once to the important concept of the derivative
of a function. The derivative, which we investigate thoroughly in chapter 4, quantifies the way a
functions value’s change.

Objectives

At the end of this chapter the readers should be able to:

 Understand the concept of limit.


 Study and use a formal definition of limit.
 Evaluate a limit using properties of limits.
 Evaluate one-sided limit.
 Determine limit at infinity and infinite limits.
 Determine the horizontal and the vertical asymptote of the graph of a function.
 Determine continuity at a point and continuity on an interval.
 Understand and use the Intermediate Value Theorem.

89
APPLIED MATHEMATICS I MODULE

3.1 Definitions of Limit

We have been evaluating the limit of a function by using its intuitive definition. That is we have
said that limit of 𝑓(𝑥) as 𝑥 approaches to 𝑎 is 𝐿 and write

lim f ( x)  L
xa

if we can make 𝑓(𝑥) close enough to 𝐿 by choosing 𝑥 close enough to 𝑎 but distinct from 𝑎.
Although this intuitive definition is sufficient for solving limit problems it is not prices enough. In

this section we see the formal definition of limit, which we call 𝑡ℎ𝑒   definition of limit.

Definition 3.1(Formal definition of limit)

The limit of 𝑓(𝑥) as 𝑥 approaches 𝑎 is 𝐿, written

lim f ( x)  L
xa

if every   0 , there exists 𝑎   0 such that f ( x)  L   whenever 0  x  a   .

In definition 3.1 above we should not that:

a. The absolute value symbol is read as “the distance between” for instance x  a is the

distance between 𝑥 and 𝑎.


b. Notice that|𝑥 − 𝑎| > 0. In other words 𝑥 is not equal to 𝑎.
So with this in mind we can read the definition as:
“The distance between 𝑓(𝑥) and 𝐿 can be made smaller than any positive number  , whenever the
distance between 𝑥 and 𝑎 is less than some number  and 𝑥 does not equal 𝑎.” Fig 3.1 below
represents this idea pictorially as:

Fig 3.1

90
APPLIED MATHEMATICS I MODULE

If we wish to use a form of Definition 3.1 that does not contain absolute value symbols we can
have the following alternative definition of limit.
Definition 3.2: lim f ( x)  L if and only if for every   0 , there is a   0 such that if 𝑥 is in the
xa

open interval (𝑎 − 𝛿, 𝑎 + 𝛿) and 𝑥 ≠ 𝑎 then 𝑓(𝑥) is in the open interval L   , L    .


Using either of the definitions of limit given above we can prove the following theorem.
Theorem 3.1: If lim f ( x)  L and lim f ( x)  M , then L  M .
xa xa

The above theorem tells us that if a limit of a function 𝑓(𝑥) at 𝑎 exists then it must be unique.
3.2 Examples on Limit
Even if it is very difficult to us the formal definition of limit to handle all limit problems, let us see
how we can use it for evaluating some important limits that may help us in developing rules by the
way of which we can evaluate limits without using the formal definition.
Example 1: Assume that lim 4 x  7  1 . By using properties of inequalities, determine a 𝛿 > 0
x 2

such that
if 0 < |𝑥 − 2| < 𝛿 then |(4𝑥 − 7) − 1| < 0.01
Solution: By considering|(4𝑥 − 7) − 1| < 0.01, we can see that
|(4𝑥 − 7) − 1| < 0.01 ⟺ |4𝑥 − 8| < 0.01
⟺ 4|𝑥 − 2| < 0.01
⟺ |𝑥 − 2| < 0.0025
So now it is clear that if we choose 𝛿 = 0.0025 the statement holds, but to check our result holds
we proceed as follows:

If 0 < |𝑥 − 2| < 𝛿 then |𝑥 − 2| < 0.0025


0.01
⟹ |𝑥 − 2| <
4
⟹ 4|𝑥 − 2| < 0.01
⟹ |4𝑥 − 8| < 0.01
⟹ |(4𝑥 − 7) − 1| < 0.01
Thus we have shown that the choice of 𝛿 = 0.0025 satisfies the statement
if 0 < |𝑥 − 2| < 𝛿 then |(4𝑥 − 7) − 1| < 0.01
This example is for the specific   0.01. The general case can be seen as follows.

91
APPLIED MATHEMATICS I MODULE

Example 2: Show that lim 4 x  7   1.


x 2

Solution:
We need to show that given   0 then there exists 𝛿 > 0 such that
if 0 < |𝑥 − 2| < 𝛿 then |(4𝑥 − 7) − 1| < 
To choose an appropriate 𝛿 we start with |(4𝑥 − 7) − 1| <  then we have
|4𝑥 − 8| <  ⟹ 4|x − 2| < 
𝜖
⟹ |x − 2| <
4
𝜖
Hence, we let 𝛿 =
4

This proves that lim 4 x  7   1.


x 2

Example 3: Prove that lim 7  7.


x 5

Solution:
Begin by letting   0 be given. Find 𝛿 > 0 so that
if 0 < |𝑥 − 5| < 𝛿 then |𝑓(𝑥) − 7| < 
i.e. |7 − 7| <  i.e. |0| <  . But this trivial inequality is always true, no matter what value is
1
chosen for 𝛿. For example 𝛿 = 2 will work.

Thus, if 0 < |𝑥 − 5| < 𝛿 then it follows that |𝑓(𝑥) − 7| <  .


This completes the proof.
A similar proof as example 2 shows us that for any number 𝑎 and 𝑘

lim k  k . ………………………….. (1)


x k

Example 4: Prove that lim kx  ka for any real number 𝑘.


x a

Solution: from (1) it is clear that if 𝑘 = 0


lim kx  lim 0  0  0.a  k .a
x a x a

If 𝑘 ≠ 0, letting   0 we must find a 𝛿 > 0 so that


0 < |𝑥 − 𝑎| ⟹ |𝑘𝑥 − 𝑘𝑎| < 

Since,

92
APPLIED MATHEMATICS I MODULE

|𝑘𝑥 − 𝑘𝑎| <  ⟹ |𝑘||𝑥 − 𝑎| < 



⟹ |𝑥 − 𝑎| <
k


Now, choose 𝛿 =
k

Thus, lim kx  ka for any real number𝑘.


x a

Example 5: Prove that lim


x 1
x  3  4
2

Solution:
Begin by letting   0 be given. Find 𝛿 > 0 (which depends on  ) so that
If 0 < |𝑥 − 1| < 𝛿, then |𝑓(𝑥) − 4| <  . Begin with |𝑓(𝑥) − 4| <  and “solve for” |𝑥 − 1|.
Then
|𝑓(𝑥) − 4| <  ⟺ |(𝑥 2 + 3) − 4| < 
⟺ |𝑥 2 − 1| < 
⟺ |(𝑥 − 1)(𝑥 + 1)| < 
⟺ |𝑥 − 1||𝑥 + 1| < 
We will now replace the term |𝑥 + 1| with an appropriate constant and keep the term |𝑥 − 1|, since
this is the term we wish to “solve for.” To do this, we will arbitrary assume that 𝛿 ≤ 1 [This is a
valid assumption to make since, in general, once we find a 𝛿 that works, all smaller values of a 𝛿
also work].
Then
|𝑥 − 1| < 𝛿 ≤ 1 ⟹ −1 < 𝑥 − 1 < 1
⟹0<𝑥<2
⟹ 1 < |𝑥 + 1| < 3
It follows that
|𝑥 − 1||𝑥 + 1| < |𝑥 − 1|(3) < 
⟺ 3|𝑥 − 1| < 

⟺ |𝑥 − 1| <
3

93
APPLIED MATHEMATICS I MODULE


Now choose 𝛿 = 𝑚𝑖𝑛 {1, }
3
Thus, if 0 < |𝑥 − 1| < 𝛿, it follows that |𝑓(𝑥) − 4| <  .

Hence, lim
x 1
x  3  4
2

2 1
Example 6: Prove that lim 
x 3 x3 3
Solution: Begin by letting   0 be given. Find 𝛿 > 0 (which depends on  ) so that
1 1
If 0 < |𝑥 − 3| < 𝛿, then |𝑓(𝑥) − 3| <  . Begin with |𝑓(𝑥) − 3| <  and

“solve for”|𝑥 − 3|. Then


1 2 1
|𝑓(𝑥) − | <  ⟺ | − |< 
3 𝑥+3 3
3 2 1𝑥 +3
⟺| − |< 
3𝑥 + 3 3𝑥 +3
6 − (𝑥 + 3)
⟺| |< 
3(𝑥 + 3)
|3 − 𝑥|
⟺ <
|3||𝑥 + 3|
|𝑥 − 3|
⟺ <
|3||𝑥 + 3|
1 |𝑥 − 3|
⟺ <
3 |𝑥 + 3|
1 1
⟺ |𝑥 − 3| <
3 |𝑥 + 3|
We will now replace the term |𝑥 + 3| with an appropriate constant and keep the term |𝑥 − 3|, since
this is the term we wish to “solve for.” To do this, we will arbitrary assume that 𝛿 ≤ 1 [This is a
valid assumption to make since, in general, once we find a 𝛿 that works, all smaller values of a 𝛿
also work].
Then
|𝑥 − 3| < 𝛿 ≤ 1 ⟹ −1 < 𝑥 − 3 < 1
⟹2<𝑥<4
⟹ 5 < |𝑥 + 3| < 7

94
APPLIED MATHEMATICS I MODULE

1 1 1
⟹ < <
7 |𝑥 + 3| 5
It follows that
1 1 1 1
|𝑥 − 3| < |𝑥 − 3| < 
3 |𝑥 + 3| 3 5
1
⟺ |𝑥 − 3| < 
15
⟺ |𝑥 − 3| < 15 
Now choose 𝛿 = 𝑚𝑖𝑛{1,15  }
1
Thus, if 0 < |𝑥 − 3| < 𝛿, it follows that |𝑓(𝑥) − | <  .
3

2 1
Hence, lim 
x 3 x3 3

Example 7: Prove that 


lim 2  x  5
x9

Solution:
Begin by letting   0 be given. Find 𝛿 > 0 (which depends on  ) so that
If 0 < |𝑥 − 9| < 𝛿, then |𝑓(𝑥) − 5| <  . Begin with |𝑓(𝑥) − 5| < 
and “solve for” |𝑥 − 9|. Then
|𝑓(𝑥) − 5| <  ⟺ |(2 + √𝑥) − 5| < 

⟺ |√𝑥 − 3| < 

√𝑥 + 3
⟺ |(√𝑥 − 3) |< 
√𝑥 + 3
𝑥−9
⟺| |< 
√𝑥 + 3
1
⟺ |𝑥 − 9| <
|√𝑥 + 3|
We will now replace the term |√𝑥 + 3| with an appropriate constant and keep the term |𝑥 − 9|,
since this is the term we wish to “solve for.” To do this, we will arbitrary assume that 𝛿 ≤ 1 [This
is a valid assumption to make since, in general, once we find a 𝛿 that works, all smaller values of
a 𝛿 also work].

95
APPLIED MATHEMATICS I MODULE

Then
|𝑥 − 9| < 𝛿 ≤ 1 ⟹ −1 < 𝑥 − 9 < 1
⟹ 8 < 𝑥 < 10
⟹ √8 < √𝑥 < √10
⟹ √8 + 3 < |√𝑥 + 3| < √10 + 3
1 1 1
⟹ < <
√10 + 3 |√𝑥 + 3| √8 + 3
It follows that
1 1
|𝑥 − 9| < |𝑥 − 9| <
|√𝑥 + 3| √8 + 3
⟺ |𝑥 − 9| < (√8 + 3) 

Now choose 𝛿 = 𝑚𝑖𝑛{1, (√8 + 3)𝜀}


Thus, if 0 < |𝑥 − 9| < 𝛿, it follows that|𝑓(𝑥) − 5| <  .

Hence, 
lim 2  x  5
x9

1
Example 8: Prove that lim 3x sin 0
x0 x
Solution:
We need to show that given   0 , then there exists 𝛿 > 0 such that
1
0 < |𝑥 − 0| < 𝛿 implies|3𝑥𝑠𝑖𝑛 𝑥 − 0| <  . For”|𝑥 − 9|.

Looking for 𝛿
1
|3𝑥𝑠𝑖𝑛 | < 
𝑥

1
⟺ |3𝑥| |𝑠𝑖𝑛 | < 
𝑥
1 1
⟺ 3|𝑥| |𝑠𝑖𝑛 | ≤ 3|𝑥|. 1 <  ; since |𝑠𝑖𝑛 | ≤ 1
𝑥 𝑥

⟺ |𝑥| <
3

96
APPLIED MATHEMATICS I MODULE


Hence we let 𝛿 =
3

1
Thus, lim 3x sin 0
x0 x

Negation of the Existence of a Limit


Next we present an example of a function that does not have a limit at a certain point. For a function
𝑓 not to have real number𝐿, the statement “𝐿 is the limit at𝑎” is false. What does it mean for that
statement to be false? By definition 3.1, “is the limit of 𝑓 at 𝑎” means that:
For every   0 there is a number 𝛿 > 0 such that
if 0 < |𝑥 − 𝑎| < 𝛿, then |𝑓(𝑥) − 𝐿| < 
For this statement to be false, there must be some   0 such that for every 𝛿 > 0 it is false that
if 0 < |𝑥 − 𝑎| < 𝛿, then |𝑓(𝑥) − 𝐿| <  …………………………………….… (2)
But, to say that (2) is false is the same as to say that there must be a number 𝑥 such that
0 < |𝑥 − 𝑎| < 𝛿 and |𝑓(𝑥) − 𝐿| ≥ 

Thus to say that the statement lim f ( x)  L is false is the same as to say that there
x a

is some  >0such that for every 𝛿 > 0 there is a number 𝑥 satisfying


0 < |𝑥 − 𝑎| < 𝛿 and |𝑓(𝑥) − 𝐿| ≥ 
Example 9: Let 𝑓 be defined by
𝑥2 ; 𝑓𝑜𝑟 𝑥 > 0
𝑓(𝑥) = {
−1; 𝑓𝑜𝑟 𝑥 ≤ 0
Show that lim f ( x) does not exist
x0

Solution:
Let 𝐿 be any number. We will prove that the statement “𝐿 is the limit of 𝑓 at 0” is false by letting
1
 and showing that for any 𝛿 > 0 there is an 𝑥 satisfying
2
1
0 < |𝑥 − 𝑎| < 𝛿 and |𝑓(𝑥) − 𝐿| ≥ 2 = 

Let 𝛿 be any positive number.


1 𝛿
If 𝐿 ≤ − 2, then we let 𝑥 = and note that 𝑓(𝑥) = 𝑥 2 , so that
2

97
APPLIED MATHEMATICS I MODULE

𝛿2 𝛿2 1 1
|𝑓(𝑥) − 𝐿| = | − 𝐿| ≥ | + | > = 
4 4 2 2
1 𝛿
If 𝐿 ≥ − 2, then we let 𝑥 = − 2 and note that 𝑓(𝑥) = −1, so that
1 1
|𝑓(𝑥) − 𝐿| = |−1 − 𝐿| = |1 + 𝐿| ≥ |1 − | = = 
2 2
In either case we have shown that for any 𝛿 > 0 there is an 𝑥 satisfying
1
0 < |𝑥 − 𝑎| < 𝛿 and |𝑓(𝑥) − 𝐿| ≥ 2 = 

Therefore, 𝑓 has no limit at 0. (𝑖. 𝑒. lim f ( x) 𝑑𝑜𝑒𝑠 𝑛𝑜𝑡 𝑒𝑥𝑖𝑠𝑡)


x0

A Activity 3.1

1. Using the definition of limit, prove that

a. c.

b. d.

3.3 One-Sided Limit


The notion of limit discussed in the preceding sections can be extended to one-sided limit as we
can see from the definition below.
Definition 3.3
a. A number 𝐿 is the right-hand limit of 𝒇 at 𝒂 denoted by lim f ( x)  L
x a

if for every   0 there is a number   0 such that


if 0  x  a   , then f ( x)  L  

b. A number 𝐿 is the left-hand limit of 𝒇 at 𝒂 denoted by lim f ( x)  L


x a

if for every   0 there is a number   0 such that


if    x  a  0 , then f ( x)  L  

98
APPLIED MATHEMATICS I MODULE

Example: Show that lim x  1  0


x 1

Solution: Let   0 be given. We need to show that there is a   0 such that

if 0  x  1   , then x 1  0  

from x 1   squaring both sides we get 0  x  1   2

Hence choose   
2

Then

if 0  x  1   , then x 1  0  x 1    

 Below we give a theorem that relates one-sided limit with the general limit the student can
see Robert Ellis and Danny Gulick for the proof of the theorem.


Activity 3.2
1. Show that

a. b.

Theorem 3.2: lim f ( x) exists and is equal to 𝐿 if and only if lim f ( x)  L and lim f ( x)  L
xa x a x a

both exists and both are equal to 𝐿.


Example : Observe that in Example 1 even if the right hand side of 𝑓 at 1 exists since the left side
limit of 𝑓 at 1 does not exist, as the function is not defined for 𝑥 < 1 then

lim x  1 does not exist.


x 1

99
APPLIED MATHEMATICS I MODULE

3.4 Infinite Limits and Infinite Limits at Infinity


According to Definition 3.1 if a function 𝑓 has a limit 𝐿 at 𝑎 then 𝐿 is a real number, so if the value
of a function 𝑓 becomes larger and larger in absolute value as 𝑥 approaches 𝑎 from the right or
from the left of 𝑎 the 𝑓 has no limit at 𝑎. Now we introduce a definition that addresses such a case.

Infinite Limits
Definition 3.4 Let 𝑓 be defined on some open interval(𝑎, 𝑐).
a. If ∀𝑁, ∃𝛿 > 0 such that
if 0 < 𝑥 − 𝑎 < 𝛿 then 𝑓(𝑥) > 𝑁
Then lim f ( x)  
x a

b. If ∀𝑁, ∃𝛿 > 0 such that


if 0 < 𝑥 − 𝑎 < 𝛿 then 𝑓(𝑥) < 𝑁
Then lim f ( x)  
x a

c. In either case (𝑎) or (𝑏) the vertical line 𝑥 = 𝑎 is called a vertical asymptote of the graph
of 𝑓, and we say that 𝑓 has an infinite right- hand limit at 𝒂.
There are analogous definitions for the limits
lim f ( x)   and lim f ( x)  
xa x a

Note if lim f ( x)  lim   then we write simply lim   for the common expression and say
x a x a x a

that the limit of 𝑓(𝑥) as 𝑥 approaches 𝑎 is ∞ and that 𝑓 has an infinite limit at 𝑎.
Example: Show that lim 1   . Show also that the line 𝑥 = 0 is a vertical asymptote of the graph
x 0 2
x
of 1 .
2
x
Solution: Observe that for any 𝑁 > 0,

if 0  x  1 , then 1
N
2
N x
Thus lim 1   , and that the line 𝑥 = 0 is a vertical asymptote of the graph of 1 .
2  2
x 0 x x

100
APPLIED MATHEMATICS I MODULE

Once more for any 𝑁 > 0, if 


1
 x 0, then 1
N
2
N x
Thus lim 1   , and that the line 𝑥 = 0 is a vertical asymptote of the graph of 1 .
2  2
x0 x x
1 1 1
Finally since lim 2
   lim 2
, we have lim 2
.
x 0 x 0 x 0
x x x
Limits at Infinity
Until now the limits we have seen have been limits of a function 𝑓 at a number 𝑎. Now we
consider the limit of 𝑓 as 𝑥 becomes larger and larger in absolute value.

Definition 3.5
a. lim f ( x)  L if for every   0 there is a number 𝑀 such that
x

if 𝑥 > 𝑀, then f ( x)  L  

b. lim f ( x)  L if for every   0 there is a number 𝑀 such that


x

if 𝑥 < 𝑀, then f ( x)  L  

c. If either lim f ( x)  L or lim f ( x)  L , then we call the horizontal line 𝑦 = 𝐿 a


x x

horizontal asymptote of the graph of 𝑓.


1 1
Example: Show that lim 2
 0 and lim 2
0
x  x 
x x
1
Solution: Let   0 . To show that lim 2
 0 we must find an 𝑀 such that
x 
x
1 1 1
if 𝑥 > 𝑀, then 2
0  2
 2

x x x
But then
1 1
If x  , then 
 x
2

1 1
Therefore we let M  and conclude that lim  0.
 x 
x
2

101
APPLIED MATHEMATICS I MODULE

1 1
To show that lim  0 , we simply choose M  
x 
x
2

Then 𝑀 < 0, and thus

1 1 1
if 𝑥 < 𝑀, then 2
0  2
 2

x x M
1
This proves that lim 2
 0.
x 
x
1
Note here that the line 𝑦 = 0 is the horizontal asymptote of the graph of 2
.
x

A Activity 3.3
1. Find the vertical asymptote (if any) of the graph of the following functions.

a. d.

b. e.

c. f.

2. Find the horizontal asymptote (if any) of the graph of the following functions.

a. c.

b. d.

Infinite Limits at Infinity


We now see the last possible formal definition of limit that is not considered yet.
Definition 3.6 lim f ( x)   if for any real number 𝑁 there is some number 𝑀 such that
x

102
APPLIED MATHEMATICS I MODULE

If 𝑥 > 𝑀, then 𝑓(𝑥) > 𝑁

Note: the definition of


lim f ( x)   , lim f ( x)   , and lim f ( x)   are completely analogous.
x x x

Example: Show that lim x  


3

x

 x for x  1.
3
Solution: We use the fact that x
For any 𝑁, choose 𝑀 so that 𝑀 > 1 and 𝑀 > 𝑁. Then it follows that
If 𝑥 > 𝑀, then 𝑥 3 > 𝑥 > 𝑀 > 𝑁

Therefore by definition 3.6 , we have lim x  


3

x

Similarly, we conclude that for any positive integer 𝑛, lim x   .


n

x

3.5 Basic Limit Theorems


Even if we have developed important techniques of solving limit problems by using the formal
definition, I hope by now we have realized that it is not that easy to use this definition to solve
each and every problem. Nevertheless the student had encountered in his or her earlier studies of
calculus rather easy ways of evaluating limits by the help of different rules. Here we state and
prove some of them by using Definition 3.1 and use them to evaluate more complex limit cases.
Theorem 3.3 Assume that lim f ( x)  L and lim g ( x)  M , and 𝑐 is a constant. Then
xa xa

1. lim f ( x)  g ( x)  lim f ( x)  lim g ( x)  L  M


xa xa xa

2. lim cf ( x)  cL
xa

3. lim f ( x) g ( x)  lim f ( x).lim g ( x)  L.M


xa xa xa

 f ( x)  lim f ( x) L
4. lim   x a
 ; provided that 𝑀 ≠ 0.
x a g ( x ) 
  lim
x a
g ( x) M

Proof: Here we proof (1). Statement (2), (3), 𝑎𝑛𝑑 (4) are left as exercise.
Let lim f ( x)  L and lim g ( x)  M .
xa xa

We need to show for every 𝜀 > 0 there is some 𝛿 > 0 such that
if 0 < |𝑥 − 𝑎| < 𝛿, then |𝑓(𝑥) + 𝑔(𝑥) − (𝐿 + 𝑀)| < 𝜀

103
APPLIED MATHEMATICS I MODULE

𝜀
Observe that lim f ( x)  L if and only if for every > 0 there is some 𝛿1 > 0 such that
xa 2
𝜀
if 0 < |𝑥 − 𝑎| < 𝛿1 , then |𝑓(𝑥) − 𝐿| < 2
𝜀
Similarly lim g ( x)  M if and only if for every 2 > 0 there is some 𝛿2 > 0 such that
xa

𝜀
if 0 < |𝑥 − 𝑎| < 𝛿2 , then |𝑔(𝑥) − 𝑀| < 2

Let 𝛿 = 𝑚𝑖𝑛{𝛿1 , 𝛿2 }. Then we can see that


𝜀 𝜀
if 0 < |𝑥 − 𝑎| < 𝛿, then |𝑓(𝑥) + 𝑔(𝑥) − (𝐿 + 𝑀)| ≤ |𝑓(𝑥) − 𝐿| + |𝑔(𝑥) − 𝐿| < 2 + 2 = 𝜀

Thus, lim f ( x)  g ( x)  lim f ( x)  lim g ( x)  L  M


xa xa xa

In addition to these rules you have also seen that for instance if 𝑓 is a polynomial or a rational
function and 𝑎 is in the domain of 𝑓, then
lim f ( x)  f (a)
xa

Now let as quickly go through some important limit finding techniques that would require a little
bit of caution before applying the rules in Theorem 3.3.

4
2

Example 1: Find lim x


x 2 x2

4
2

Solution: Direct substitution of 2 in x implies that we have 0⁄0 which is indeterminate. Thus
x2
we cannot use Theorem 3.9(4) but for 𝑥 ≠ 0 simplification of the rational expression would lead
us to

 4 x  2x  2
2
x   x2
x2 x2

4
2

Thus, lim x  limx  2  4


x 2 x2 x 2

Example 2: Find lim x


x 0
1 1
2
x
Solution: Again here we cannot use Theorem3.9(4), as we get from direct substitution the
indeterminate0⁄0. But for 𝑥 ≠ 0 rationalizing the denominator we have:

104
APPLIED MATHEMATICS I MODULE

1 1
2 2 2

lim x  lim x x
x 0 x 0
 1 1  1 1 1 1
2 2 2
x x x
x  x  1  1
2 2

 lim
x 0
x  112

x  x  1  1
2 2

 lim 2
x 0
x
 lim x   1  1  2
2 2

x0  x 

Example 3: Find lim x x


x0

Solution: Observe that



 x , if x  0
2

xx  2
 x , if x  0

Since x x  x for x  0 , we have


2

lim x x  lim x  0
2

x0 x0

And also x x   x for x  0 , we have


2

lim x x  lim  x  0
2

x0 x0

Therefore, we conclude that


lim x x  0 .
x0

x 1
Example 4: Prove that lim does not exist.
x1 x 1

Solution:
x 1 x 1 x 1 x 1
lim  lim   lim 1  1 and lim   lim   lim   1  1
x1 x  1 x1 x  1 x1 x1 x  1 x1   x  1 x1

Consequently

105
APPLIED MATHEMATICS I MODULE

x 1 x 1
lim   lim 
x 1 x  1 x1 x  1

x 1
Thus, lim does not exist.
x1 x 1

lim x  2 x x  2x
2 2

Example 5: Find and lim


x 1 x 1
x  2 x  2

x  2x
2
2
Solution: Dividing the numerator and the denominator of by x in the limit we have
x 1
2

1
2
lim x  2 x
2

 lim x  2
x 1
2
x  x  1
1
x
Similarly
1
2
x  2x
2

lim  lim x  2
x 1
2
x  x  1
1
x

x  2x
2
Observe that y  2 is the horizontal asymptote of the graph of f ( x)  .
x 1
2

x  2x
2
Example 6: Let f ( x)  . Find all vertical asymptotes of the graph of f .
x 1
2

Solution: Since f is not defined at x  1 and x  1 they are the possible vertical asymptotes but
to confirm our claim we use limit:

x  2x
2
x 1  2x
lim  lim   and
x1 x  1 x  1
x 1
2
x1

x  2x
2
x 1  2x
lim   lim  
x 1 x  1 x  1
x 1
2
x 1

It follows that x  1 and x  1 are the vertical asymptotes of the graph of f .


The next theorems give two additional properties of limits. For their proofs the student may
refer any major calculus books.
Theorem 3.4 If f ( x)  g ( x) for all 𝑥 in an open interval that contains 𝑎 (except possibly at
𝑎) and the limits of 𝑓 and 𝑔 both exist as 𝑥 approaches𝑎, then

106
APPLIED MATHEMATICS I MODULE

lim f ( x)  g ( x)
xa

Theorem 3.5 (The squeezing Theorem)


If f ( x)  g ( x)  h( x) for all 𝑥 in an open interval that contains 𝑎 (except possibly at 𝑎)
and lim f ( x)  lim h( x)  L . Then
xa xa

lim g ( x)  L .
xa

I don’t think the student is new for these theorems and for the special limit that is the
Consequence of especially the Squeezing Theorem. i.e.
sin x
lim 1
x 0 x
 sin x     sin x 
sin ce, cos x  x  1 for  2 , 2  . Taking both sides lim it as x  0, we get lim  1
   x0 x 
(you can see the detail proof on the book Robert Ellis)
1
Example 1: Find lim x sin .
x 0 x
1
Solution: Since  1  sin  1, x  0 , we have
x
1
 x  x sin  x, x  0
x
Moreover lim  x  0 and lim x  0
x 0 x 0

Thus, by squeezing theorem we have


1
lim x sin  0.
x 0 x


4 2

Example 2: Find lim x x .


x  x 1

x x
4 2

Solution: Simplifying we can evaluate the limit as below


x 1


lim x x
4 2

 lim x x
2 2
1  lim x x  1x 1  lim
2

x x  1  
2

x x 1 x x 1 x x 1 x

107
APPLIED MATHEMATICS I MODULE

Activity 3.4

1. Evaluate each of the following limits, if it exists.

a. d.

b. e. where

c. f.

3.6 Continuity of a Function and the Intermediate Value Theorem


Definition 3.7 A function 𝑓 is continuous at a number 𝑎 in its domain if
lim f ( x)  f (a)
xa

𝑓 is said to be discontinuous at 𝑎 if 𝑓 is not continuous at 𝑎.


Notice that Definition 3.7 implicitly requires three things if 𝑓 is continuous at 𝑎:
i. f (a) is defined (that is, 𝑎 is in the domain 𝑓 ).
ii. lim f ( x) exists (so 𝑓 must be defined on an open interval that contains 𝑎).
xa

iii. lim f ( x)  f (a) .


xa

x2
2

Example 1: Let f ( x)  lim x . Determine the number at which 𝑓 is discontinuous.


x1 x 1
Solution: Notice that 𝑓 is a rational function. Since the denominator of 𝑓 is 0 for 𝑥 = −1, 𝑓 is
defined for all 𝑥 except at −1. Thus 𝑓 is discontinuous only at 𝑥 = −1 else where it is continuous
in its domain.
Example 2: If we redefine the function 𝑓 in Example 21 as:

108
APPLIED MATHEMATICS I MODULE

 2x2
x
f ( x)   x  1 , if x  1
 3 , if x  1

Then since

 x2 x  2x  1  3
2

lim f ( x)  lim x  lim


x1 x1 x 1 x1 x 1
and hence lim f ( x)  3  f (1)
x1

Thus, 𝑓 is continuous.
Notice that we are able to make 𝑓 in Example 1 to be continuous by redefining it at – 1 as
in Example 2. Such discontinuity points like – 1 in our example are called removable
discontinuities because we can remove the discontinuity of the function by redefining the
function just at the discontinuity point. Otherwise it is called non-removable discontinuity.
1 1 , if x  0
Example 3: Let f ( x)  and g ( x)   . Then we can see that, 𝑓 is not
x
2
0 , if x  0

defined at 0 and lim f ( x)   , g is defined at 0 but lim g ( x) does not exist as lim g ( x)  1 and
x0 x0 x0

lim g ( x)  0 .
x0
Thus, both functions are not continuous at 0.
We say we have infinite discontinuity at 0 in case of 𝑓 while we say we have jump discontinuity
at 0 in case of g
Clearly combinations of continuous functions follow immediately from the corresponding results
for limits.
Theorem 3.6 If 𝑓 and 𝑔 are continuous at 𝑎 and 𝑐 is a constant, then the following
functions are also continuous at 𝑎.
i. 𝑓±𝑔 iii. 𝑓𝑔
𝑓
ii. 𝑐𝑓 iv. , if 𝑔(𝑎) ≠ 0
𝑔

So using Theorem 3.6 we can show that every polynomial function is continuous over ℝ. Every
rational function is continuous every where except at numbers where the denominator is 0.

109
APPLIED MATHEMATICS I MODULE

Another way of combining continuous functions 𝑓 and 𝑔 to get a new continuous function is to
form the composite function f  g . This fact is a consequence of the following theorem.
Theorem 3.7 If 𝑓 continuous at 𝑏 and lim g ( x)  b , then
xa


lim f g ( x)  f (b)  f lim g ( x)
xa xa

The following theorem tells us that the composition of two continuous functions at a given number
is continuous.
Theorem 3.8 If g is continuous at 𝑎 and f is continuous at g (a) , then

 f  g ( x)  f g ( x) is continuous at 𝑎.

A Activity 3.5
Where are the following functions continuous?

a. b.

3.6.1 One-Sided Continuity


Definition 3.8:
-a function f is continuous is continuous from the right at a point 𝑎 in its
domain if
lim f ( x)  f (a)
xa
-a function f is continuous is continuous from the left at a point 𝑎 in its domain if

lim f ( x)  f (a)
x a
 1 , if x  0
Example : The step function g ( x)   is continuous from he left at 0.
0 , if x  0
Since lim g ( x)  1  g (0) but it is not continuous from the right at 0 as lim g ( x)  g (0) . (verify!)
x0 x0

110
APPLIED MATHEMATICS I MODULE

3.6.2 Continuity on an Interval

Definition 3.9:
a) a function f is continuous on an open interval a, b  , if it is continuous at every point in a, b  .

b) a function f is continuous on a closed interval a, b , if it is continuous at every point in a, b 


and is also continuous from he right at 𝑎 and continuous from the left at 𝑏.

Example : Discuss the continuity of f ( x)  1  x .


2

Solution: The domain of f is the closed interval  1,1 .

At all points in the open interval  1,1 , f is continuous.


Moreover,

lim  1  x  0  f (1) …… continuous from the right


2

x1

and

lim 1  x  0  f (1) ………. Continuous from he left


2

x1

Thus, f is continuous on the closed interval  1,1

A Activity 3.6

Let . Show that is continuous on .

An important property of continuous functions is expressed by the following theorem.

Theorem 3.9 [The Intermediate Value Theorem (IVT)]


Suppose that f is continuous on the closed interval a, b and let N be any number strictly

between f (a) and f (b) . Then there exists a number c in a, b  such that f (c)  N .

In other words, if f is continuous on the closed interval a, b and if f (a) and f (b) differ or

opposite in sign , then there exists atleast one real root c in a, b  such that f (c)  0 .

111
APPLIED MATHEMATICS I MODULE

Example : Show that there is a root of the equation

4x  6x  3x  2  0 on 1,2 .
3 2

f ( x)  4x 6x  3x  2 .
3 2
Solution: Let

We are looking for a solution of the given equation, that is, a number c between 1 and 2 such
that f (c)  0 .
Therefore we take a  1, b  2, and N  0 in Theorem 3.17. We have
f (1)  4  6  3  2  1  0 and f (2)  32  24  6  2  12  0
Thus, f (1)  0  f (2) , that is, N  0 is a number between f (1) and f (2) .
Also f is continuous, since it is a polynomial.
Hence, by the IVT, there is a number c between 1 and 2 such that f (c)  0 .

4x  6x  3x  2  0 has a root c in the interval 1,2 .


3 2
In other words, the equation

Activity 3.7
1. Find the constant 𝑎, or the constant 𝑏, or the constant 𝑐 such that the function is
continuous on the entire real line.

a. c.

b. d.

2. Demonstrate that the equation has atleast one solution.


3. Verify that the Intermediate Value Theorem applies to the give interval and find
the value of guaranteed by the theorem.

a.

b.

112
APPLIED MATHEMATICS I MODULE

Self-Test Exercise
1. Find the limit . Then use the definition to prove that the limit is .

a. c.

b. d.

2. Evaluate the following limits (if it exists)

a) f.

b) g.

c) h.

d) i.

e) j.

3. Evaluate the following limits (if it exists)

a) c)

b) d)

4. Find the one-sided limit (if it exists)

a. d.

b. e.

113
APPLIED MATHEMATICS I MODULE

5. Find the values of the constants such that

6. Determine the value of such that the function is continuous on the entire real number line

7. Determine the values of such that the function is continuous on the entire real
number line

8. Determine all values of the constant such that the following function is continuous for all
real numbers

9. Determine the intervals on which the following functions are continuous

a) c)

b) d)

10. Find the vertical and horizontal asymptotes of the graph of the following functions (if it exists)

a) c)

b) d)

11. Use the Intermediate Value Theorem to show that

has a zero in the interval .

114
APPLIED MATHEMATICS I MODULE

CHAPTER FOUR
Derivatives

Introduction

In this chapter we will start looking at the next major topic in a calculus. We will be looking at
derivatives in this chapter (as well as the next chapter). This chapter is devoted almost
exclusively to finding derivatives. We will be looking at one application of them in this chapter.
We will be leaving most of the applications of derivatives to the next chapter.

Objectives

At the end of this chapter the readers should be able to:

 Find the slope of the tangent line to a curve at a point.


 Use the limit definition to find the derivative of a function.
 Understand the relationship between differentiability and continuity.
 Explain different properties of differentiation rules.
 Find the derivative of a function using the differentiation rules.
 Find the derivative of a composite function using the chain rule.
 Distinguish between functions written in implicit form and explicit form.
 Use implicit differentiation to find the derivative of a function.
4.1 The Tangent Lines

What does it mean to say that a line is tangent to a curve at a point? For a circle, the tangent line
at point P is the line that is perpendicular to the radial line at point P , as shown in figure 4.1 . For
a general curve, however, the problem is more difficult. For example, how would you define the
tangent lines shown in figure 4.2 ?

You might say that line is tangent to a curve at point P if it touches, but does not cross, the curve
at point P . This definition would work for the first curve shown in figure 4.2 , but not for the
second. Or you might say that a line is tangent to a curve if the line touches or intersects at exactly
one point-----this definition would work for a circle but not for more general curves, as the third
curve in figure 4.2 shows.

115
APPLIED MATHEMATICS I MODULE

Figure 4.1 Figure 4.2

Tangent line to a circle Tangent line to a curve at a point

Essentially, the problem of finding the tangent line at a point P boils down to the problem of
finding the slope of the tangent line at P . You can approximate this slope using a secant line
through the point of tangency and a second point on the curve, as shown in figure 4.3 . If c, f (c) 
is the point of tangency and c  x, f (c  x) is a second point on the graph of f , then the slope
of the secant line through the two points is given by

f (c  x)  f (c) f (c  x)  f (c) change in y


m     Slope of secant line
sec
(c  x)  c x change in x

Figure 4.3

The secant line through c, f (c)  and c  x, f (c  x)

You can obtain more and more accurate approximations to the slope of the tangent line by choosing
points closer and closer to the point of tangency, as shown in figure 4.4 .

116
APPLIED MATHEMATICS I MODULE

As x  0 from the left, the secant As x  0 from the right, the secant
line approaches the tangent line. line approaches the tangent line.

Figure 4.4

4.2 Definition of Derivatives

Definition: If f is defined on an open interval containing c , and if the limit

y f (c  x)  f (c)
lim  lim m
x0 x x 0 x

exists, then the line passing through c, f (c)  with slope m is the tangent line to the graph of f
at the point c, f (c)  .

Example 1: Find the slope of the graph of f ( x)  2 x  5 at the point 3,1 .

Solution: To find the slope, m , of the graph of f when c  3 , we can apply the definition of the
slope of the tangent lines, as follows

m  lim
f (3  x)  f (3)
 lim
2(3  x)  5  2(3)  5
x0 x x 0 x

6  2x  5  6  5 2x
 lim  lim 2
x0 x x0 x

Thus, the slope of f at 3,1 is m  2 .

117
APPLIED MATHEMATICS I MODULE

Example 2: Find the equation of the tangent line to the graph of f ( x)  x  7 at the point 2,3
2

Solution: First we have to find the slope, m , of the graph of f when c  2 , we can apply the
definition of the slope of the tangent lines, as follows

f (2  x)  f (2)

 2 x  (3)  (2)  (3)
2 2

m  lim  lim 
x0 x x0 x

4  4x  (x)  3  4  3 4x  (x)


2 2

 lim  lim
x 0 x x 0 x

 lim 4  x   4
x0

Hence, the equation of the tangent line to the graph of f at the point 2,3 is

y  f (c)  m( x  c)  y  (3)  4( x  2)

This becomes, y  4 x  11 .

'
Definition: The derivative of a function f at a number a , denoted by f (a) , is defined as

' f ( x)  f ( a )
f (a)  lim ……………...……….….. (1)
x a xa

provided that this limit exists.

If we write x  a  h , then x  a  h and x approaches a if and only if h approaches to 0 .

Thus, equivalent way of stating the definition of the derivative is

' f ( a  h)  f ( a )
f (a)  lim …………………………… (2)
h0 h

This last definition is more convenient for finding the derivative of a function.

118
APPLIED MATHEMATICS I MODULE

'
Note: In addition to f ( x) , other notations are used to denote the derivative of y  f (x) . The most

common are:

dy d
 f ( x), D y
' '
f ( x) ,
dx
, y, dx x

dy
The notation is read as “the derivative of y with respect to x .” Using limit notation, we can
dx
write

dy y f ( x  x)  f ( x) '
 lim  lim  f ( x)
dx x 0 x x 0 x

Example 1: Use the definitions of derivatives find the derivative of 𝑓 if 𝑓(𝑥) = √𝑥 − 1,


State the domain of 𝑓 ′ .
𝑓(𝑥+ℎ)−𝑓(𝑥) √𝑥+ℎ−1+√𝑥−1 √𝑥+ℎ−1−√𝑥−1 √𝑥+ℎ−1+√𝑥−1
Solution: 𝑓 ′ (𝑥) = lim = lim = lim .
ℎ→0 ℎ ℎ→0 ℎ ℎ→0 ℎ √𝑥+ℎ−1+√𝑥−1
(𝑥+ℎ−1)−(𝑥−1) 1 1 1
= lim ℎ(√𝑥+ℎ−1+√𝑥−1)= lim = = 2√𝑥−1
ℎ→0 ℎ→0 √𝑥+ℎ−1+√𝑥−1 √𝑥−1+√𝑥−1

We see that 𝑓 ′ (𝑥) exists if 𝑥 > 1 , so the domain of 𝑓 ′ is (1, ∞).

Example 2: Find the derivative of the function f ( x)  x  3x  2 at  1 .


2

Solution: By definition, we have

' f (1  h)  f (1)


f (1)  lim
h0 h

Thus,

'
(1)  lim
f (1  h)  f (1)
 lim 

 1 h  3(1  h)  2  1  31  2
2 2

f h0 h h o h

1  2h  h  3  3h  2
2

 lim
ho h

hh
2

 lim  lim 1  h   1
ho h h0

119
APPLIED MATHEMATICS I MODULE

We hope the student remembers that the slope of the tangent line to the graph of the function

f at a point a, f (a) is given by the derivative of 𝑓 at 𝑎 i.e


'
f (a) . Consequently, using the

Point-slope form of the equation of a line, we have the equation of the tangent line to the curve
y  f (x) at a point a, f (a) is given by

(a)x  a  .
'
y  f ( a)  f

For instance, the equation of the tangent line to the graph of f ( x)  x  3x  2 at  1,0 in our
2

Example 1 is given by

(1)x  (1)  y  0  1x  1  y  x  1


'
y  f (1)  f
'
Given a function f , we associate with it a new function f , is called the derivative of f , and

defined by:

' f ( x  h)  f ( x )
f ( x)  lim
h0 h

' '
We know that the value of f at x , f ( x) , can be interpreted geometrically as the slope of the

tangent line to the graph of f at the point x, f ( x) .

Example 3: Find the derivative of the function f ( x)  x  x  1 .

Solution: By definition of derivative, we have

' f ( x  h)  f ( x )
f ( x)  lim
h0 h

Thus,

120
APPLIED MATHEMATICS I MODULE

f
'
( x)  lim
x  h)  
x  h 1  x  x 1 
h0 h

h x  h   1  x 1
 lim
h0 h

h  x  h   1  x 1 .   xx  hh  11  x 1 
x  1
 lim , rationalize the radicals.
h0 h

 1 
 lim 1  
h0
 x  h  1  x  1 

1
 1
2 x 1

'
Definition 4.2: A function f is differentiable at a if f (a) exists. It is differentiable on an

open interval a, b or a,  or  , a  or  ,  if it is differentiable at every number in the
interval.

Example 1: Where is the function 𝑓(𝑥) = |𝑥| differentiable?


Solution: If 𝑥 > 0, then
|𝑥+ℎ|−|𝑥| (𝑥+ℎ)−𝑥 ℎ
𝑓 ′ (𝑥) = lim = lim = lim ℎ = lim 1 = 1
ℎ→0 ℎ ℎ→0 ℎ ℎ→0 ℎ→0

and so 𝑓 is differentiable for any 𝑥 > 0.


Similarly, for 𝑥 < 0 we have |𝑥| = −𝑥. Then
|𝑥+ℎ|−|𝑥| −(𝑥+ℎ)−(−𝑥) −ℎ
𝑓 ′ (𝑥) = lim = lim = lim = lim (−1) = −1
ℎ→0 ℎ ℎ→0 ℎ ℎ→0 ℎ ℎ→0

and so 𝑓 is differentiable for any 𝑥 < 0.


𝑓 ′ (0) does not exist. (Why?)
Theorem 4.3 If f is differentiable at a , then f is continuous at a .

Proof: To prove that f is continuous at a , we have to show that lim f ( x)  f (a) .


xa

121
APPLIED MATHEMATICS I MODULE

We do this by showing that the difference f ( x)  f (a) approaches 0.

For x  a we can divide and multiply by x  a .

We did this in order to involve the difference quotient.

Thus, we can use the product law of limits to write

f ( x)  f ( a )
lim f ( x)  f (a)  lim x  a 
x a xa xa

 f ( x)  f ( a ) 
 lim  . lim x  a 
x a
 xa x a

'
 f (a).0  0

Therefore,

lim f ( x)  lim f (a)  f ( x)  f (a)


xa xa

 lim f (a)  lim f ( x)  f (a)


xa xa

 f ( a )  0  f ( a)

Hence, f is continuous at a .

Note: The converse of Theorem 4.3 is false; that is, there are functions that are continuous but
not differentiable at a point.

For instance, consider the function f ( x)  x .

Clearly, f ( x)  x is continuous at 0 because

lim f ( x)  lim x  0  f (0) .


x0 x0

But, as we have seen in Example 1 that f is not differentiable at 0 .

122
APPLIED MATHEMATICS I MODULE

Let us remind you some of the differentiation rules that you have developed in your high school
calculus course. We advise the student to check on these results using the definition of derivative.

Derivatives of Polynomials and Exponential Functions


Constant Functions
𝑑
Derivative of a constant function (𝑐) = 0
𝑑𝑥

Power Functions
The Power Rule: If n is any real number, then
𝑑
(𝑥 𝑛 ) = 𝑛𝑥 𝑛−1
𝑑𝑥

Example 1. (a) If 𝑓(𝑥) = 𝑥 6 , then𝑓 ′ (𝑥) = 6𝑥 5 . (b) If 𝑦 = 𝑥1000 ,then 𝑦 ′ = 1000𝑥 999
𝑑𝑦 𝑑
(c) If𝑦 = 𝑡 4 , then 𝑑𝑡 = 4𝑡 3 . (d) 𝑑𝑟 (𝑟 3 ) = 3𝑟 2 .
1 𝑑𝑦 2 3
(𝑒) 𝑓(𝑥) = ⟹ = − 𝑥3 (f) 𝑦 = √𝑥 2 Exercise
𝑥2 𝑑𝑥

Exponential Functions
The Derivative of the exponential function𝑓(𝑥) = 𝑎 𝑥 :
𝑓 ′ (𝑥) = 𝑎 𝑥 𝑙𝑛𝑎

The Derivative of the exponential function𝑓(𝑥) = 𝑎 𝑥 :


𝑓 ′ (𝑥) = 𝑎 𝑥 𝑙𝑛𝑎

𝑑
Example 11: 𝑑𝑥 (2𝑥 ) = 2𝑥 𝑙𝑛2.

Definition of the Number e


𝑒 ℎ −1
e is the number such that lim =1
ℎ→0 ℎ

Note: e ≈2.71828
Derivative of the Natural Exponential Function
𝑑
(𝑒 𝑥 ) = 𝑒 𝑥
𝑑𝑥

Example 2: If 𝑓(𝑥) = 𝑒 𝑥 − 𝑥, find 𝑓 ′ .

123
APPLIED MATHEMATICS I MODULE

Derivatives of a Logarithmic Function


𝑑 1 𝑑 1
(log 𝑎 𝑥) = 𝑥 ln 𝑎 and 𝑑𝑥 (ln 𝑥) = 𝑥
𝑑𝑥

Example 3: Differentiate 𝑦 = ln(𝑥 3 + 1).

Derivatives of Trigonometric Functions


Example 4: Use definitions of derivatives show that the derivatives of 𝑓(𝑥) = 𝑠𝑖𝑛𝑥 is
𝑓 ′ (𝑥) = 𝑐𝑜𝑠𝑥
Derivatives of Trigonometric Functions
𝑑 𝑑
(𝑠𝑖𝑛𝑥) = 𝑐𝑜𝑠𝑥 (𝑐𝑠𝑐𝑥) = −𝑐𝑠𝑐𝑥𝑐𝑜𝑡𝑥
𝑑𝑥 𝑑𝑥
𝑑 𝑑
(𝑐𝑜𝑠𝑥) = −𝑠𝑖𝑛𝑥 (𝑠𝑒𝑐𝑥) = 𝑠𝑒𝑐𝑥𝑡𝑎𝑛𝑥
𝑑𝑥 𝑑𝑥
𝑑 𝑑
(𝑡𝑎𝑛𝑥) = 𝑠𝑒𝑐 2 𝑥 (𝑐𝑜𝑡𝑥) = −𝑐𝑠𝑐 2 𝑥
𝑑𝑥 𝑑𝑥

𝑠𝑒𝑐𝑥
Example 5: Differentiate 𝑓(𝑥) = 1+𝑡𝑎𝑛𝑥.
𝒔𝒆𝒄𝒙(𝒕𝒂𝒏𝒙−𝟏)
Solution: 𝒇′ (𝒙) = (𝟏+𝒕𝒂𝒏𝒙)𝟐

4.3 Basic Differentiation Rules

Theorem: If f and g are differentiable , then

a. cf  (x)  c f (x) , where c is an arbitrary constant.


' '

b.  f  g  ( x)  f ( x)  g ( x )
' ' '

c.  fg  (x)  f (x)g(x)  f (x) g (x)


' ' '

'
f
' '
f ( x) g ( x)  f ( x) g ( x)
  ( x)  ; Provided that g ( x  0) .
g ( x)
d. 2

g

124
APPLIED MATHEMATICS I MODULE

Activity 4.1
1. Find the derivative of each of the following functions.

a. f.

b. g.

c. h.

d. i.

e. j.

4.4 The Chain Rule


The rules that we have introduced till now are not enough to find composition of functions. Thus,
we need to develop an appropriate technique to handle these cases.

g ( x) and f g ( x) both


' '
Theorem 4.5 [The Chain Rule]: If the derivatives,

exists, then

 f  g ( x)  f ( g(x)). g ( x)
' ' '

Example 1: Find 𝐹 ′ (𝑥) if 𝐹(𝑥) = √𝑥 2 + 1.


Solution: We expressed 𝐹 as 𝐹(𝑋) = (𝑓𝑜𝑔)(𝑥) = 𝑓(𝑔(𝑥)) 𝑤ℎ𝑒𝑟𝑒 𝑓(𝑢) = √𝑢 𝑎𝑛𝑑 𝑔(𝑥) =
𝑥 2 + 1.
Since
1 −1⁄ 1
𝑓 ′ (𝑢) = 2 𝑢 2 =2 and 𝑔′ (𝑥) = 2𝑥
√𝑢
1 𝑥
Then 𝐹 ′ (𝑥) = 𝑓 ′ (𝑔(𝑥))𝑔′ (𝑥) = . 2𝑥 = √𝑥 2
2√𝑥 2 +1 +1

125
APPLIED MATHEMATICS I MODULE

h ( x) if h( x)  cos 2x .
'
Example 2: Find

Solution: Let f ( x)  cos x and g ( x)  2 x .


' '
Then, h  f  g . Since also g ( x)  2 and f ( x)   sin x

We conclude that

h ( x )   f  g  ( x)  f ( g ( x)). g ( x)   sin 2 x 2  2 sin 2 x


' ' ' '

h ( x) if h( x)  1 x .
' 2
Example 3: Find

Solution: Let g ( x)  1  x and f ( x)  x .consequently h  f  g .


2

' ' 1
Then g ( x)  2 x and f ( x) 
2 x
for 𝑥 > 0.

Therefore,

h ( x)  f g ( x).g ( x) 
' ' 1 x
.2 x 
'
.
2 1 x 1 x
2 2

Activity 4.2
1. Find the first derivative of the following functions

a. c.

b. d.

2. Find the equation of the tangent line to the curve at the given point.

a.

b.

126
APPLIED MATHEMATICS I MODULE

4.5 Higher Order Derivatives


If 𝑓 is a differentiable function, then its derivative 𝑓′ is also a function, so 𝑓′ may have a derivative
of its own, denoted by (𝑓 ′ )′ = 𝑓′′ . This new function 𝑓′′ is called the second derivative of 𝑓
because it is the derivative of 𝑓 ′ .Using Leibniz notation; we write the second derivative of 𝑦 =
𝑑 𝑑𝑦 𝑑2 𝑦
𝑓(𝑥) as ( ) = 𝑑𝑥 2
𝑑𝑥 𝑑𝑥

Another notation is 𝑓 ′′ (𝑥) = 𝐷2 𝑓(𝑥).


The third derivative 𝒇′′′ is the derivative of the second derivative: 𝑓 ′′′ = (𝑓 ′′ )′.
𝑑 𝑑2 𝑦 𝑑3 𝑦
𝑦 ′′′ = 𝑓 ′′′ (𝑥) = 𝑑𝑥 (𝑑𝑥 2 ) = 𝑑𝑥 3 = 𝐷3 𝑓(𝑥)

The process can be continued. The fourth derivative 𝑓′′′′ is usually denoted by 𝑓 (4) . In general, the
𝑛𝑡ℎ derivative of 𝑓 is denoted by 𝑓 (𝑛) and is obtained𝑓 from by differentiating 𝑛 times. If 𝑦 =
𝑓(𝑥), then we write
𝑑𝑛 𝑦
𝑦 (𝑛) = 𝑓 (𝑛) (𝑥) = 𝑑𝑥 𝑛 = 𝐷𝑛 𝑓(𝑥)
''
Example 1: Let f ( x)  cos 2 x . Then find a formula for f ( x) .
'
Solution: Since f ( x)  2 sin 2 x , it follows that
''
f ( x)  4 cos 2 x .
''
Example 2: Let f ( x)  x . Then find a formula for f ( x) .

' 1
Solution: Since f ( x)  , it follows that
2 x
'' 1 32
f ( x)  
4x
.

th
In general, for any positive integer n  3 we can define the n derivative of f at a , denoted by
(n)
f (a) as
( n 1) ( n 1)
(n) f ( x)  f (a)
f (a)  lim
x a xa
Whenever this limit exists.

127
APPLIED MATHEMATICS I MODULE

Note: The second derivative, the third derivative, and so on are called higher order derivatives,
to distinguish them from the first derivative.
x 1 (n)
Example 3: Let f ( x)  . Then find a formula for f ( x) .
x 1

(1) 2(1!) , it follows that


1
' 2
f ( x)   
x 1 x 1
Solution: Using quotient rule, 2 11

(1) 2(2!) ,
2
'' 4
f ( x)  
x 1 x 1 3 21

(1) 2(3!) ,
3
''' 12
f ( x)   
x 1 x 1 4 31

(1) 2(4!)
4
( 4) 48
f ( x)  
x 1 x 1 5 41

.
.
.

Continuing this process we get

(1) 2(n!) ; for n  1


n
(n)
f ( x) 
x 1 n 1

(1) 2(10!)  2(10!) .


10
(10)
f (2) 
21
For instance, 101

128
APPLIED MATHEMATICS I MODULE

Activity 4.3

1.Let . Then find all higher derivatives of .

2.Let . Then find .

3. Let be a polynomial function of degree with leading coefficient , then


find
a) b) c)

4. Find a formula for the derivative of , if


a. b.

5. If for all , then what is the relationship between

4.6 Implicit Differentiation


The following problems require the use of implicit differentiation. Implicit differentiation is
nothing more than a special case of the well-known chain rule for derivatives. The majority of
differentiation problems in pre-calculus involve functions y written explicitly as functions of x .
For example, if

y  3x  sin(7 x  5)
2

Then the derivative of y is


' dy
y  dx  6 x  7 cos(7 x  5) .
However, some functions y are written IMPLICITLY as functions of x . A familiar example of
this is the equation
2
x y  25 ,
2

Which represents a circle of radius five centered at the origin. Suppose that we wish to find the
slope of the line tangent to the gragh of this equation at the point 3,4 .

129
APPLIED MATHEMATICS I MODULE

How could we find the derivative of y in this instance? One way is to first write y explicitly as
a funcion of x . Thus,

25 x
2
x y  25  y  
2 2
.

Where the positive square root represents the top semi-circle and the negative square root
represents the bottom semi-circle. Since the point 3,4 lies on the bottom semi-circle given by

y 25 x
2

The derivative of y is
' dy x
y  dx 
25 x
2

Thus, the slope , m , of the line tangent to the gragh at the point 3,4 is

m  y  dx 
' dy 3 3.

253 4
2

Unfortunately, not every equation involving x and y can be solved explicitly for y . For the sake
of illustration we will find the derivative of y WITHOUT writing y explicitly as a function of
2
x . Recall that the derivative of a function of x squared, ( f ( x)) , can be found using the chain
rule:

( x))   2 f ( x) dx  f ( x)  2 f ( x) f


d  2 d '

dx 
(f ( x)

2
Since y symbolically represents a function of x , the derivative of y can be found in the same

fashion:

 2 y y  2 y y .
d  2 d '
y
dx   dx
2
x y  25 . Differentiate both sides of the equation, getting
2
Now begin with

130
APPLIED MATHEMATICS I MODULE

d  2
dx  x
 y
2
  d 25  d
 dx dx
x  dxd  y   dxd 25
2 2

And so
'
2x  2 y y  0
'
Hence, 2 y y  2 x

dy ' x
Thus,  y  y.
dx
Therefore, the slope of the line tangent to the gragh at the point 3,4 is
' dy  (3) 3
m y  dx  (4)  4 .
This second mehod illustrates the process of implicit differentiation. It is important to note that the
derivative expression for explicit differentiation involves x only, while the derivative expression
for implicit differentiation may involve BOTH x AND y .
The following problems range in difficulty from average to challenging.
𝑑𝑦
Example 1: If 𝑥 2 + 𝑦 2 = 25, then find .
𝑑𝑥

Solution: Differentiate both sides of the equation 𝑥 2 + 𝑦 2 = 25 :


𝑑 𝑑
( 𝑥2 + 𝑦2) = (25)
𝑑𝑥 𝑑𝑥
𝑑 𝑑
(𝑥 2 ) + (𝑦 2 ) = 0
𝑑𝑥 𝑑𝑥
𝑑 𝑑 𝑑𝑦 𝑑𝑦
Since (𝑦 2 ) = (𝑦 2 ) = 2𝑦 𝑑𝑥
𝑑𝑥 𝑑𝑦 𝑑𝑥
𝑑𝑦
Thus 2𝑥 + 2𝑦 𝑑𝑥 = 0
𝑑𝑦 𝑥
= −𝑦
𝑑𝑥

dy 3
x y 4.
3
Example 2: Using implici differentiation find for
dx
Solution: Differentiating both sides of the equation, getting
d  3

dx  x  y
3
  d 4  d
 dx dx
x  dxd  y   dxd 4
3 3

3x  3 y y  0  3 y y  3x
2 2 ' 2 ' 2
So that,

131
APPLIED MATHEMATICS I MODULE

  x2 .
' dy
Thus, y 
dx y
Example 3: Using implici differentiation find
dy
dx
for x  y  x  y  1 .
2

Solution: Differentiating both sides of the equation, getting


d 
x  y   dxd x  y 1  dxd x  y   dxd x  dxd  y  dxd 1 .
dx 
2 2

Applying the chain rule on


d 
x  y  , we get
dx 
2

d 
dx 
x  y   2x  y  dxd x  y  dxd x  y   2x  y 1  dydx 
2 2

So that
 dy 
2x  y 1    1   0  2( x  y)  2( x  y)  1 
dy dy dy
 dx  dx dx dx

 2( x  y)  1  2( x  y)  1  2( x  y)  1
dy dy dy
Hence, 2( x  y) 
dx dx dx
dy
Thus, solving for , we get
dx
dy 2( x  y )  1
 .
dx 2( x  y )  1

dy
Example 4: Using implici differentiation find for y  sin3x  4 y  .
dx
Solution: Differentiating both sides of the equation, getting
d
 y   d sin3x  4 y   dy  cos(3x  4 y). d 3x  4 y 
dx dx dx dx
dy d d  dy dy
So that  cos(3x  4 y)  (3x)  (3 y)   3 cos(3x  4 y)  4 cos(3x  4 y) .
dx  dx dx  dx dx
dy
Solving for , we get
dx
dy 3 cos(3x  4 y)
 .
dx 1  4 cos(3x  4 y )

132
APPLIED MATHEMATICS I MODULE

dy 3 2
for y  x y  x y .
2 3
Example 5: Using implici differentiation find
dx
Solution: Differentiating both sides of the equation, getting
d
 y   d  x2 y  x3 y   dy  d  x2 y   d  x3 y 
3 2 3 2

dx dx   dx dx   dx  
Using product rule, we have
dy  d

dx  dx
x  y  x dxd  y    dxd x  y  x dxd  y 
2 3 2 3 3 2 3 2

3 2 dy 2 3 dy
 2 xy  3x  3x y  2 x y
2 2
y dx dx
dy
Factoring out , we get
dx
dy 
  2 x y   2xy  3x y
2 2 3 3 2 2

dx 1 3x y 
3 2
2xy  3x y
2
dy
Thus,  .
13x y  2x
2 2 3
dx y

x   y  x  9
3
at x  1. .
2
Example 6: Find an equation of the tangent line to the gragh of

x   y  x  9 . If
2 3
Solution: Begin with x  1, then

1  y 1  9  y  3
2 3

Hence the tangent line passes through the point 1,3 .


Now by using implicit differentiation, we have
d  2
dx  x
  y  x   dxd 9  2x  3 y  x  dydx 1  0
3 2

dy 
3  y  x   3 y  x
2 2
 2x 
dy 3 y  x  2 x

 
2

 
So that .
dx  dx 3 yx
2

Thus, the slope , m , of the tangent line to the gragh at 1,3 is

m 

dy 3 31  2(1) 10 5


 .
2

dx 3 31
2
12 6 
133
APPLIED MATHEMATICS I MODULE

And the equation of the tangent line is

y 3 
5
x  1  y  5 x  13 .
6 6 6
2

Example 7: Using implici differentiation find d y 2


or y
''
for x y
4 4
 25 .
dx
Solution: Differentiating both sides of the equation implicitly with respect to x , we get
3

  x3 .
3 dy dy
4x  4 y 0
3

dx dx y
2

To find d y 2
or y
''
we differentiate this expression for
dy
using the quotient rule and
dx dx
remembering that y is a function of x :

2 
d y2  y''  d  
3 
x3    y
3 d
x  x dxd  y 
3 3 3

y 
dx
dx dx  
y  3
2

y 3x  x  3 y . dx 


3 2 3 2 dy
 6
y
dy
If we now substitute the value of into this expression, we get
dx
 
y 3x  x   x
3
3 2 3 
3
2  y 
d y   
dx
2

y  3
2

3x  x  y 
2 4 4

 
7
y
25 , since
x y
2 4
  3x  25
4
7
y
−75𝑥 2
=
𝑦7

134
APPLIED MATHEMATICS I MODULE

Activity 4.4

1. Using implicit differentiation find for

a. d.

b. e.

c. f.

2. Find an equation of the tangent line o he gragh of at the point.

3. Find the slope of the gragh at the point .

4. Using implicit differentiation find for

a. c.

b. d.

4.7. Derivatives of Inverse Trigonometric Functions


Arcsine Function:
Recall the definition of the arcsine function:
𝜋 𝜋
𝑦 = 𝑠𝑖𝑛−1 𝑥 ⇒ sin 𝑦 = 𝑥, where − 2 ≤ 𝑦 ≤ 2

Differentiating sin 𝑦 = 𝑥 implicitly with respect to x, we obtain


𝑑𝑦 𝑑𝑦 1
cos 𝑦
=1 𝑜𝑟 =
𝑑𝑥 𝑑𝑥 cos 𝑦
Now cos 𝑦 ≥ 0, 𝑠𝑖𝑛𝑐𝑒 −𝜋⁄2 ≤ 𝑦 ≤ 𝜋⁄2 , 𝑠𝑜

cos 𝑦 = √1 − 𝑠𝑖𝑛2 𝑦 = √1 − 𝑥 2
𝑑𝑦 1 1
Thus = cos 𝑦 = √1−𝑥 2
𝑑𝑥

𝑑 1
(𝑠𝑖𝑛−1 𝑥) =
𝑑𝑥 √1 − 𝑥 2

135
APPLIED MATHEMATICS I MODULE

Derivatives of Inverse Trigonometric Functions


𝒅 𝟏 𝒅 𝟏
(𝒔𝒊𝒏−𝟏 𝒙) = (𝒄𝒔𝒄−𝟏 𝒙) = −
𝒅𝒙 √𝟏 − 𝒙𝟐 𝒅𝒙 𝒙√𝒙𝟐 − 𝟏
𝒅 𝟏 𝒅 𝟏
(𝒄𝒐𝒔−𝟏 𝒙) = − (𝒔𝒆𝒄−𝟏 𝒙) =
𝒅𝒙 √𝟏 − 𝒙𝟐 𝒅𝒙 𝒙√𝒙𝟐 − 𝟏
𝒅 𝟏 𝒅 𝟏
(𝒕𝒂𝒏−𝟏 𝒙) = (𝒄𝒐𝒕−𝟏 𝒙) = −
𝒅𝒙 𝟏 + 𝒙𝟐 𝒅𝒙 𝟏 + 𝒙𝟐

Example : Find

a.
d
dx

arcsin 3x
2
 c.
d
dx

arctan e
2x

b.
d
dx
arccos(ln x) d.
d
dx

arc sec 3x
2

Solution:
a. d

arcsin 3x 
2
 1 d
3x  2 6x .
1 3x 2 19x
dx 2 dx 4

b. d
arccos(ln x)   1 d
ln x    1 .
dx
1 ln x dx
1 ln x
2 2
x

  e   2e2 x
2x
d 1 d
arctan e 
2x 2x

1 e  1 e 
c. 2 2
.
dx 2x dx

d. d

arc sec 3x 
2
 1 d
3x  
2 2
dx
3x
2
3x2 1 2 dx x 9x
4
1

136
APPLIED MATHEMATICS I MODULE

Activity 4.5
Find the derivative of the following functions and simplify where possible

a. d.

b. e.

c. f.

Hyperbolic Functions
Definition of the Hyperbolic Functions
𝒆𝒙 −𝒆−𝒙 𝟏
𝒔𝒊𝒏𝒉 𝒙 = 𝒄𝒔𝒄𝒉 𝒙 = 𝒔𝒊𝒏𝒉 𝒙
𝟐
𝒆𝒙 +𝒆−𝒙 𝟏
𝒄𝒐𝒔𝒉 𝒙 = 𝒔𝒆𝒄𝒉 𝒙 = 𝒄𝒐𝒔𝒉 𝒙
𝟐
𝒔𝒊𝒏𝒉 𝒙 𝒄𝒐𝒔𝒉 𝒙
𝒕𝒂𝒏𝒉 𝒙 = 𝒄𝒐𝒔𝒉 𝒙 𝒄𝒐𝒕𝒉 𝒙 = 𝒔𝒊𝒏𝒉 𝒙

Hyperbolic Identities
sinh( x)   sinh x cosh( x)  cosh x

cosh 2 x  sinh 2 x  1 1  tanh2 x  sec h2 x


sinh( x  y)  sinh x cosh y  cosh x sinh y

cosh( x  y)  cosh x cosh y  sinh x sinh y

Example : Prove (a) cosh 2 x  sinh 2 x  1(b) 1  tanh2 x  sec h2 x


Solution:
𝑒 𝑥 +𝑒 −𝑥 2 𝑒 𝑥 −𝑒 −𝑥 2 𝑒 2𝑥 +2+𝑒 −2𝑥 𝑒 2𝑥 +2+𝑒 −2𝑥 4
(a) 𝑐𝑜𝑠ℎ2 𝑥 − 𝑠𝑖𝑛ℎ2 𝑥 = ( ) −( ) = − =4=1
2 2 4 4

(b) Exercise

137
APPLIED MATHEMATICS I MODULE

4.8. Derivatives of Hyperbolic Functions


𝑑 𝑑 𝑒 𝑥 − 𝑒 −𝑥 𝑒 𝑥 + 𝑒 −𝑥
(𝑠𝑖𝑛ℎ 𝑥) = = = cosh 𝑥
𝑑𝑥 𝑑𝑥 2 2
We list the differentiation formula for the hyperbolic functions in the following Table.
𝒅 𝒅
(𝒔𝒊𝒏𝒉 𝒙) = 𝒄𝒐𝒔𝒉 𝒙 (𝒄𝒔𝒄𝒉 𝒙) = −𝒄𝒔𝒄𝒉 𝒙 𝒄𝒐𝒕𝒉 𝒙
𝒅𝒙 𝒅𝒙
𝒅 𝒅
(𝒄𝒐𝒔𝒉 𝒙) = 𝒔𝒊𝒏𝒉 𝒙 (𝒔𝒆𝒄𝒉 𝒙) = −𝒔𝒆𝒄𝒉 𝒙 𝒕𝒂𝒏𝒉 𝒙
𝒅𝒙 𝒅𝒙
𝒅 𝒅
(𝒕𝒂𝒏𝒉 𝒙) = 𝒔𝒆𝒄𝒉𝟐 𝒙 (𝒄𝒐𝒕𝒉 𝒙) = −𝒄𝒔𝒄𝒉𝟐 𝒙
𝒅𝒙 𝒅𝒙
𝑑 𝑑 sinh √𝑥
Example 2: 𝑑𝑥
(cosh √𝑥) = sinh √𝑥. 𝑑𝑥 √𝑥 = 2 √𝑥

Inverse Hyperbolic Functions


𝑦 = 𝑠𝑖𝑛ℎ−1 𝑥 ⟺ sin ℎ𝑦 = 𝑥
𝑦 = 𝑐𝑜𝑠ℎ−1 𝑥 ⇔ cosh 𝑦 = 𝑥 𝑎𝑛𝑑 𝑦 ≥ 0
𝑦 = 𝑡𝑎𝑛ℎ−1 𝑥 ⟺ tanh 𝑦 = 𝑥

Inverse hyperbolic functions can be expressed in terms of logarithms.


In particular, we have:
cosh 1 x  ln( x  x 2  1) xR

sinh 1 x  ln( x  x 2  1) x 1
1 1 x
tanh 1 x  ln( ) 1  x  1
2 1 x

Example 2: Show that 𝑠𝑖𝑛ℎ−1 𝑥 = ln(𝑥 + √𝑥 2 + 1).


Solution Let 𝑦 = 𝑠𝑖𝑛ℎ−1 𝑥.Then
𝑒 𝑦 −𝑒 −𝑦
𝑥 = sinh 𝑦 = 2

So 𝑒 𝑦 − 2𝑥 − 𝑒 −𝑦 = 0
Or, multiplying by 𝑒 𝑦 ,
𝑒 2𝑦 − 2𝑥𝑒 𝑦 − 1 = 0
(𝑒 𝑦 )2 − 2𝑥(𝑒 𝑦 ) − 1 = 0

138
APPLIED MATHEMATICS I MODULE

2𝑥±√4𝑥 2 +4
𝑒𝑦 = = 𝑥 ± √𝑥 2 + 1
2

Note that 𝑒 𝑦 > 0 , but 𝑥 − √𝑥 2 + 1 < 0. Thus, the minus sign is inadmissible and we have
𝑒 𝑦 = 𝑥 + √𝑥 2 + 1
Therefore 𝑦 = ln(𝑒 𝑦 ) = ln(𝑥 + √𝑥 2 + 1)

Derivatives of Inverse Hyperbolic Functions


Theorem 3.7

1.
d
sinh x 
1 1
, for every x 4.
d
csc h x 1 1
,
dx
1 x dx
1 x
2 2
x

2.
d
cosh x 
1 1
, for x  1 5.
d
sec h x  
1 1
, for 0  x  1
dx 1 dx
1 x
2 2
x x

3.
d
tanh x 
1 1
, for x  1 6.
d
coth x  1 1
1 x 1 x
2 2
dx dx

1 dx
Proof: To proof (1), let y  sinh x . Then sinh y  x and  cosh y .
dy

Since cosh y  0 and y  sinh y  1 , we have cosh y  1sinh


2 2 2
cosh y.

Then applying the method for inverse functions, we have

dx

d
sinh y   dx1
1

1

1

1 .
dy dx cosh y
1sinh 1 x
2 2
dy y

Observe that we could have done the proof (1) by using formula (1) of theorem 3.6 as below

139
APPLIED MATHEMATICS I MODULE

 
d
sinh 
1
x 
d 
ln  x 
dx  x
2 
 1 
 x
1 
1 
x 

dx
x  x  1 
2 2
1

 x
x  1  
2
1  1
  
.
x x  1  x  1  1 x
2 2 2

The proofs of the formulas 2-6 in the above theorem are left as an exercise.

Example 3:
d
dx
sinh 1
tan x .

Solution: d 
dx sinh
1
tan x   1 d
tan x   1 2
sec x
x  1 dx
2 2
tan sec x

1
 x  sec x .
2

sec x sec

140
APPLIED MATHEMATICS I MODULE

Self-Test Exercises

1. Find the derivative of the following functions by using the definition of derivative

a) c)

b) d)

2. Find the derivative of the following functions by using the definition of derivative at the given
point

a) b)

3. Find the derivative of the following functions

a) c)

b) d)

4. Find an equation of the tangent line to the graph of at the given point

a) b)

5. Find by implicit differentiation

a) c)

b) 𝑦𝑒 𝑥 + 𝑥𝑒 𝑦 = 𝑥𝑦 d)

6. Find an equation of the tangent line to the graph of the following functions at the given point

a) b)

7. Find a formula for the derivative of the following functions

a) b)

141
APPLIED MATHEMATICS I MODULE

CHAPTER FIVE
Applicaions of Derivatives
Introduction:
In the previous chapter we focused almost exclusively on the computation of derivatives. In this
chapter will focus on applications of derivatives. It is important to always remember that we
didn’t spend a whole chapter talking about computing derivatives just to be talking about them.
There are many very important applications to derivatives. The two main applications that we’ll
be looking at in this chapter are using derivatives to determine information about graphs of
functions and optimization problems. These will not be the only applications however. We will
be revisiting limits and taking a look at an application of derivatives that will allow us to
compute limits that we haven’t been able to compute previously. We will also see how
derivatives can be used to estimate solutions to equations.

Objectives:
At the end of this chapter the readers should be able to:
 Define extrema of a function on an interval.
 Understand and use Rolle’s theorem.
 Understand and use the Mean Value Theorem.
 Determine intervals on which a function is increasing or decreasing.
 State and apply the first and second derivative test to find relative extrema of a function.
 Evaluate the the relative extrema of a function.
 Determine intervals on which a function is concave upward or concave down ward.
 Find any points of inflection of the gragh of a function.
 Analyze and sketch the graph of a function.

142
APPLIED MATHEMATICS I MODULE

5.1 Extrema of a Function


Definition 1: A function f has an absolue maximum at c if f (c)  f ( x) for all x in D , where
D is in the domain of f . The number f (c) is called the maximum value of f on D . Similarly,
f has an absolue minimum at c if f (c)  f ( x) for all x in D and the number f (c) is called
the minimum value of f on D . The maximum and minimum values of f are called the extreme
vales of f .

Definition 2: A function f has a local maximum (or relative maximum) value at c if there is an
open interval I containing c such that f (c)  f ( x) for all x in I . Similarly, f has a local
minimum (or relative minimum) value at c if there is an open interval I containing c such that
f (c)  f ( x) for all x in I .

Example 1: Consider the function f ( x)  x . Then f ( x)  f (0) because  0 for all x .


2 2
x
Therefore, f (0)  0 is the absolute (and local) minimum value of f . This corresponds to the fact

that the origin is the lowest point on the parabola y  x . However, there is no highest point on
2

the parabola and so this function has no maximum value.

Example 2: Consider the function f ( x)  x . We see that this funcion has neither an absolute
3

maximum value nor an absolute minimum value. In fact, it has no local extereme values either.
Theorem 1:If f has a relative (local) extremum (that is, maximum or minimum) value at c , and

f (c) 0.
' '
that f (c) exists, then

Definition 3: A number c in the domain of a function f is a critical number of f if eiher

f (c) 0
' '
or f (c) does not exist.

3 8
Example 3: Find the critical numbers of the function f ( x)  4x 5  x 5 .

Solution: The derivative of f is given by

12  2  8 x 3 5 12  8 x .
f ( x) 
'
x 5 5  2
5 5x 5

143
APPLIED MATHEMATICS I MODULE

3
f (c) 0
' if 12  8x  0 , that is x  ' does not exist when x  0 . Thus
Therefore,
2
and f ( x)
3
the critical numbers are x  0 and x  .
2
To find the absolue extreme value of a function on a closed interval a similar theorem to theorem
1 is given below.

Theorem 2: (Maximum-Minimum Theorem)


Let f be continuous on a closed interval a, b . Then f has a maximum and a minimum value
on a, b .
Note that according to Maximum-Minimum Theorem an exreme value can be taken on more
than once.
The following theorem will simplify our effort of searching for an extreme value on a closed
interval.
Theorem 3: Let f be defined on a, b . If an absolute extreme value of f on a, b occurs at a

number c in a, b  at which f has a derivative, then f (c) 0 .


'

In using theorem 2 to find the extreme value we follow the three-step procedure below:
i. Find the values of f at the critical numbers of f in a, b  .
ii. Find the values of f (a) and f (b) .
iii. The largest of the values from steps 1 and 2 is the absolute maximum value; and the
smallest of these values is the absolute minimum value.
Example 4: Find the absolute maximum and minimum values of the function
𝑓(𝑥) = 𝑥 3 − 3𝑥 2 + 1 −1⁄ ≤ 𝑥 ≤ 4
2
1
Solution: Since 𝑓 is continuous on [− 2 , 4] , we can use the Closed Interval Method:
𝑓(𝑥) = 𝑥 3 − 3𝑥 2 + 1
𝑓 ′ (𝑥) = 3𝑥 2 − 6𝑥 = 3𝑥(𝑥 − 2)
Since 𝑓 ′ (𝑥) exists for all 𝑥, the only critical numbers of 𝑓occur when 𝑓 ′ (𝑥) = 0 , that is,𝑥 = 0 or 𝑥 = 2
1
.Notice that each of these critical numbers lies in the interval (− 2 , 4).

The values of𝑓 at these critical numbers are

144
APPLIED MATHEMATICS I MODULE

𝑓(0) = 1 𝑓(2) = −3
The values of𝑓 at the endpoints of the interval are
1 1
𝑓 (− 2) = 8 𝑓(4) = 17

Comparing these four numbers, we see that the absolute maximum value is𝑓(4) = 17 and the absolute
minimum value is(2) = −3 .
Note that in this example the absolute maximum occurs at an endpoint, whereas the absolute minimum
occurs at a critical number.

Activity 5.1
1. Find the critical numbers each of the following functions.

a. c.

b. d.

2. Find the extreme values (if any) of the given function on the given inerval. Determine at
which numbers in the interval these values occur.

a) c)

b)

3. Show that is a critical number of the function , but doesnot have a local

exremum value at .

4. Prove that the function has neither a local maximum nor a local

minimum.

5.2 The Rolle’s Theorem and the Mean Value Theorem (MVT)
Theorem : (Rolle’s Theorem)
Let f be continuous on the closed interval a, b and differentiable on the open interval a, b  . If

f (a)  f (b) then there is atleast one number c in a, b  such that
'
f (c )  0

145
APPLIED MATHEMATICS I MODULE

Figure 1 shows the graphs of four such functions. In each case: it appears that there is at least one
point(𝑐, 𝑓(𝑐)) on the graph where the tangent is horizontaland therefore𝑓 ′ (𝑐) = 0. Thus, Rolle’s Theorem
is plausible.

Example 1: Let f ( x)  x  2x . Find all value(s) of c in the interval  2,2 such that
'
(c )  0
4 2
f
.

Solution: Note that the function f ( x)  x  2x satisfies the conditions of Rolle’s Theorem.
4 2

That is, f is continuous on the interval  2,2 and differentiable on the open interval  2,2 .
Moreover f (2)  8  f (2) .

Thus there exists atleast one number c in  2,2 such that


'
f (c )  0 .

'
Now, f ( x)  x  2x  ( x)  4 x  4 x
4 2 3
f
 4x x 1
2

 4 xx  1x  1
To find c , we have
'
f (c)  0  4c(c  1)(c  1)  0

Hence, c  0,  1, 1 also 0,  1, 1  2,2


Thus all values of c that satisfies he Rolle’s Theorem are c  0,  1, 1 .
Example 2: Prove that the equation 𝑥 3 + 𝑥 − 1 = 0has exactly one real root.
Solution: First we use the Intermediate Value Theorem to show that a root exists.

146
APPLIED MATHEMATICS I MODULE

Let 𝑓(𝑥) = 𝑥 3 + 𝑥 − 1 . Then 𝑓(0) = −1 < 0 and 𝑓(1) = 1 > 0. Since 𝑓 is a polynomial, it is
continuous, so the Intermediate Value Theorem states that there is a number 𝑐 between 0 and 1
such that𝑓(𝑐) = 0. Thus, the given equation has a root.
To show that the equation has no other real root, we use Rolle’s Theorem and argue by
contradiction. Suppose that it had two roots 𝑎 and 𝑏. Then 𝑓(𝑎) = 0 = 𝑓(𝑏) and, since 𝑓 is a
polynomial, it is differentiable on (𝑎, 𝑏) and continuous on [𝑎, 𝑏] . Thus, by Rolle’s Theorem,
there is a number 𝑐 between 𝑎 and 𝑏 such that 𝑓 ′ (𝑐) = 0 . But
𝑓 ′ (𝑥) = 3𝑥 2 + 1 ≥ 1 𝑓𝑜𝑟 𝑎𝑙𝑙 𝑥
(Since 𝑥 2 ≥ 0 ) so 𝑓 ′ (𝑥) can never be 0. This gives a contradiction. Therefore, the equation can’t
have two real roots.

Theorem 4: Let f be continuous on a, b and differentiable on a, b  . Then there is a number c
in a, b  such that

f (b)  f (a)
f (c) 
'
ba
Example 3: Let f ( x)  x  8x  5 . Find a number c in 0,3 that satisfies the Mean Value
3

Theorem.

Solution: Since f is coninuous on 0,3 and '


f (c) should satisfy the condition
f (3)  f (0) 85
f (c) 
'
 1
3 0 3

We seek a number c in 0,3 such that f (c) 1 . But


'

f ( x) 3x
' 8

So that c must saisfy


8 1 c  3
2 2
3c
Hence, c   3

Since  3  0,3 , the value of c that satisfies the Mean Value Theorem in the interval 0,3 is

c 3

147
APPLIED MATHEMATICS I MODULE

5.3 Applications of The Mean Value Theorem

Theorem 5:
'
a. Let f be continuous on an interval I . If f ( x)  0 for each interior point x of I ,

then f is constant on I .
' '
b. Let f and g be continuous on an interval I . If f ( x)  g ( x) for each interior point

x of I , then f  g is constant on I . In other words, there is a constant c such that


f ( x)  g ( x)  c for all x in I .

'
Example 1: Let f be such that f ( x)  2 and f (0)  2 . Determine the function f .

Solution: Since f and  2 x are both antiderivatives of g ( x)  2 x , by Theorem 5(b) there is a


constant c such that f ( x)  g ( x)  c
So that f ( x)  2 x  c , for the appropriate constant c .
To determine c , we have the assumption f (0)  2 , which yields
f (0)  0  2(0)  c  2  c  2 .
Thus, f ( x)  2 x  2 .
Example 2: To illustrate the Mean Value Theorem with a specific function, let’s consider 𝑓(𝑥) = 𝑥 3 −
𝑥, 𝑎 = 0, 𝑏 = 2. Since f is a polynomial, it is continuous and differentiable for all x , so it is certainly
continuous on [0,2] and differentiable on (0,2). Therefore, by the Mean Value Theorem, there is a number 𝑐
in (0,2) such that
𝑓(2) − 𝑓(0) = 𝑓 ′ (𝑐)(2 − 0)
Now (2) = 6, 𝑓(0) = 0 , and 𝑓 ′ (𝑥) = 3𝑥 2 − 1 , so this equation becomes
6 = (3𝑐 2 − 1)2 = 6𝑐 2 − 2
4
Which gives 𝑐 2 = 3 , that is, 𝑐 = ± 2⁄√3 . But 𝑐 must lie in(0,2) , so 𝑐 = 2⁄√3.

148
APPLIED MATHEMATICS I MODULE

Activity 5.2
1. Verify that the function bellow satisfies the hypothesis of the Mean-Value Theorem on the
given interval. Then find all numbers that satisfy the conclusion of the Mean Value
Theorem.

a. c.

b. d.

2. Let . Show that there is no value of such that . Why

does this not contradict the Mean Value Theorem?


3. Determine all functions satisfying the given conditions.

a.

b.

c. ; for any positive integer .

4. Show that the equation has exactly one real root.

5. Show that the equation has at most two real roots.

5.4 First and Second Derivative Tests

We hope you remember that a function that is increasing or decreasing on an interval I is called
monotonic on I and we used the test in the theorem bellow to identify whether a function is
monotonic or not on a given interval.
Theorem 5: Suppose f is continuous on a, b and differentiable on a, b  .

a) If f ( x)  0
' for all x in a, b  , then f is increasing on a, b .

b) If f ( x)  0
' for all x in a, b  , then f is decreasing on a, b .

149
APPLIED MATHEMATICS I MODULE

Example 1: Find where the function 𝑓(𝑥) = 3𝑥 4 − 4𝑥 3 − 12𝑥 2 + 5 is increasing and where it is
decreasing.

Solution: 𝑓 ′ (𝑥) = 12𝑥 3 − 12𝑥 2 − 24𝑥 = 12𝑥(𝑥 − 2)(𝑥 + 1)


To use the I/D Test we have to know where 𝑓 ′ (𝑥) > 0 and where𝑓 ′ (𝑥) < 0. This depends on the signs of
the three factors of 𝑓 ′ (𝑥) , namely, 12𝑥 ,𝑥 − 2 , and 𝑥 + 1. We divide the real line into intervals whose
endpoints are the critical numbers −1, 0 , and 2 and arrange our work in a chart. A plus sign indicates that
the given expression is positive, and a minus sign indicates that it is negative. The last column of the chart
gives the conclusion based on the I/D Test. For instance,𝑓 ′ (𝑥) < 0 for 0 < 𝑥 < 2 , so f is decreasing on
(0, 2). (It would also be true to say that f is decreasing on the closed interval [0,2] .)
Interval 12𝑥 𝑥−2 𝑥+1 𝑓 ′ (𝑥) 𝑓

𝑥 < −1 − − − − 𝑑𝑒𝑐𝑟𝑒𝑎𝑠𝑖𝑛𝑔 𝑜𝑛 (−∞, −1)

−1 < 𝑥 < 0 − − + + 𝑖𝑛𝑐𝑟𝑒𝑎𝑠𝑖𝑛𝑔 𝑜𝑛 (−1,0)

0<𝑥<2 + − + − 𝑑𝑒𝑐𝑟𝑒𝑎𝑠𝑖𝑛𝑔 𝑜𝑛 (0,2)

𝑥>2 + + + + 𝑖𝑛𝑐𝑟𝑒𝑎𝑠𝑖𝑛𝑔 𝑜𝑛 (2, ∞)

Theorem 6: (The First Derivative Test)


Suppose that c is a critical number of a continuous function f .

' changes from positive to negative at c , then f has a local maximum at c .


a) If f ( x)
' changes from negative to positive at c , then f has a local minimum at c .
b) If f ( x)
' does not change sign at c (that is, ' is positive on both sides of c or
c) If f ( x) f ( x)
negative on both sides of c ), then f has no local extremum at c .

8  x  and find its interval of monotonocity.


1
Example 2: Find the local extrema of f ( x)  x 3

Solution: By the product rule we have


2

f ( x)  3 x 8  x  x
' 1 
3
1
3

150
APPLIED MATHEMATICS I MODULE

8  x  3x 42  x 
 2
 2
3x 3
3x 3

f ( x) 0
' when x  2 more over ' does not exist when x  0 .
The derivative f ( x)
Hence, the critical numbers are 0 and 2 .

'
Below we will give the sign chart for f ( x)
0 2
42  x   
2   
3x 3

'   
f ( x)

Thus, by first derivative test, f has

 Does not have an extreme value at x  0 .

 A local maximum value at x  2 and the local maximum value is f (2)  63 2 .


Also, f is
 Increasing on  ,0  0,2 and
 Decreasing on 2,  
Theorem 7: (The Second Derivative Test)

Suppose '' is coninuous on an open inerval of a function.


f

f ( x) 0 f (c) 0 , then
a) If ' and '' f has a local minimum value at c .

f ( x) 0 f (c)0 , then
b) If ' and '' f has a local maximum value at c .

Example 3: Use he Second Derivaive Test to find the relaive extrema of

f ( x)  3x  8x  4
4 3

Solution:

151
APPLIED MATHEMATICS I MODULE

f ( x) 12x3  24x2 12x2 x  2


'

f ( x) 12x2x  20
To find the critical point, '

Hence, the critical numbers are x  0 and x  2 .

f ( x) 36x2  48x
Also, ''

f (2)  480 f (0) 0


Now, '' and ''

Thus, by the second derivative test f has a relative minimum value at x  2 and he relaive
minimum value is f (2)  12.

f (0) 0 , the second derivaive test fails.


Since ''

In the last case, x  0 could still be a relative maximum, relative minimum or neither; but the
Second Derivative Test fails to produce any useful information.
If you used the First Derivative Test, you would find out that x  0 is not relative extremum.
Definition : Let f be differentiable on an open inerval I . The gragh of f is concave upward
' '
on I if f is increasing on the interval and concave downward on I if f is deccreasing on

the interval.
The following graghical interpretation of concavity is useful.
1. Let f be differentiable at c . If the gragh of f is concave upward at c, f (c)  , then the
gragh of f lies above the tangent line at c, f (c)  on some open interval containing c . [see
figure 4.5a]
2. Let f be differentiable at c . If the gragh of f is concave downward at c, f (c)  , then the
gragh of f lies below the tangent line at c, f (c)  on some open interval containing c . [see
figure 4.5b]

152
APPLIED MATHEMATICS I MODULE

Figure 4.5a figure 4.5b


The gragh of f lies above its The gragh of f lies below its
tangent lines. tangent lines
Theorem 7: (The Test for Concavity)
Suppose f is twice differentiable on an interval I .

f ( x) 0
a) If '' for all x in I , then the graph of f is concave upward on I .

f ( x) 0
b) If '' for all x in I , then the graph of f is concave downward on I .

Definition 4: A point c, f (c)  on a curve is called a point of inflection if the curve changes
from concave upward to concave downward or from concave downward to concave upward at
c, f (c) .
Theorem: If c, f (c)  is a point of inflection of the gragh of f , then either
'' ''
f (c)  0 or f is

undefined at x  c .
Example 1: Determine the interval of concavity and the inflection point of the funcion

f ( x)  x  3 x  1
3

f ( x) 3x2 33x 1x 1


Solution: f ( x)  x  3x  1 , then
3 '

f ( x)  0 when x
Since ' 2
 1 , the criical numbers are x  1. Also

f ( x) 6x
''

f ( x) 0 when f ( x)0
Hence, '' x  0 and '' when x  0 .

Thus, f is concave upward on 0,   and concave downward on  ,0 .


Since concavity changes at he point x  0 , the inflection point of f is 0, f (0)  0,1 .

153
APPLIED MATHEMATICS I MODULE

Example 2: Determine the interval of concavity and the inflection point(s) (if any) of the
funcion f ( x)  x  4x
4 3

Soluion: differentiating f twice we ge


'
( x)  4x 12 x
3 2
f
''
( x)  12x  24 x
2
f
 12 x( x  2)
0 2
12 x   
x2   
''   
f ( x)  12 x( x  2)

Thus, - f is concave upward on  ,0  2,  and


- f is concave downward on 0,2
Since concavity changes at both points x  0 and x  2 , the inflection points are 0, f (0) and
2, f (2) which equals 0,0 and 2,16 .
Example 3: Discuss the curve 𝑦 = 𝑥 4 − 4𝑥 3 with respect to concavity, points of inflection, and local
maxima and minima.
Solution: If 𝑓(𝑥) = 𝑥 4 − 4𝑥 3 , then
𝑓 ′ (𝑥) = 4𝑥 3 − 12𝑥 2 = 4𝑥 2 (𝑥 − 3)
𝑓 ′′ (𝑥) = 12𝑥 2 − 24𝑥 = 12𝑥(𝑥 − 2)
To find the critical numbers we set𝑓 ′ (𝑥) = 0 and obtain𝑥 = 0and𝑥 = 3. To use theSecond Derivative Test
we evaluate𝑓 ′′ at these critical numbers:
𝑓 ′′ (0) = 0 𝑓 ′′ (3) = 36 > 0
Since𝑓 ′ (3) = 0 and 𝑓 ′′ (3) > 0,𝑓(3) = −27 is a local minimum. Since𝑓 ′′ (0) = 0 , theSecond Derivative
Test gives no information about the critical number 0. But since𝑓 ′ (𝑥) < 0for 𝑥 < 0 and also for0 < 𝑥 <
3, the FirstDerivative Test tells us that𝑓 doesnot have a local maximum or minimum at 0. [In fact, the
expression for𝑓 ′ (𝑥)shows thatf decreases to the left of 3 and increases to the right of 3.]
Since 𝑓 ′′ (𝑥) = 0 when 𝑥 = 0or 2 , we divide the real line into intervals with these numbers as endpoints
and complete the following chart.

154
APPLIED MATHEMATICS I MODULE

Interval 𝑓 ′′ (𝑥) = 12𝑥(𝑥 − 2) concavity

(−∞, 0) + + + + + + + Upward

(0,2) −−−−−−−− Downward

(2, ∞) + + + + + + + Upward

The point(0,0) is an inflection point since the curve changes from concave upward toconcave downward
there. Also(2, −16) is an inflection point since the curve changesfrom concave downward to concave
upward there.

Activity 5.3
1. Find the critical numbers (if any) and the intervals on which the function is increasing or
decreasing

a. c.

b. d.
2. Use the First or the Second Derivative Test to find any relative extrema (if any) of the
following functions

a. c.

b. d.

3. Find the intervals of concavity and the inflection points(if any) of the function

a) c)

b) d)

155
APPLIED MATHEMATICS I MODULE

5.5 Graph Sketching


Guidelines for Sketching a Curve
The following checklist is intended as a guide to sketching a curve𝑦 = 𝑓(𝑥) by hand. Notevery
item is relevant to every function. (For instance, a given curve might not have an asymptote or
possess symmetry.) But the guidelines provide all the information you need to make a sketch that
displays the most important aspects of the function.
A. Domain It’s often useful to start by determining the domain𝐷of𝑓, that is, the set of valuesof
𝑥 for which 𝑓(𝑥)is defined.
B. Intercepts
C. Symmetry
(i) If𝑓(−𝑥) = 𝑓(𝑥) for all𝑥in𝐷, the function is an even function and the curve is symmetric
about the -axis.
(ii) If 𝑓(−𝑥) = −𝑓(𝑥)for all 𝑥 in, then𝑓 is an odd function and the curve is symmetric about
the origin.
iii) If𝑓(𝑥 + 𝑝) = 𝑓(𝑥) for all𝑥 in D, where𝑝 is a positive constant, then𝑓 is calleda periodic
function and the smallest such number is called the period. For instance,𝑦 = sin 𝑥has period2𝜋.
D. Asymptotes
(i) Horizontal Asymptotes. If either lim 𝑓(𝑥) = 𝐿 or lim 𝑓(𝑥) = 𝐿, then the line 𝑦 = 𝐿is a
𝑥→∞ 𝑥→−∞

horizontalasymptote of the curve𝑦 = 𝑓(𝑥).If it turns out that lim 𝑓(𝑥) = ∞(−∞) , then we do
𝑥→∞

not have anasymptoteto the right.


(ii) Vertical Asymptotes. The line𝑥 = 𝑎 is a verticalasymptote if at least one of the following
statements is true:
lim 𝑓(𝑥) = ∞ lim 𝑓(𝑥) = ∞
𝑥→𝑎+ 𝑥→𝑎−

lim 𝑓(𝑥) = −∞ lim 𝑓(𝑥) = −∞


𝑥→𝑎+ 𝑥→𝑎−

(For rational functions you can locate the vertical asymptotes by equating the denominator to 0
after canceling any common factors. But for other functions this method does not apply.)
(iii) Oblique Asymptotes. Some curves have asymptotes that are oblique, that is, neither horizontal
nor vertical. If
lim [𝑓(𝑥) − (𝑚𝑥 + 𝑏)] = 0
𝑥→∞

156
APPLIED MATHEMATICS I MODULE

then the line𝑦 = 𝑚𝑥 + 𝑏 is called an oblique/slant asymptote. For rational functions, slant
asymptotes occur when the degree of the numerator is one more than the degree of the
denominator. In such a case the equation of the oblique asymptote can be found by long
division.
E. Intervals of Increase or Decrease Use the I/D Test. Compute𝑓 ′ (𝑥) and find the intervalson
which𝑓 ′ (𝑥) ispositive (𝑓 is increasing) and the intervals on which 𝑓 ′ (𝑥)is negative(𝑓 is
decreasing).
F. Local Maximum and Minimum Values
G. Concavity and Points of Inflection Compute𝑓 ′′ (𝑥) and use the Concavity Test.
H. Sketch the Curve Using the information in items A–G, draw the graph. Sketch the asymptotes
as dashed lines.
2

Example: If g ( x)  x , discuss and sketch the graph of g .


1 x
2

Solution:
1. Analize the 𝒙 − 𝒊𝒏𝒕𝒆𝒓𝒄𝒆𝒑𝒕𝒔 𝒂𝒏𝒅 𝒚 − 𝒊𝒏𝒕𝒆𝒓𝒄𝒆𝒑𝒕𝒙 − 𝒊𝒏𝒕𝒆𝒓𝒄𝒆𝒑𝒕𝒔 𝒔
𝑥 − 𝑖𝑛𝑡𝑒𝑟𝑐𝑒𝑝𝑡 = (0,0) and 𝑥 − 𝑖𝑛𝑡𝑒𝑟𝑐𝑒𝑝𝑡 = (0,0)
2. Analize the first derivative.
'
g ( x) 
2x ; This has a root at x  0 . Possible local maximum or minimum here.
1 x  2
2

Notice that neither g (x) nor its derivative are defined at x  1 and x  1 .
The derivative is negative for x  0 , except at x  1 , where it is not defined. It is positive for
x  0 , except at x  1, where it is not defined.

That is g is decreasing on  ,1   1,0 and increasing on 0,1  1,  .


3. Analize the second derivative.
2  6x ; There are no values of x where the second derivative equals zero, so the
2
''
( x) 
1 x 2
g 3

graph of g has no inflection points.


''
g (0)  2 ; At x  0 , a critical number, the second derivative is positive, so the graph is

concave up at this point, and has a local minimum.

157
APPLIED MATHEMATICS I MODULE

4. Find horizontal asymptotes.


2

lim x
2

 1
and lim x  1
1 x 1 x
x  2 x  2

Hence, h( x)  1 is a horizontal asymptote.


5. Find vertical asymptotes.
Since g is undefined at x  1 and x  1 , examine the limits of g as x approaches these values.
2 2 2 2

lim x 2    lim  x 2 and lim x 2    lim  x 2


x 1
1 x x 1
1 x x 1
1 x x1
1 x
Hence, g has a vertical asymptotes at x  1 and x  1 .

6. Put it all together.


r : 5,  4.99.. 5 Range of graghing

Notice that all the aspects of the gragh you found in your analysis are present: a local minimum at
x  0 , vertical asymptotes at x  1 and x  1 , a horizontal asymptote at y  1 , downward
sloping when x  0 , upward sloping when x  0 .
Other Applications of Derivatives
Rate of Change
The first interpretation/application of a derivative is rate of change. If f(x) represents a quantity at
any x then the derivative f (a) represents the instantaneous rate of change of f (x) at x = a.
Example: Suppose that the amount of water in a holding tank at t minutes is given by V (t) =2t2-
16t +35. Determine each of the following.
(a) Is the volume of water in the tank increasing or decreasing at t =1 minute?

158
APPLIED MATHEMATICS I MODULE

(b) Is the volume of water in the tank increasing or decreasing at t =5 minutes?


(c) Is the volume of water in the tank changing faster at t =1 or t =5 minutes?
(d) Is the volume of water in the tank ever not changing? If so, when?

Solution
We are going to need the rate of change of the volume to answer these questions. This means that
we will need the derivative of this function since that will give us a formula for the rate of change
at any time t.
The derivative is.
𝑑𝑉
V’(t) =4t –16 OR = 4𝑡 − 16
𝑑𝑡

if the rate of change is positive then the quantity will be increasing and if the rate of change is
negative then the quantity will be decreasing.
(a) Is the volume of water in the tank increasing or decreasing at t =1minute?
In this case all that we need is the rate of change of the volume at t =1or,
𝑑𝑉
𝑉 ′ = −12 or | = −12
𝑑𝑡 𝑡=1

So, at t
(b) Is the volume of water in the tank increasing or decreasing at t =5 minutes?(exercise)
(c) Is the volume of water in the tank changing faster at t =1 or t =5 minutes?
To answer this question all that we look at is the size of the rate of change and we don’t worry
about the sign of the rate of change. All that we need to know here is that the larger the number
the faster the rate of change.
(d) Is the volume of water in the tank ever not changing? If so, when?
The volume will not be changing if it has a rate of change of zero. In order to have a rate of change
of zero this means that the derivative must be zero. So, to answer this question we will then need
to solve
𝑑𝑉
V ‘(t) =0 OR =0
𝑑𝑡

This is easy enough to do.


4t -16 =0 or t =4
So at t =4 the volume isn’t changing. Note that all this is saying is that for a brief instant the volume
isn’t changing. It doesn’t say that at this point the volume will quit changing permanently.

159
APPLIED MATHEMATICS I MODULE

Velocity
Definition: If 𝑓(𝑡) is the position of a particle moving on a coordinate line, then the instantaneous
velocity of the particle at time 𝑡 is defined by
𝑑𝑣
𝑣(𝑡) = 𝑣 ′ (𝑡) = 𝑑𝑡

Example: Suppose that the position of an object after t hours is given by,
𝑡
𝑔(𝑡) = 𝑡+1

Answer both of the following about this object.


(a) Is the object moving to the right or the left at t =10 hours?
(b) Does the object ever stop moving?
Solution:
The derivative is,
1
𝑔′ (𝑡) = (𝑡+1)2

(a) Is the object moving to the right or the left at t =10 hours?
To determine if the object is moving to the right (velocity is positive) or left (velocity is negative)
we need the derivative at t =10.
1
𝑔′ (10) = 121

So the velocity at t =10 is positive and so the object is moving to the right at t =10.
(b) Does the object ever stop moving?
The object will stop moving if the velocity is ever zero. However, note that the only way a rational
expression will ever be zero is if the numerator is zero. Since the numerator of the derivative (and
hence the speed) is a constant it can’t be zero.
Therefore, the velocity will never stop moving.
In fact, we can say a little more here. The object will always be moving to the right since the
velocity is always positive.
Acceleration
Definition: If f(t) is the position function of a particle moving on a coordinate line, then the
instantaneous acceleration of the particle at time t is defined by

160
APPLIED MATHEMATICS I MODULE

dv
a(t) = v ′ (t) = dt

Or alternatively, since v(t) = f ′ (t),


d2 f
a(t) = f ′′ (t) = dt2

Example: Let 𝑓(𝑡) = 𝑡 3 − 6𝑡 2 be the position function of a particle moving along a 𝑓 −


axis,where𝑓 is in meters and 𝑡 is in seconds.Find the instantaneous acceleration 𝑎(𝑡).
𝑑𝑣
Solution: 𝑎(𝑡) = = 6𝑡 − 12.
𝑑𝑡

upward there.

Activity 5.4
Discuss and sketch the gragh of if

a. c.

b. d.

Note: Your sketches should look like one of the graphs below:

161
APPLIED MATHEMATICS I MODULE

5.6 Indeterminate Forms and L’Hôpital’s Rule

Suppose we are trying to analyze the behavior of the function


ln 𝑥
𝐹(𝑥) = 𝑥−1

Although 𝐹 is not defined when 𝑥 = 1 , we need to know how 𝐹 behaves near 1. In particular,
we would like to know the value of the limit
𝑙𝑛𝑥
lim 𝑥−1
𝑥→1

In computing this limit we can’t apply Law of the limit of a quotient, because the limit of the
denominator is 0. Infract, although the limit in the above function exists, its value is not obvious
0
because both numerator and denominator approach 0 and is not defined.
0
𝑓(𝑥)
In general, if we have a limit of the form lim 𝑔(𝑥)
𝑥→𝑎

Where both 𝑓(𝑥) → 0 and 𝑔(𝑥) → 0 as 𝑥 → 0 , then this limit may or may not exist and is called
𝟎
an indeterminate form of type 𝟎.

Theorem 1: (Cauchy’s Formula)

If f and g are continuous on a, b and differentiable on a, b  and if


'
g ( x)  0 for every x in

a, b , then there is a number c in a, b  such that

162
APPLIED MATHEMATICS I MODULE

'
f (b)  f (a)

f (c )
g (b)  g (a) '
g (c )
Proof: We first note that g (b)  g (a)  0 , because otherwise g (a)  g (b) and by Rolle’s

Theorem, there is a number c in a, b  such that


' '
g (c)  0 , contrary to our assumption about g
.
Let us introduce a new function h as foolows:
h( x)   f (b)  f (a)g ( x)  g (b)  g (a) f ( x)
For every x in a, b . It follows that h is continuous on a, b and differentiable on a, b  and

that h(a)  h(b) . By Rolle’s Theorem there is a number c in a, b  such that h (c)  0 ; that is,
'

 f (b)  f (a) g (c)  g (b)  g (a) f


' '
(c)  0 .
'

Thus,
f (b)  f (a)

f (c ) .
g (b)  g (a) '
g (c )
This is equivalent to Cauchy’s formula.

Indeterminate Forms
If f ( x) and g ( x) are two functions, then we know that

f ( x) lim f ( x)
lim  x a
x a g ( x) lim g ( x)
x a

f ( x)
If lim f ( x)  0 and lim g ( x)  0 , then the expression is said to have the indeterminate form
xa xa g ( x)
0
, at x  a .
0
f ( x)
If lim f ( x)   and lim g ( x)   , then the expression is said to have the indeterminate
xa xa g ( x)

form , at x  a .

0,1 , 

The other indeterminate forms are   , 0  ,
0 0
.

163
APPLIED MATHEMATICS I MODULE

0
The Indeterminate Form .
0
̂pital’s rule)
Theorem 1: (L’H𝒐
Let f (x) and g (x) be two functions such that
' ' '
lim f ( x)  0 and lim g ( x)  0 , f (a) and g (a) exists and g (a)  0 . Then
xa xa

'
f ( x) lim f ( x)
lim  x a ' .
x a g ( x )
lim g ( x)
x a

Proof: Suppose f (x) and g (x) satisfy the conditions of Cauchy’s mean value theorem in the
interval a, x . Then we have
'
f ( x)  f (a)

f (c) , where c  a, x .
g ( x)  g (a ) '
g (c )
Since, f (a)  0 , g (a)  0 and as x  a, c  a , we get
'
f ( x)

f (c )
'
g ( x) g (c )
' '
f ( x) lim f (c ) f ( x)
Hence, lim  c a
'
 lim ' ; (replacing c by x )
x a g ( x )
lim g (c) xa g ( x)
c a

' '
If f (a)  g (a)  0 , then this theorem can be extended as follows:
''
lim f ( x))
c a
 lim
f ( x)
x a ''
lim g ( x) g ( x)
c a

'''

 lim
f ( x) '' ''
(a)  g (a)  0
xa '''
, if f
g ( x)

and so on.
𝑙𝑛𝑥
Example 1: Find lim 𝑥−1 .
𝑥→1

Solution: Since
lim ln 𝑥 = ln 1 = 0 𝑎𝑛𝑑 lim (𝑥 − 1) = 0
𝑥→1 𝑥→1

164
APPLIED MATHEMATICS I MODULE

we can apply l’Hospital’s Rule:


𝑑
𝑙𝑛𝑥 (ln 𝑥) 1⁄
𝑥
lim 𝑥−1 = lim 𝑑𝑥
𝑑 = lim
𝑥→1 𝑥→1 (𝑥−1) 𝑥→1 1
𝑑𝑥

1
= lim 𝑥 = 1
𝑥→1

Example 2:Evaluate lim 1 3


x

.
x 0 x
Solution: Both the numerator and denominator have the limit 0 as x  0 . Hence the quotient has

the indeterminate form 0 . By L’H𝑜̂pital’s rule


0

lim 13  lim 3


x x
ln 3
  ln 3
x0 x x0 1
x  sin x
Example 3: Evaluate lim 3 .
x 0
x
Solution: Both the numerator and denominator have the limit 0 as x  0 . Hence the quotient has

the indeterminate form 0 . By L’H𝑜̂pital’s rule


0
x  sin x 1  cos x  0 
lim  lim   form
3x  0 
x0 3 x0 2
x
sin x  0 
 lim  0  form
x 0 6 x

cos x
 lim
x0 6
1
 .
6

Example 4: Evaluate lim



ln 1 x
2
.
x 0 ln cos 2 x 

x0

Solution: Observe that lim ln 1 x  0  lim ln cos 2 x 
2
 x0

Hence the quotient has the indeterminate form 0 .


0
Thus by L’H𝑜̂pital’s rule we get

165
APPLIED MATHEMATICS I MODULE

 2x

lim
ln 1 x 2

 lim
 1  x
2
 1
 lim  .
 2 x 
x0 ln cos 2 x  x0  
1 x  2 tan 2 x 
x0  2 tan 2 x 2

x 1
 lim , since lim 1
1 x
 2
x0 tan 2 x x 0

 x 
 lim  .(cos 2 x)

x0 sin 2 x

x
 lim . lim (cos 2 x)
x0 sin 2 x x0

1
  1  .
1
2 2

 arctan x
Example 5: Evaluate lim 2 .
x  1
x
  1 
Solution: Observe that lim   arctan x   0  lim , since lim arctan x  .

x 2
 x0 x x 2

Hence the quotient has the indeterminate form 0 .


0
Thus by L’H𝑜̂pital’s rule we get

 1
 arctan x
 lim 1 x
2 2

lim 2  lim x 1.


1
1 x
2
x  1 x  x 
2
x x
x
  2x
x

Example 6: Evaluate lim e e .


x0 2 x  sin 2 x
Solution: Both the numerator and denominator have the limit 0 as x  0 . Hence the quotient has

the indeterminate form 0 . By L’H𝑜̂pital’s rule


0
x x
  2x  2
x x
0
lim e e  lim e e ,  0  form
x0 2 x  sin 2 x x0 2  2 cos 2 x

166
APPLIED MATHEMATICS I MODULE

x

x
0
 lim e e ,  0  form
x 0 4 sin 2 x
x

x
0
 lim e e ,  0  form
x 0 8 cos 2 x
2 1
  .
8 4

sec x  2 tan x .
2

Example 7: Evaluate lim


x
 1  cos 4 x
4

x

Solution: Observe that lim sec x  2 tan x  0  lim 1  cos 4 x  .
2
 x
4 4

Hence the quotient has the indeterminate form 0 .


0
Thus by L’H𝑜̂pital’s rule we get

x  2 tan x x tan x  2 sec x


2 2 2

lim sec  lim 2 sec


x
 1  cos 4 x x
  4 sin 4 x
4 4

xtan x  1
 lim sec
2
0

,  0  form
x 2 sin 4 x
4

sec x.sec x  tan x  12 sec x tan x 


 2 2 2

 lim   

x
4

 8 cos 4 x 


sec 0
4

 lim 4
x
 8 cos 
4

4 1
  .
8 2

167
APPLIED MATHEMATICS I MODULE

̂pital’s rule)
Theorem 2: (L’H𝒐
Let f (x) and g (x) be two functions such that
' ' '
lim f ( x)   or   and lim g ( x)   or   , f (a) and g (a) exists and g (a)  0 . Then
xa xa

'
f ( x) lim f ( x)
lim  x a ' .
x a g ( x )
lim g ( x)
x a

4 tan x
Example 1: Evaluate lim  .
   1  sec x
x  
2

Solution: Observe that the limit has the inditerminate form  . Then by L’H𝑜̂pital’s rule we

have
2

 lim  4 sec  lim 


4 tan x x 4 sec x
lim 
 
x  
1  sec x  
x 
2
sec x tan x x 
2
tan x  
2
By using the trigonometric identity, we have
4
4 sec x cos x  4

tan x sin x sin x
cos x
Consequently
2

 lim  4 sec  lim 


4 tan x x 4 sec x 4
lim   lim   4.

 
x  
1  sec x  
x 
2
sec x tan x x 
2
tan x  
x 
2
sin x  
2
𝑒𝑥
Example 2: Calculate lim .
𝑥→∞ 𝑥 2

Solution: We have lim 𝑒 𝑥 = ∞ and lim 𝑥 2 = ∞, so l’Hospital’s Rule gives


𝑥→∞ 𝑥→∞
𝑑
𝑒𝑥 (𝑒 𝑥 ) 𝑒𝑥
lim = lim 𝑑𝑥
𝑑 = lim
𝑥→∞ 𝑥 2 𝑥→∞ (𝑥 2 ) 𝑥→∞ 2𝑥
𝑑𝑥

Since𝑒 𝑥 → ∞ and2𝑥 → ∞ as𝑥 → ∞ , the limit on the right side is also indeterminate, but a second
application of L’Hospital’s Rule gives
𝑒𝑥 𝑒𝑥 𝑒𝑥
lim = lim = lim =∞
𝑥→∞ 𝑥 2 𝑥→∞ 2𝑥 𝑥→∞ 2

168
APPLIED MATHEMATICS I MODULE

2x

Example 3: Evaluate lim e 2


.
x 
x
Solution: Observe that the limit has the inditerminate form  . Then by L’H𝑜̂pital’s rule we

have
 
2x 2x

lim e 2  lim 2e ,    form


x 
x x 2 x
2x 2x

 lim 2e  lim 4e  lim 2e   .


2x

x  2x x  2 x 

Particularly in a similar fashion we can show that


x

lim e n
  , for every real number n .
x 
x
Other Indeterminate Forms



Various inditerminate forms, such as   , 0  ,
0 0
0,1 and , can usually be converted

into the indeterminate form 0 or  and then evaluaed by one of the versions of L’H𝑜̂pital’s
0 
rule given in the above theorem.
2
Example 1: Evaluate lim x ln x .
x 0

Solution: Since lim x  0 and lim ln x   the given limit is of the form 0   (more
2

x 0 x0

precisely, 0  () ). However, we can transform it into the inditerminate form  by writing

it as
ln x
lim x ln x  lim
2

x 0 x 0 1 2
x
Now, applying the L’H𝑜̂pital’s rule we get
1 2

 lim  x
ln x x
ln x  lim  lim 0.
2
lim x
x0 x 0 1 2 x0  2 3
x 0

2
x x

169
APPLIED MATHEMATICS I MODULE

x
Example 2: Evaluate lim x .
x 0

x e
0 x x ln x
Solution: The limit evidently has the indeterminate form 0 . But then since and

consequently

lim x  lim e
x x ln x

x0 x0

Since the exponential function is continuous, it follows that

 ex0
x ln x lim ( x ln x )
lim e
x0

But since
1
ln x x  lim ( x)  0
lim x ln x  lim  lim
x 0  1

x 0 x 0 1 x 0
2
x
x
By L’H𝑜̂pital’s rule we get

lim x  lim e  e  1.
x x ln x 0

x0 x0

Example 3: Show that


 1
lim 1  e.
 x
x


Solution: Observe that the limit has the indetrminate form 1
x
 1
x
 1
1
Since, 1   e  x  , we have
ln

 x
x
x
 1
 1
x
ln1   1
lim ln1
lim 1   lim e  x   ex  x 
x  x x 

But lim ln 
1
x

 1 ln 1  1   0
1   lim x ln 1    lim
 
x ,
 0  form
 x
x  x  x x  1
x
Thus by L’H𝑜̂pital’s rule we get
1  
 1 

 lim

1 1  2

x  x   lim 1
1
x  1 2 x 
1 1
x
x

170
APPLIED MATHEMATICS I MODULE

Thus, lim 1


 1
 e  e.
1

 x
x 

 1 1
Example 4: Find lim x   .
x 0  
 e 1 x 
Solution: Observe that the limit has the indetrminate form    .
However, if the difference is written as a single fraction, then
 1 1 x e 1 ,
x
0
lim x    lim  0  form
x 0   x0
 e 1 x  xe  x
x

Thus by L’H𝑜̂pital’s rule we get

x  e  1  lim 1 e
x x

lim
xe  x 0 xe  e 1
x  x x
x0 x

e
x
1.
 lim 
xe  2e
x x
x 0 2

 1 1 1
Hence, lim x   .
x0  
 e 1 x  2

Example 5: Compute lim


𝜋
(sec 𝑥 − tan 𝑥) .
𝑥→( )−
2
𝜋
Solution: First notice that sec 𝑥 → ∞ and tan 𝑥 → ∞ as → ( 2 )− , so the limit is indeterminate.Here

we use a common denominator:


1 sin 𝑥
lim
𝜋
(sec 𝑥 − tan 𝑥) = lim −(cos 𝑥 − cos 𝑥)
𝑥→( )− 𝑥→(𝜋 ⁄2)
2

1−sin 𝑥 − cos 𝑥
= lim − = lim − =0
𝑥→(𝜋 ⁄2) cos 𝑥 𝑥→(𝜋 ⁄2) − sin 𝑥

Note that the use of l’Hospital’s Rule is justified because1 − sin 𝑥 → 0 andcos 𝑥 → 0as→ (𝜋⁄2)

171
APPLIED MATHEMATICS I MODULE

Self-Test Exercise
1. Find the absolute extrema of the following functions on the given interval

a) b) c)

2. Find the value of the number guaranteed by the Mean Value Theorem of the following functions
on the given interval

a) c)

b) d)

3. Derermine the values such that the function satisfies the hypothesis of the Mean
Value Theorem on the interval .

4. Find the interval of monotonocity of the following functions

a) b) c)

5. Use first or second derivative test to find all the relative extrema of the following functions
a) d)

b) e)

c) f)

6. Find the interval of concavity and the inflection point (if it exists) of the following functions

a) c)

b) d)

172
APPLIED MATHEMATICS I MODULE

7. Find the absolute extrema of the following functions on the given interval

b) b) c)

8. Find the value of the number guaranteed by the Mean Value Theorem of the following functions
on the given interval

c) c)

d) d)

9. Derermine the values such that the function satisfies the hypothesis of the Mean
Value Theorem on the interval .

10. Find the interval of monotonocity of the following functions

b) b) c)

11. Use first or second derivative test to find all the relative extrema of the following functions
d) d)

e) e)

f) f)

12. Find the interval of concavity and the inflection point (if it exists) of the following functions

c) c)

d) d)

173
APPLIED MATHEMATICS I MODULE

CHAPTER SIX

Integration

Introduction

Integration is not as straight forward as differentiation; there are no rules that absolutely guarantee
obtaining indefinite integrals of a function. In this chapter we develop techniques for using the
basic integration formulas to obtain indefinite integrals of more complicated functions. Therefore
we discuss a strategy for integration in this section. Also we consider integrals having an
unbounded limit of integration and integrals of discontinuous functions on an interval. Finally,
integration has a wide variety of applications. In this section look at some applications of integrals
in finding the area of a region bounded by a curve and the volume a solid region.

Objectives

At the end of this chapter the readers should be able to:

 Know anti derivative

 Find the indefinite and definite integral of different functions using different techniques
of integrations.

 Define the improper integral of a function.

 Evaluate an improper integral that has an infinite limit of integration.

 Evaluate an improper integral that has an infinite discontinuity.

 Evaluate improper integrals of different functions.

 Find the area of a region between two curves using integration.

 Find the volume of a solid with known cross-sections.

 Find the volume of a solid of revolution.

174
APPLIED MATHEMATICS I MODULE

Anti-derivative formula
Before we see techniques of integration let us revise the integrals of important functions in the
following table.

Derivative Indefinite integral


𝐷𝑋 (𝑥) = 1
∫ 1𝑑𝑥 = 𝑥 + 𝑐
𝑥𝑟+1 𝑥 𝑟+1
𝑟 ∫ 𝑥 𝑟 𝑑𝑥 = + 𝑐,(r≠ −1)
𝐷𝑋 ( ) = 𝑥 (𝑟 ≠ −1) 𝑟+1
𝑟+1
𝐷𝑋 (𝑠𝑖𝑛𝑥) = 𝑐𝑜𝑠𝑥
∫ 𝑐𝑜𝑠𝑥𝑑𝑥 = 𝑠𝑖𝑛𝑥 + 𝑐
𝐷𝑋 (𝑐𝑜𝑠𝑥) = −𝑠𝑖𝑛𝑥
∫ 𝑠𝑖𝑛𝑥𝑑𝑥 = −𝑐𝑜𝑠𝑥 + 𝑐
𝐷𝑋 (𝑡𝑎𝑛𝑥) = 𝑠𝑒𝑐2 𝑥
∫ 𝑠𝑒𝑐𝑑𝑥 = 𝑡𝑎𝑛𝑥 + 𝑐
𝐷𝑋 (−𝑐𝑜𝑡𝑥) = 𝑐𝑠𝑐2 𝑥
∫ 𝑐𝑠𝑐𝑑𝑥 = −𝑐𝑜𝑡𝑥 + 𝑐
𝐷𝑋 (𝑠𝑒𝑐𝑥) = 𝑠𝑒𝑐𝑥𝑡𝑎𝑛𝑥
∫ 𝑠𝑒𝑐𝑥𝑡𝑎𝑛𝑥𝑑𝑥 = 𝑠𝑒𝑐𝑥 + 𝑐
𝐷𝑋 (−𝑐𝑠𝑐𝑥) = 𝑐𝑠𝑐𝑥𝑐𝑜𝑡𝑥
∫ 𝑐𝑠𝑐𝑥𝑐𝑜𝑡𝑑𝑥 = −𝑐𝑠𝑐𝑥 + 𝑐
𝐷𝑋 (𝑒𝑥 ) = 𝑒𝑥
∫ 𝑒𝑥 𝑑𝑥 = 𝑒𝑥 + 𝑐
𝑎𝑥 𝑎𝑥
𝐷𝑋 =( )=𝑎𝑥 ∫ 𝑎 𝑑𝑥 =𝑥
+𝑐
𝑙𝑛𝑎
𝑙𝑛𝑎
𝐷𝑋 =(ln|𝑥|)=
1 1
𝑥 ∫ 𝑑𝑥 = ln|𝑥| + 𝑐
𝑥
𝑥 1 1 𝑥
𝐷𝑋= ( 𝑠𝑖𝑛−1 )= ∫ 𝑑𝑥 = 𝑠𝑖𝑛−1
+𝑐
𝑎 √𝑎2 −𝑥2
√𝑎2 − 𝑥2 𝑎
𝑥 1 1 𝑥
𝐷𝑋= (𝑐𝑜𝑠−1 ) = − ∫ 𝑑𝑥 = −𝑐𝑜𝑠−1 + 𝑐
𝑎 √𝑎2 − 𝑥2 √𝑎2 − 𝑥2 𝑎
1 𝑥 1 1 1 𝑥
𝐷𝑋= ( 𝑡𝑎𝑛−1 )= ∫ 2 𝑑𝑥 = 𝑡𝑎𝑛−1
+𝑐
𝑎 𝑎 𝑎2 +𝑥2
𝑎 + 𝑥2 𝑎 𝑎
1 𝑥 1 1 1 𝑥
𝐷𝑋= ( 𝑠𝑒𝑐−1 ) = ∫ 𝑑𝑥 = 𝑠𝑒𝑐−1 + 𝑐
𝑎 𝑎 𝑥√𝑥2 − 𝑎2 𝑥√𝑥2 − 𝑎2 𝑎 𝑎

175
APPLIED MATHEMATICS I MODULE

6.1- Anti derivatives; indefinite integrals


In the past chapter we’ve been given a function and asking what the derivative of this function
was. Starting with this section we are not going to turn things around. We now want to ask what
function we differentiated to get the function

Definition1: A function F is called anti derivative of f on an interval I if F  x   f  x  for all


'

x in I .

Example 1:
What function did we differentiate to get the following function?

f  x   x 4 +3x-9
Solution:
Let‘s actually start by getting the derivative of this function to help us see how we’re going to
have to approach this problem. The derivative of this function is
The point of this was to remind us how differentiate works. When differentiating powers of x we
multiply the term by the original exponent and the drop the exponent by one .Now ,let’s go back
and work the problem .In fact let’s start with the first term .we got x 4 by differentiating a
function and since we drop the exponent by one it look like we must have differentiated x 5 .
However, if we had differentiated x 5 we would have 5x 4 and we don’t have 5 in front of our first
term, so the 5 needs to cancel out after we’ve differentiated .It looks the like we would have to
differentiate 1 x 5 in order to get x 4 .
5

Likewise the second term ,in order to get 3x after differentiating we would have to differentiate
3 2
x .Again the fraction is there to cancel out the 2 we pick up in the differentiation .
2

The third term is just a constant and we know that if we differentiate x we get 1.so, it looks like
we had to differentiate −9x to get the last term.
Putting all of this together give the following function,
1 5 3 2
F  x  x  x  9x
5 2

Our answer is easy to check .simply differentiate F  x  .

F '  x   x4 +3x-9  f  x 

176
APPLIED MATHEMATICS I MODULE

So, it look like we got the correct function. We know that the derivative of the constant is zero
and any of the following will also give f  x  up on differentiating.

1 3
F  x   x 5  x 2  9 x  10
5 2
1 3
F  x   x 5  x 2  9 x  1954
5 2
1 3 3469
F  x   x5  x 2  9x 
5 2 123
etc.
In fact, any function of the form
1 5 3 2
F  x  x  x  9x  C , C is a constant
5 2

Will give f  x  up on differentiating.

Theorem 1: If F  x  is an ant derivative of f  x  on an interval I , then for any constant C the


function F  x   C is also an ant derivative of f  x  on that interval. Moreover, each ant
derivative of f  x  on the interval I can be expressed in the form F  x   C by choosing the
constant C appropriately.
Proof: Exercise
The process of finding ant derivatives is called ant differentiation or integration.
Example 2:

Find an Ant derivative of f  x   sin x that satisfies F  0   3 .

Solution:
Since the derivative of −𝑐𝑜𝑠𝑥 is sin x ,the general ant derivative

F  x    cos x  C

Gives all the ant derivatives of f  x  .the condition f  0   3 determines a specific value for C
.substituting x  0 in to F  x    cos x  C gives

F  0   cos x  C  1  C

Since F  0   3 , solving for C gives C  4 .so

177
APPLIED MATHEMATICS I MODULE

F  x    cos x  4

Is the anti-derivative we satisfying F (0) =3

Activity 6.1:
Find the general ant derivative of each of the following functions

Definition 2:

If F  x  is any ant derivative of f  x  the most general ant derivative of f  x  is called an

indefinite integral and denoted

 f  x  dx F  x   C
In this definition the  is called the integral symbol f  x  is called the integral x is called the
integral variable and the ′𝐶′ is called the constant of integration.

Note that often we will just say integral instead of indefinite integral (or definite integral for that
matter when we get to those). It will be clear from the context of the problem that we are talking
about an indefinite integral (or definite integral).
The process of finding the indefinite integral is called integration or integrating f(x). If we need
to be specific about the integration variable we will say that we are integrating f(x) with respect
to x.

178
APPLIED MATHEMATICS I MODULE

Example3: Evaluate the following indefinite integral.

x  3x  9dx
4

Solution:
Since this is really asking for the most general ant derivative we just need to re use the final
answer from the above example.

The indefinite integral is,


1 5 3 2
x  3x  9dx  x  x  9x  c
4

5 2

A couple of warnings are now in order. One of the more common mistakes that students make
with integrals (both indefinite and definite) is to drop the dx at the end of the integral. This is
required! Think of the integral sign and the dx as a set of parenthesis. You already know and are
probably quite comfortable with the idea that every time you open a parenthesis you must close it.
With integrals, think of the integral sign as an “open parenthesis” and the dx as a “close
parenthesis”.
If you drop the dx it won’t be clear where the integrand ends. Consider the following variations of
the above example.
1 5 3 2
x  3x  9dx 
x  x  9x  c
4

5 2
1 5 3 2
 x  3xdx  9  5 x  2 x  c  9
4

1 5
 x dx  3x  9  5 x  c  3x  9
4

You only integrate what is between the integral sign and the dx.
Knowing which terms to integrate is not the only reason for writing the dx down. In the Substitution
Rule section we will actually be working with the dx in the problem and if we aren’t in the habit
of writing it down it will be easy to forget about it and then we will get the wrong answer at that
stage. The moral of this is to make sure and put in the dx! At this stage it may seem like a silly
thing to do, but it just needs to be there, if for no other reason than knowing where the integral
stops.
On a side note, the dx notation should seem a little familiar to you. We saw things like this a couple
of sections ago. We called the dx a differential in that section and yes that is exactly what it is. The
dx that ends the integral is nothing more than a differential. The next topic that we should discuss

179
APPLIED MATHEMATICS I MODULE

here is the integration variable used in the integral. Actually there isn’t really a lot to discuss here
other than to note that the integration variable doesn’t really matter. For instance,

Changing the integration variable in the integral simply changes the variable in the answer. It is
important to notice however that when we change the integration variable in the integral we also
changed the differential (dx, dt, or dw) to match the new variable. This is more important that we
might realize at this point.
Another use of the differential at the end of integral is to tell us what variable we are integrating
with respect to. To see why this is important take a look at the following two integrals.

 2xdx  2tdx
The first integral is simple enough.

 2xdx  x c
2

The second integral is also fairly simple, but we need to be careful. The dx tells us that we are
integrating x’s. That means that we only integrate x’s that are in the integrand and all other
variables in the integrand are considered to be constants. The second integral is then,

 2tdx  2tx  c
So, it may seem silly to always put in the dx, but it is a vital bit of notation that can cause us to
get the incorrect answer if we neglect to put it in.
Now, there are some important properties of integrals that we should take a look at.

Properties of the indefinite integral

1.  kf  x  dx  k  f  x  dx Where k is any number. So we can facto multiplicative

Constants out of indefinite integrals.

2.   f  x  dx   f  x  dx .This is really the first property with k=-1 and so no proof of this
property will be given.

3,  f  x   g  x  dx   f  x  dx   g  x  dx in other word the integral of sum or difference of


function is the sum or difference of the individual integrals. This rule can be extended to as many
functions as we need.

180
APPLIED MATHEMATICS I MODULE

Proof 1:

 kf  x  dx  k  f  x  dx .where k is any number.


This is a very simple proof. Suppose that F  x  is an anti -derivative of f  x  i.e. F  x   f  x  .
'

Then by the basic properties of derivatives we also have that,

 kF  x   kF '  x   kf  x 
'

And so kF  x  is an ant derivative of kf  x  .i.e.  kF  x    kf  x  .in other words,


'

 kf  x  dx  kF  x   c  k  f  x  dx
Proof 3:

 f  x   g  x dx   f  x dx   g  x dx


Not listed in the properties above were integrals of products and quotients. The reason for this is
simple. Just like with derivatives each of the following will NOT work.

 f  x  g  x dx   f  x  g  x dx
f  x  f  x  dx
 g  x dx 
 g  x  dx

With derivatives we had a product rule and a quotient rule to deal with these cases. However, with
integrals there are no such rules. When faced with a product and quotient in an integral we will
have a variety of ways of dealing with it depending on just what the integrand is.
We can now answer this question easily with an indefinite integral.
f  x    f '  x  dx

Example 4: If f ' x   x 4  3x  9 then find f x 

Solution:
By this point in this section this is a simple question to answer
1 5 3 2
f  x    f '  x  dx   x 4  3x  9dx  x  x  9x  c
5 2

181
APPLIED MATHEMATICS I MODULE

Example 5: Find the general indefinite integral

 10 x  2sec2 x dx


4

Solution:

 10x  2sec2 x dx   10 x 4 dx   2sec2 xdx


4

 10 x 4 dx  2 sec 2 xdx


x5
 10  2 tan x  c
5
 2 x5  2 tan x  c

cos x
Example 6: Evaluate  dx
sin 2 x
Solution:

cos x  1  cos x 
 sin
2
x
dx      dx
 sin x  sin x 

  csc x cot xdx


  csc x  c

t 2  2t 4
Example 7: Evaluate  dt
t4
Solution

t 2  2t 4 1 
 t 4 dx    t 2  2 dt   t  2dt
2

t 1 1
  2t  c    2t  c
1 t

182
APPLIED MATHEMATICS I MODULE

Activity 6.2:
1. Evaluate each of the following indefinite integrals

2. Evaluate each of the following integrals

6.2. Partitions
Let us consider any region 𝑅 bounded by the graph of a nonnegative f that is continuous
on an interval [a, b], by the x axis, and by the lines 𝑥 = 𝑎 and 𝑥 = 𝑏. We need to find
the area of the region R. To do this, let us list three basic properties that the area of a
region possesses:

1. The Rectangle Property: The area of a rectangle is the product of the lengths of its base
and height.
2. The Addition Property: The area of a region composed of several smaller regions that
overlap in at most a line segment is the sum of the areas of the smaller regions.
3. The Comparison Property: The area of a region that contains a second region is at least
as large as the area of the second region.
6.2.1 Partitions, Upper and Lower sum, Riemann Sums
Definition 1: A partition of the interval [a, b] is a finite set ℘ of points x0 ,x1 , … , xn
such that a = x0 < x1 < ⋯ < xn = b. That is,

183
APPLIED MATHEMATICS I MODULE

℘ = {x0 , x1 , … , xn }.
By definition, any partition of [a, b] must contain both a and b. Except for the end
points a and b, the number of points and their placement in [a, b] is arbitrary. For
example, {0, 1, 3⁄2 , 2, 3}, {0, 1,2, 3}, , {0,3} are partitions of the interval [0, 3] (Figure
4.2). However, {0, 1⁄2, 1, 4⁄3, 3⁄2} is not a partition of [0, 2] because it does not
3
contain 2. Similarly, {1, 4⁄3, 3⁄2}is not a partition of [0, 2].

The n subintervals into which a partition ℘ = {𝑥0 , 𝑥1 , … , 𝑥𝑛 } divides [a, b] are[𝑥0 , 𝑥1 ],


[𝑥1 , 𝑥2 ], [𝑥2 , 𝑥3 ], … , [𝑥𝑛−1 , 𝑥𝑛 ] and their lengths are 𝑥1 − 𝑥0 , 𝑥2 − 𝑥1 , 𝑥3 − 𝑥2 , … , 𝑥𝑛 −
𝑥𝑛−1 respectively. We denote the length 𝑥𝑘 − 𝑥𝑘−1 of the kth subinterval [𝑥𝑘−1 , 𝑥𝑘 ]
by△ 𝑥𝑘 . Thus
△ 𝑥𝑘 = 𝑥𝑘 − 𝑥𝑘−1
Example 1: For the partition {0, 1, 3⁄2 , 2, 3} of [0, 3]. Find the length of each
subinterval.

Solution:
𝑥0 = 0, 𝑥1 = 1, 𝑥2 = 3⁄2 , 𝑥3 = 2, 𝑥4 = 3
And
△ 𝑥1 = 𝑥1 − 𝑥0 = 1 − 0 = 1 △ 𝑥2 = 𝑥2 − 𝑥1 = (3⁄2) − 1 = 1⁄2 △
𝑥3 = 𝑥3 − 𝑥2 = 2 − 3⁄2 = 1⁄2 △ 𝑥4 = 𝑥4 − 𝑥3 = 3 − 2 = 1
Note that the length of the interval [a, b] can be written in terms of the lengths △ 𝑥1 , △
𝑥2 ,△ 𝑥3 , … ,△ 𝑥𝑛 of the subintervals. Indeed,
𝑏 − 𝑎 = (𝑥1 − 𝑥0 ) + (𝑥2 − 𝑥1 ) + ⋯ + (𝑥𝑛 − 𝑥𝑛−1 )
=△ 𝑥1 +△ 𝑥2 + ⋯ +△ 𝑥𝑛

184
APPLIED MATHEMATICS I MODULE

Activity 6.2:1

Find at least four partitions for each of the following intervals and find the length of each
sub interval.

a. [1, 4]
b. [0, 3]

6.2.2 Lower and Upper Sums


Having chosen a partition ℘ of the interval [a, b], we inscribe over each subinterval
derived from ℘ the largest rectangle that lies inside the region R. Since we are assuming
that 𝑓 is continuous on [a, b], we know from the Maximum -Minimum Theorem that for
each k between 1 and n there exists a smallest value 𝑚𝑘 of f on the 𝑘𝑡ℎ
subinterval[𝑥𝑘−1 , 𝑥𝑘 ]. If we choose 𝑚𝑘 as the height of the 𝑘𝑡ℎ rectangle 𝑅𝑘 , then 𝑅𝑘
will be the tallest rectangle that can be inscribed in 𝑅 over[𝑥𝑘−1 , 𝑥𝑘 ]. Doing the same
for each subinterval, we create n inscribed rectangles 𝑅1 , 𝑅2 , … , 𝑅𝑛 all lying inside the
region 𝑅.

𝑅𝑛
𝑅3
𝑓
𝑅2 𝑅4

𝑅1

a = 𝑥0 𝑥1 𝑥2 𝑥3 𝑥4 … 𝑥𝑛−1 𝑥𝑛 = 𝑏

Figure 1

For each k between 1 and n, 𝑅𝑘 has base of length △ 𝑥𝑘 and height 𝑚𝑘 . Hence the area
of 𝑅𝑘 is the product𝑚𝑘 △ 𝑥𝑘 . The sum

185
APPLIED MATHEMATICS I MODULE

𝑚1 △ 𝑥1 + 𝑚2 △ 𝑥2 + ⋯ + 𝑚𝑛 △ 𝑥𝑛
of the areas of all the rectangles should be no larger than the area of the region 𝑅 under
the curve 𝑓, above the 𝑥 axis, and between the vertical lines 𝑥 = 𝑎 and𝑥 = 𝑏. This sum
is called the lower sum of 𝑓associated with the partition ℘ and is denoted by𝐿𝑓 (℘).
Even though it is difficult to find the actual area of the region R, this area must be at
least as large as the lower sum 𝐿𝑓 (℘).That is,
𝐿𝑓 (℘) = 𝑚1 △ 𝑥1 + 𝑚2 △ 𝑥2 + ⋯ + 𝑚𝑛 △ 𝑥𝑛 (1)
Example 1: Let 𝑓(𝑥) = 𝑥 2 for0 ≤ 𝑥 ≤ 3. Find 𝐿𝑓 (℘) and 𝐿𝑓 (℘′ ) for the partitions
3 1 3 5
(a) ℘ = {0, 1, 2 , 2, 3} (b) ℘′ = {0, 2 , 1, 2 , 2, 2 , 3}

Solution:

a) The subintervals associated with the partition ℘ are [0, 1], [1, 3⁄2], [3⁄2 , 2], and [2, 3].
Computing the minimum value of 𝑓 on each of these subintervals, we find that

𝑚1 = 𝑓(0) = 0, 𝑚2 = 𝑓(1) = 1, 𝑚3 = 𝑓(3⁄2) = 9⁄4, 𝑚4 = 𝑓(2) = 4.


Moreover,
△ 𝑥1 = 1 − 0 = 1, △ 𝑥2 = (3⁄2) − 1 = 1⁄2

△ 𝑥3 = 2 − (3⁄2) = 1⁄2, △ 𝑥4 = 3 − 2 = 1
Therefore by (1),

𝐿𝑓 (℘) = 0 ∙ 1 + 1 ∙ 2 + 4 ∙ 2 + 4 ∙ 1 = 45⁄8
1 9 1

b) It can be seen from Figure 4.4 (b) that the subintervals associated with the partition ℘′ are [0, 1/2],
[1/2, 1], [1, 3/2], [3/2, 2], [2, 5/2], and [5/2, 3]. Computing the minimum value of 𝑓 on each of
these subintervals, we find that

𝑚1 = 𝑓(0) = 0, 𝑚2 = 𝑓(1⁄2) = 1⁄4, 𝑚3 = 𝑓(1) = 1, 𝑚4 = 𝑓(3⁄2) = 9⁄4

𝑚5 = 𝑓(2) = 4, 𝑚6 = 𝑓(5⁄2) = 25⁄4.

Moreover,

186
APPLIED MATHEMATICS I MODULE

△ 𝑥1 = (1⁄2) − 0 = 1⁄2, △ 𝑥2 = 1 − 1⁄2 = 1⁄2, △ 𝑥3 = (3⁄2) − 1 = 1⁄2, △

𝑥4 = 2 − 3⁄2 = 1⁄2, △ 𝑥5 = (5⁄2) − 2 = 1⁄2, △ 𝑥6 = 3 − 5⁄2 = 1⁄2

Therefore by (1),

1 1 1 1 9 1 1 25 1
𝐿𝑓 (℘′ ) = 0 ∙ + ∙ + 1 ∙ + ∙ + 4 ∙ + ∙
2 4 2 2 2 2 2 4 2
1 1 9 25
= 2 (4 + 1 + 2 + 4 + )=8
4

Example 2: Let 𝑓(𝑥) = 1 + sin 𝑥 for 0 ≤ 𝑥 ≤ 2𝜋. Find 𝐿𝑓 (℘) for the partition
2𝜋 4𝜋
℘ = {0, , 𝜋, , 2𝜋}.
3 3

The graph of 𝑓 is the graph of sin 𝑥 shifted vertically upward 1 unit .Calculating the
2𝜋 2𝜋 4𝜋 4𝜋
minimum value of f on each of the subintervals [0, ], [ 3 , 𝜋], [𝜋, ], and [ 3 , 2𝜋]
3 3

associated with ℘ we find that


𝑚1 = 𝑓(0) = 1, 𝑚2 = 𝑓(𝜋) = 1,

𝑚3 = 𝑓(4𝜋⁄3) = 1 − √3⁄2, 𝑚4 = 𝑓(3𝜋⁄2) = 0.


Moreover,
△ 𝑥1 = 2𝜋⁄3, △ 𝑥2 = 𝜋⁄3, △ 𝑥3 = 𝜋⁄3, △ 𝑥4 = 2𝜋⁄3
From (1) it follows that
𝐿𝑓 (℘) = 𝑚1 △ 𝑥1 + 𝑚2 △ 𝑥2 + ⋯ + 𝑚𝑛 △ 𝑥𝑛

= 1(2𝜋⁄3) + 1(𝜋⁄3) + (1 − √3⁄2) (𝜋⁄3) + 0(2𝜋⁄3)

= 4𝜋⁄3 − (𝜋√3⁄6)

In a similar manner to the one that involves inscribing rectangles to compute a


lower sum, one can also circumscribe rectangles and compute another sum the so
called, an upper sum. To this end, let
℘ = {𝑥0 , 𝑥1 , … , 𝑥𝑛 }

187
APPLIED MATHEMATICS I MODULE

be a given partition of the interval [a, b] in which the function f is continuous and
nonnegative. Then, by the maximum – Minimum Theorem for each k between 1 and n
there exists a largest value of 𝑀𝑘 of f on the 𝑘𝑡ℎ subinterval [𝑥𝑘−1 , 𝑥𝑘 ].
Now, if we let 𝑀𝑘 to be the height of the 𝑘𝑡ℎ rectangle 𝑅𝑘 then 𝑅𝑘 will be the smallest
possible rectangle that circumscribes the appropriate portion of the region which lies
under the graph of f (Figure 4.6). The area of 𝑅𝑘 is𝑀𝑘 ∆𝑥𝑘 , and the sum
𝑀1 ∆𝑥1 + 𝑀2 ∆𝑥2 + ⋯ + 𝑀𝑛 ∆𝑥𝑛
of the areas of the circumscribed rectangles should be no smaller than the area of the
required region R. We denote this sum 𝑈𝑓 (℘) and call it the upper sum of f associated
with the partition ℘. Thus,
𝑈𝑓 (℘) = 𝑀1 ∆𝑥1 + 𝑀2 ∆𝑥2 + ⋯ + 𝑀𝑛 ∆𝑥𝑛 (2)
Even though it is difficult to find the actual area of the region R, it cannot be larger than
𝑈𝑓 (℘) for any partition ℘of [a, b].

𝑦 𝑀𝑛

𝑓 𝑅𝑛
𝑅2 𝑅3

𝑅1

a = 𝑥0 𝑥1 𝑥2 𝑥3 𝑥4 … 𝑥𝑛−1 𝑥𝑛 = 𝑏 𝑥

Figure 2

Example 3: Let 𝑓(𝑥) = 𝑥 2 for 0 ≤ 𝑥 ≤ 3. Find 𝑈𝑓 (℘) and 𝑈𝑓 (℘′ ) for the partitions

3 1 3 5
(a) ℘ = {0, 1, 2 , 2, 3} (b) ℘′ = {0, 2 , 1, 2 , 2, 2 , 3}

188
APPLIED MATHEMATICS I MODULE

Solution:

a) we have that the subintervals associated with the partition ℘ are [0, 1], [1, 3⁄2], [3⁄2 , 2], and [2,
3] (Figure 4.7) with lengths

△ 𝑥1 = 1 − 0 = 1, △ 𝑥2 = (3⁄2) − 1 = 1⁄2

△ 𝑥3 = 2 − (3⁄2) = 1⁄2, △ 𝑥4 = 3 − 2 = 1

Computing the maximum value of 𝑓 on each of these subintervals, we find that

𝑀1 = 𝑓(1) = 1, 𝑀2 = 𝑓(3⁄2) = 9⁄4


𝑀3 = 𝑓(2) = 4, 𝑀4 = 𝑓(3) = 9

Therefore by (2),

𝑈𝑓 (℘) = 𝑀1 ∆𝑥1 + 𝑀2 ∆𝑥2 + ⋯ + 𝑀𝑛 ∆𝑥𝑛


9 1 1 105
=1∙1+4∙2+4∙2+9∙1= 8

a) It can be seen from Figure 5.8 that the subintervals associated with the partition ℘′ are [0, 1/2],
[1/2,1], [1, 3/2], [3/2,2], [2, 5/2], and [5/2,3]. with lengths

△ 𝑥1 = (1⁄2) − 0 = 1⁄2, △ 𝑥2 = 1 − 1⁄2 = 1⁄2, △ 𝑥3 = (3⁄2) − 1 = 1⁄2, △


𝑥4 = 2 − 3⁄2 = 1⁄2, △ 𝑥5 = (5⁄2) − 2 = 1⁄2, △ 𝑥6 = 3 − 5⁄2 = 1⁄2.

Note:

i) Since 𝑚𝑘 and 𝑀𝑘 are the minimum and maximum values of 𝑓 on the subinterval
[𝑥𝑘−1 , 𝑥𝑘 ] 𝑜𝑓 [𝑎, 𝑏], 𝑚𝑘 ≤ 𝑀𝑘 and hence
𝐿𝑓 (℘) ≤ 𝑈𝑓 (℘)
for any partition ℘ of [a, b].

ii) As the number of subintervals of the interval [a, b] increases, the lower sum increases and the
upper sum decreases. Thus, the more points we insert in to a partition ℘, the closer the numbers
𝐿𝑓 (℘) and 𝑈𝑓 (℘) are to one another. Moreover, if ℘ and ℘′ are partitions of [a, b] with the
number of points in ℘′ fewer than that of ℘, then,
𝐿𝑓 (℘′ ) ≤ 𝐿𝑓 (℘) ≤ 𝑈𝑓 (℘) ≤ 𝑈𝑓 (℘′ )

iii) The area of the region R should be a number between 𝐿𝑓 (℘)and 𝑈𝑓 (℘). That is,

𝐿𝑓 (℘) ≤ 𝐴𝑟𝑒𝑎 𝑜𝑓 𝑅 ≤ 𝑈𝑓 (℘)

189
APPLIED MATHEMATICS I MODULE

Activity 6.2.2

Compute the upper sum and lower sum in each of the following for the indicated partitions

5 3
1. 𝑓(𝑥) = 7; ℘ = {−3, − 2 , − 2 , 0}

2. 𝑓(𝑥) = 𝑥 + 2; ℘ = {−1, 1,2}


1 1 3
3. 𝑓(𝑥) = 𝑥 + 2; ℘ = {−1, − 2 , 0, 2 , 1, 2 , 2}
1 1 3
4. 𝑓(𝑥) = 𝑥 2 ; ℘ = {−1, − 2 , 0, 2 , 1, 2 , 2}
1 1 3
5. 𝑓(𝑥) = 𝑥 4 ; ℘ = {−1, − 2 , 0, 2 , 1, 2 , 2}
−1
6. 𝑓(𝑥) = ; ℘ = {−4, −3, −2, −1}
𝑥
𝜋 𝜋
7. 𝑓(𝑥) = 𝑠𝑖𝑛 𝑥 ; ℘ = {0, , }
4 2
𝜋 𝜋 𝜋 𝜋
8. 𝑓(𝑥) = 𝑠𝑖𝑛 𝑥 ; ℘ = {0, , , , }
6 4 3 2

6.2.3 Riemann Sums

In section 5.3.2, we have seen the properties of lower and upper sums of a function in a
given interval. In this section, we are going to see another sum which lies in between
the lower and upper sums.
Definition 1: Let 𝑓 be continuous on [a, b] and let ℘ = {𝑥0 , 𝑥1 , … , 𝑥𝑛 } be any partition
of [a, b]. For each 𝑘 between 1 and n, let 𝑡𝑘 be an arbitrary number in[𝑥𝑘−1 , 𝑥𝑘 ]. Then
the sum
𝑓(𝑡1 )∆𝑥1 + 𝑓(𝑡2 )∆𝑥2 + ⋯ + 𝑓(𝑡𝑛 )∆𝑥𝑛
is called a Riemann sum for 𝑓 on [a, b] and is denoted by ∑𝑛𝑘=1 𝑓(𝑡𝑘 )∆𝑥𝑘 .
Thus
∑𝑛𝑘=1 𝑓(𝑡𝑘 )∆𝑥𝑘 = 𝑓(𝑡1 )∆𝑥1 + 𝑓(𝑡2 )∆𝑥2 + ⋯ + 𝑓(𝑡𝑛 )∆𝑥𝑛
Note that if 𝑚𝑘 is the minimum value and 𝑀𝑘 is the maximum value of 𝑓 on [𝑥𝑘−1 , 𝑥𝑘 ]
and if 𝑡𝑘 is an arbitrary number in [𝑥𝑘−1 , 𝑥𝑘 ], then
𝑚𝑘 ≤ 𝑓(𝑡𝑘 ) ≤ 𝑀𝑘 for k = 1, 2, 3… n so that
𝐿𝑓 (℘) = 𝑚1 ∆𝑥1 + 𝑚2 ∆𝑥2 + ⋯ + 𝑚𝑛 ∆𝑥𝑛
≤ 𝑓(𝑡1 )∆𝑥1 + 𝑓(𝑡2 )∆𝑥2 + ⋯ + 𝑓(𝑡𝑛 )∆𝑥𝑛

190
APPLIED MATHEMATICS I MODULE

≤ 𝑀1 ∆𝑥1 + 𝑀2 ∆𝑥2 + ⋯ + 𝑀𝑛 ∆𝑥𝑛

= 𝑈𝑓 (℘) (1)

Thus from (1) one can easily conclude that every Riemann sum of a function on a given
interval lies between the lower sum and the upper sum of the function on that interval.
Note that if f is continuous on [a, b], then the maximum minimum theorem implies that
each of 𝑚𝑘 and 𝑀𝑘 has the form 𝑓(𝑡𝑘 ) for an appropriate 𝑡𝑘 in [𝑥𝑘−1 , 𝑥𝑘 ], so that 𝐿𝑓 (℘)
and 𝑈𝑓 (℘) are Riemann sums of f on [a, b]. As the next example indicates, we can find
many Riemann sums for a given function on a given interval.
1 3
Example 2: Let 𝑓(𝑥) = 𝑥 for0 ≤ 𝑥 ≤ 3, and let℘ = {0, 2 , 1, 2 , 2, 3}. Find the Riemann

sum for each of the following choices of 𝑡𝑘 .

a. 𝑡𝑘 = 𝑥𝑘−1 , the left endpoint of the subinterval (Figure3(a)).


b. 𝑡𝑘 = 𝑥𝑘 , the right endpoint of the subinterval (Figure 3(b)).
𝑥𝑘−1 +𝑥𝑘
c. 𝑡𝑘 = , the midpoint of the subinterval (Figure 3c)).
2

1 1 3 3
Solution: The subintervals associated with ℘ are [0, 2] , [2 , 1] , [1, 2] , [2 , 2] , [2,3] with

lengths
1 1 1 1
∆𝑥1 = 2 , ∆𝑥2 = 2 , ∆𝑥3 = 2, ∆𝑥4 = 2 , ∆𝑥5 = 1

a. In this case, the Riemann sum and the lower sum coincide. Thus,

1 1 3 3
𝑚1 = 𝑓(0) = 0, 𝑚2 = 𝑓 (2) = 2, 𝑚3 = 𝑓(1) = 1, 𝑚4 = 𝑓 (2) = 2, 𝑚5 =

𝑓(2) = 2 Hence the Riemann sum is


1 1 1 1 1 3 7
∑5𝑘=1 𝑓(𝑥𝑘−1 ) ∆𝑥𝑘 = 0 ∙ + ∙ + ∙ 1 + ∙ + 1 ∙ 2 =
2 2 2 2 2 2 2

191
APPLIED MATHEMATICS I MODULE

1 3
Figure 6(a) 0 2
1 2
2 3 𝑥

a. In this case, the Riemann sum and the upper sum coincide. Thus,

1 1 3 3
𝑀1 = 𝑓 ( ) = 𝑀2 = 𝑓(1) = 1, 𝑀3 = 𝑓 ( ) = , 𝑀4 = 𝑓(2) = 2, 𝑀5 = 𝑓(3) = 3. Thus the
2 2 2 2

Riemann sum will be

1 1 1 1 3 1 11
∑5𝑘=1 𝑓(𝑥𝑘 ) ∆𝑥𝑘 = ∙ + ∙ 1 + ∙ + ∙ 2 + 1 ∙ 3 =
2 2 2 2 2 2 2

1 3
0 2
1 2
2 3 𝑥

Figure 6 (b)

𝑥0 +𝑥1 0+1⁄2 1
𝑡1 = = =
2 2 4

𝑥1 +𝑥2 1⁄ +1 3
2
𝑡2 = 2
= 2
=4

192
APPLIED MATHEMATICS I MODULE

𝑥2 +𝑥3 1+3⁄2 5
𝑡3 = 2
= 2
=4
𝑥3 +𝑥4 3⁄ +2 7
2
𝑡4 = 2
= 2
=4

𝑥4 +𝑥5 2+3 5
𝑡5 = 2
= 2
=2

Thus,
1 1 3 3
𝑓(𝑡1 ) = 𝑓 (4) = 4 𝑓(𝑡2 ) = 𝑓 (4) = 4

5 5 7 7
𝑓(𝑡3 ) = 𝑓 (4) = 4 𝑓(𝑡4 ) = 𝑓 (4) = 4

5 5
𝑓(𝑡5 ) = 𝑓 ( ) =
2 2

Thus the Riemann sum will be

𝑥𝑘−1 +𝑥𝑘 1 1 1 3 1 5 1 7 5
∑5𝑘=1 𝑓( ) ∆𝑥𝑘 = 4∙2+2∙4+2∙4+2∙4+1∙2
2

1 3 5 7 5 9
=8+8+8+8+2=2

𝑦 𝑓(𝑥) = 𝑥

1 3
0 2
1 2
2 3 𝒙

Figure 3 (c)

193
APPLIED MATHEMATICS I MODULE

Activity 6.2.3

1. Let f(x) = x 2 on [0, 5] and let℘ = {0, 1, 2, 4, 9⁄2 , 5}. Then find the Riemann sum for
each of the following choices of t k .
a. t k = xk−1, the left endpoint of the subinterval
b. t k = xk , the right endpoint of the subinterval
xk−1 +xk
c. t k = , the midpoint of the subinterval
2

6.3 Definite Integral

Definition1: Let 𝑓 be continuous on[𝑎, 𝑏]. The definite integral of 𝑓 from 𝑎 to 𝑏 is the unique
number 𝐼 satisfying
𝐿𝑓 (℘) ≤ I ≤ 𝑈𝑓 (℘)

for every partition ℘ of [a, b]. This integral is denoted by


𝑏
∫ 𝑓(𝑥)𝑑𝑥
𝑎

The symbol ʃ is called an integral sign, the numbers 𝑎 and 𝑏 are called the lower and

Upper limits of integration respectively and the function 𝑓 appearing in the integrals is called the
integrand.

Note that the definite integral depends only on the integrand. The variable 𝑥 appearing in the
integral is a "dummy variable"; it may be replaced by another variable with no change in
meaning. That is

𝑏 𝑏 𝑏
∫𝑎 𝑓(𝑥)𝑑𝑥 = ∫𝑎 𝑓(𝑢)𝑑𝑢 = ∫𝑎 𝑓(𝑡)𝑑𝑡

Now, we formally define area in terms of the definite integral.

194
APPLIED MATHEMATICS I MODULE

Definition2: Let 𝑓 be continuous and non negative on the closed interval[𝑎, 𝑏], then the area of
the region bounded by the graph of𝑓, the x-axis and the vertical lines 𝑥 = 𝑎 and 𝑥 = 𝑏 is given
by

𝑏
𝐴𝑟𝑒𝑎 = ∫ 𝑓(𝑥)𝑑𝑥
𝑎

Figure 4.7

Theorem1: If a function is continuous on a closed interval[𝑎, 𝑏], then it is integrable over [a, b].

Example 1:

6
a. Since the function 𝑓(𝑥) = 2𝑥 + 2 is continuous on [2,6], the integral ∫2 (2𝑥 + 2)𝑑𝑥 exists.
b. Since the function 𝑓(𝑥) = 2𝑥 3 + 2𝑥 is continuous on [2, 4] it is integrable over that interval.
1
c. Since the function 𝑓(𝑥) = is continuous on [3, 5] it is integrable on that interval.
𝑥2
1
d. Since the function 𝑓(𝑥) = 𝑥 2 is not continuous on [-2, 5] it is not integrable on that interval.

Properties of the Definite Integral

𝑏
In the definition of definite integral,∫𝑎 𝑓(𝑥)𝑑𝑥, we implicitly assumed that 𝑎 < 𝑏. But this
definition makes sense even if 𝑎 > 𝑏. In this case,

𝑏 𝑎
∫ 𝑓(𝑥)𝑑𝑥 = − ∫ 𝑓(𝑥)𝑑𝑥
𝑎 𝑏

𝑎
If𝑎 = 𝑏, then∫𝑎 𝑓(𝑥)𝑑𝑥 = 0.

We now develop some basic properties of integrals that will help us to evaluate integrals in a
simple manner.

195
APPLIED MATHEMATICS I MODULE

Theorem2: Suppose 𝑓and 𝑔 are integrable functions over [a, b] and k is any constant number,
then 𝑘𝑓, 𝑓 + 𝑔, and 𝑓 − 𝑔 are integrable over [a, b], and

𝑏
1. ∫𝑎 𝑐𝑑𝑥 = 𝑐(𝑏 − 𝑎), Where c is any constant.
𝑏 𝑏 𝑏
2. ∫𝑎 [𝑓(𝑥) + 𝑔(𝑥)] 𝑑𝑥 = ∫𝑎 𝑓(𝑥)𝑑𝑥 + ∫𝑎 𝑔(𝑥)𝑑𝑥.
𝑏 𝑏
3. ∫𝑎 𝑐𝑓(𝑥)𝑑𝑥 = 𝑐 ∫𝑎 𝑓(𝑥)𝑑𝑥, where c is any constant.
𝑏 𝑏 𝑏
4. ∫𝑎 [𝑓(𝑥) − 𝑔(𝑥)] 𝑑𝑥 = ∫𝑎 𝑓(𝑥)𝑑𝑥 − ∫𝑎 𝑔(𝑥)𝑑𝑥.
5. If f is continuous on [a, b] and if 𝑐 is any point in [a, b], then
𝑏 𝑐 𝑏
∫𝑎 𝑓(𝑥)𝑑𝑥 = ∫𝑎 𝑓(𝑥)𝑑𝑥 + ∫𝑐 𝑔(𝑥)𝑑𝑥 .
𝑏
6. If 𝑓(𝑥) ≥ 0 for 𝑎 ≤ 𝑥 ≤ 𝑏, then∫𝑎 𝑓(𝑥)𝑑𝑥 ≥ 0.
𝑏 𝑏
7. If 𝑓(𝑥) ≥ 𝑔(𝑥) for 𝑎 ≤ 𝑥 ≤ 𝑏, then∫𝑎 𝑓(𝑥)𝑑𝑥 ≥ ∫𝑎 𝑔(𝑥)𝑑𝑥.
8. If 𝑚 ≤ 𝑓(𝑥) ≤ 𝑀 for 𝑎 ≤ 𝑥 ≤ 𝑏, then
𝑏
𝑚(𝑏 − 𝑎) ≤ ∫𝑎 𝑓(𝑥)𝑑𝑥 ≤ 𝑀(𝑏 − 𝑎)

Proof of Property 1: Since f assumes only the value c, it follows that for any partition ℘ =
{𝑥0 , 𝑥1 , … , 𝑥𝑛 } of [a, b] and for any 𝑘 between 1and𝑛, we have

𝑚𝑘 = 𝑐 = 𝑀𝑘 .

Consequently

𝐿𝑓 (℘) = 𝑈𝑓 (℘) = 𝑐∆𝑥1 + 𝑐∆𝑥2 + ⋯ + 𝑐∆𝑥𝑛

= 𝑐(∆𝑥1 + ∆𝑥2 + ⋯ + ∆𝑥3 ) = 𝑐(𝑏 − 𝑎)

𝑏
Therefore by definition 5.3.2∫𝑎 𝑐𝑑𝑥 = 𝑐(𝑏 − 𝑎)

2
Example 2: Use the properties of integrals to evaluate∫0 (2 + 5𝑥)𝑑𝑥.

Solution: Using properties 1, 2, and 3 of the integral we have

2 2 2
∫ (2 + 5𝑥)𝑑𝑥 = ∫ 2𝑑𝑥 + ∫ 5𝑥 𝑑𝑥
0 0 0

196
APPLIED MATHEMATICS I MODULE

2 2
= ∫0 2𝑑𝑥 + 5 ∫0 𝑥 𝑑𝑥

𝑥2 2
= 2𝑥|20 + 5 | = 4 + 10 = 14
2 0

8 5 8
Example3: If it is known that ∫0 𝑓(𝑥)𝑑𝑥 = 20 and ∫0 𝑓(𝑥)𝑑𝑥 = 11, find∫5 𝑓(𝑥)𝑑𝑥 .

Solution: By property 2, we have

8 5 8
∫ 𝑓(𝑥)𝑑𝑥 = ∫ 𝑓(𝑥)𝑑𝑥 + ∫ 𝑓(𝑥)𝑑𝑥
0 0 5

Which implies

8 8 5
∫ 𝑓(𝑥)𝑑𝑥 = ∫ 𝑓(𝑥)𝑑𝑥 − ∫ 𝑓(𝑥)𝑑𝑥
5 0 0

= 20−11 = 9.

𝑏
If𝑓(𝑥) ≥ 0, then ∫𝑎 𝑓(𝑥)𝑑𝑥 represents the area under the graph of𝑓, so the geometric
interpretation of property 6 is simply that areas are positive. Property 7 says that a bigger
function has a bigger integral. This follows directly from properties 6 and 4 because𝑓 − 𝑔 ≥ 0.

Proof of property 8: Since𝑚 ≤ 𝑓(𝑥) ≤ 𝑀, property 7 gives

𝑏 𝑏 𝑏
∫𝑎 𝑚 𝑑𝑥 ≤ ∫𝑎 𝑓(𝑥)𝑑𝑥 ≤ ∫𝑎 𝑀𝑑𝑥.

Using property 1 to evaluate the integrals on the left and right sides, we obtain

𝑏
𝑚(𝑏 − 𝑎) ≤ ∫𝑎 𝑓(𝑥)𝑑𝑥 ≤ 𝑀(𝑏 − 𝑎)

1
Example4: Evaluate ∫−1|𝑥| 𝑑𝑥

Solution: By using the addition property, we do have

1 0 1
∫−1|𝑥| 𝑑𝑥 = ∫−1|𝑥| 𝑑𝑥 + ∫0 |𝑥| 𝑑𝑥

0 1
= ∫−1 −𝑥 𝑑𝑥 + ∫0 𝑥 𝑑𝑥

197
APPLIED MATHEMATICS I MODULE

1 1 1 1
= − (2 𝑥 2 ) |0−1 + (2 𝑥 2 ) |10 = 2 + 2 = 1

Example5: Using the comparison property, find the lower and upper bounds for

1
∫−1 √1 + 𝑥 4 𝑑𝑥

Solution: 𝑓(𝑥) = √1 + 𝑥 4 is continuous on the interval [-1, 1] and

1
𝑓 ′ (𝑥) = √1+𝑥 4 4𝑥 3 . The critical point is 𝑥 = 0 which is in the interval [-1, 1]. Maximum-

Minimum Theorem it assumes its maximum and its minimum on [-1, 1]. These values occur
either at the end points or at the critical points. Now,

𝑓(−1) = √2 = 𝑓(1) Which is maximum value and 𝑓(0) = 1which minimum value is.
Therefore, by the comparison property we have

1
2 = 1[1 − (−1)] ≤ ∫−1 √1 + 𝑥 4 𝑑𝑥 ≤ √2[1 − (−1)] = 2√2

So that

1
2 ≤ ∫−1 √1 + 𝑥 4 𝑑𝑥 ≤ 2√2

Hence 2 is a lower bound and 2√2 an upper bound for the given integral.

4
Estimate the values of the integral ∫1 √𝑥 𝑑𝑥.

Activity 6.3

4
1. Show that ∫1 √1 + x 2 dx ≥ 7.5 (Hint: 1 + x 2 ≥ x 2 ).
4
2. Estimate the values of the integral ∫1 √x dx.

6.4 The Fundamental theorem of calculus


The main purpose of this section is to evaluate definite integrals. The Fundamental Theorem of
Calculus is appropriately named because it establishes a connection the two branches of calculus:

198
APPLIED MATHEMATICS I MODULE

differential calculus and integral calculus. The first part of the fundamental Theorem of calculus
deals with functions defined by an equation of the form
𝑥
𝑔(𝑥) = ∫𝑎 𝑓(𝑡)𝑑𝑡

Where 𝑓 is continuous function on [𝑎, 𝑏] and 𝑥 varies between 𝑎 and𝑏. Observe that 𝑔 depends
only on𝑥, which appears as the variable upper limit in the integral. If x is a fixed number then the
integral is a definite number.

Theorem 1 (Fundamental theorem of calculus part I)

If 𝑓 is continuous on [a, b], then the function defined by


𝑥
𝑔(𝑥) = ∫ 𝑓(𝑡)𝑑𝑡 𝑎≤𝑥≤𝑏
𝑎

is continuous on [a, b] and differentiable on (𝑎, 𝑏), and 𝑔′ (𝑥) = 𝑓(𝑥).

𝑥
Example1: Find the derivative of𝑔(𝑥) = ∫0 √1 + 𝑡 2 𝑑𝑡.

Solution: Since 𝑓(𝑡) = √1 + 𝑡 2 is continuous, Fundamental Theorem of Calculus Part I gives

𝑔′ (𝑥) = √1 + 𝑥 2 .
𝑥
Example 2: Find the derivative of the function 𝑔(𝑥) = ∫0 𝑡 sin(𝑡 3 ) 𝑑𝑡.

Solution: The function ℎ(𝑡) = 𝑡 sin(𝑡 3 ) is continuous, application of the fundamental theorem of
calculus part I gives

𝑔′ (𝑥) = 𝑥 sin(𝑥 3 ).
𝑥
Example 3: Find the derivative of the function 𝑔(𝑥) = ∫0 2𝑡 cos(𝑡 2 ) 𝑑𝑡.

Solution: The function 𝑓(𝑡) = 2𝑡 cos(𝑡 2 ) is continuous, by part I of the Fundamental Theorem
of Calculus we have

𝑔′ (𝑥) = 2𝑡 cos(𝑡 2 ).

199
APPLIED MATHEMATICS I MODULE

The Fundamental Theorem of Calculus, part I enables us to find a function F(x) whose derivative
is a given function f(x). In addition to this it asserts that every continuous function on [a, b] is a
derivative of some function.

Theorem2 (Fundamental theorem of calculus part II)

Suppose 𝑓 is continuous on a closed interval[𝑎, 𝑏], then

 f ( x)dx  F (b)  F (a)


a

Where F is any anti derivative of f, that is, function such that F’=f

3
Example 4: Evaluate ∫1 𝑒 𝑥 .

Solution: the function 𝑓(𝑥)= 𝑒 𝑥 is continuous everywhere and we know that derivative of
𝐹(𝑥) = 𝑒 𝑥 so part 2of the fundamental theorem of calculus gives

3
∫ 𝑒 𝑥 = 𝐹(3) − 𝐹(1) = 𝑒 3 − 𝑒
1

6 𝑑𝑥
Example5: Evaluate ∫3
𝑥

Solution The given integral is

6
1
∫ 𝑑𝑥
3 𝑥

1
An anti-derivative of 𝑓(𝑥) = 𝑥 is 𝐹(𝑥) = ln(𝑥) and because 3≤x≤6, we can write 𝐹(𝑥) =

ln(𝑥).so

61 6 6
∫3 𝑥 𝑑𝑥= ln𝑥] 3= ln6-ln3 =ln3 =ln2

200
APPLIED MATHEMATICS I MODULE

Activity 6.4

1. Evaluate the integral

2 5
1.∫−1 𝑥 3 − 2𝑥𝑑𝑥 2. ∫−2 6 𝑑𝑥

4 83
3. ∫1 (5 − 2𝑡 + 3𝑡 2 )𝑑𝑥 4. ∫1 √𝑥𝑑𝑥

𝜋
2𝜋
5.∫𝜋 𝑐𝑜𝑠𝜃𝑑𝜃 6. ∫04 𝑠𝑒𝑐𝜃𝑡𝑎𝑛𝜃𝑑𝜃

2. Evaluate

𝜋 𝑠𝑖𝑛𝑥 𝑖𝑓 0 ≤ 𝑥 < 𝜋⁄2


1.∫0 𝑓(𝑥)𝑑𝑥 Where 𝑓(𝑥)={
𝑐𝑜𝑠𝑥 𝑖𝑓 0 ≤ 𝑥 < 𝜋⁄2

𝜋 𝑠𝑖𝑛𝑥 𝑖𝑓 0 ≤ 𝑥 < 𝜋⁄2


2. ∫0 𝑓(𝑥)𝑑𝑥 Where 𝑓(𝑥)={
𝑐𝑜𝑠𝑥 𝑖𝑓 0 ≤ 𝑥 < 𝜋⁄2

6.5 Techniques of integration

Overview: In this section, we are going to discuss Integration by substitution, by parts and by
partial fraction, Trigonometric integrals, Integration by trigonometric substitution. That is we
develop techniques for using the basic integration formulas

To obtain indefinite integrals of more complicated functions. Integration is not as straightforward


as differentiation; there are no rules that absolutely guarantee obtaining an indefinite integral of a
function. Therefore we discuss a strategy for integration.

6.5.1 Integration by Substitution

The formulas for indefinite integrals in Table (6.1) are limited in scope, because we cannot use
them directly to evaluate such as

 3x  5dx or  cos(5x)dx

201
APPLIED MATHEMATICS I MODULE

In this section we shall develop a simple but powerful method for changing the variable of
integration so that these integrals (and many others) can be evaluated by using the formulas in
Table (6.1).

Method of Substitution

If the integral to be evaluated is of the form


'
f ( g ( x) g ' ( x)dx

We substitute u  g ( x) and du  g ' ( x)dx, then the integral becomes  f (u)du


Example1: Evaluate∫ √2𝑥 + 1 𝑑𝑥.

Solution: let 𝑢 = 2𝑥 + 1.the𝑑𝑢 = 2𝑑𝑥, so 𝑑𝑥 = 𝑑𝑢⁄2.then the substitution rule gives

𝑑𝑢 1 1
∫ √2𝑥 + 1 𝑑𝑥 = ∫ √𝑢 = ∫ 𝑢 ⁄2 𝑑𝑢
2 2
3
1 𝑢 ⁄2 1 3⁄
= 2 3⁄2
+c=3 𝑢 2 +c

1 3⁄
=3 (2𝑥 + 1) 2 +c
Example 2: Evaluate ∫ 𝑒 5𝑥 𝑑𝑥
1
Solution: If we let𝑢 = 5𝑥, then𝑑𝑢 = 5𝑑𝑥, so 𝑑𝑥 = 5 𝑑𝑢. Therefore

1 1 1
∫ 𝑒 5𝑥 𝑑𝑥=5 ∫ 𝑒 𝑢 𝑑𝑢 = 5 𝑒 𝑢 + 𝑐 = 5 𝑒 5𝑥 + 𝑐

Example 3: Evaluate ∫ 𝑡𝑎𝑛𝑥𝑑𝑥

Solution:

First we write the term in terms of sine and cosine:

𝑠𝑖𝑛𝑥
∫ 𝑡𝑎𝑛𝑥𝑑𝑥 = ∫ 𝑑𝑥
𝑐𝑜𝑠𝑥

This suggests that we should substitute 𝑢 = 𝑐𝑜𝑠𝑥 since then 𝑑𝑢 = −𝑠𝑖𝑛𝑥𝑑𝑥 and so 𝑠𝑖𝑛𝑥 𝑑𝑥 =
−𝑑𝑢:
𝑠𝑖𝑛𝑥 𝑑𝑢
∫ 𝑡𝑎𝑛𝑥𝑑𝑥 = ∫ 𝑐𝑜𝑠𝑥 𝑑𝑥=− ∫ 𝑢

202
APPLIED MATHEMATICS I MODULE

= −ln(𝑢) + 𝑐 = ln(𝑐𝑜𝑠𝑥) + 𝑐
Since – ln(𝑐𝑜𝑠𝑥 ) = ln(𝑐𝑜𝑠𝑥) = 𝑙𝑛 (1⁄𝑐𝑜𝑠𝑥) = ln(𝑠𝑒𝑐𝑥).then

∫ 𝑡𝑎𝑛𝑥𝑑𝑥 = ln(𝑠𝑒𝑐𝑥) + 𝑐

It is not always easy to decide what substitution u = g(x) is needed to transform an indefinite
integral into a form that can be readily evaluated. It may be necessary to try several different
possibilities before finding a suitable substitution. In most cases no substitution will simplify the
integrand properly. The following guidelines may be helpful.

Guidelines for changing variables in indefinite integrals:

1. Decide on a reasonable substitution u = g(x).

2. Calculate du  g ' ( x)dx .

3. Using 1 and 2, try to transform the integral into a form that involves only the variable u.
If necessary, introduce a constant factor k into the integrand and compensate by 1/k. If
any part of the resulting integrand contains the variable x, use a different substitution in
1.

4. Evaluate the integral obtained in 3, obtaining an ant derivative involving u.

5. Replace u in the ant derivative obtained in guideline 4 by g(x). The final result should
Contain only the variable x.

The following examples illustrate the use of the guidelines.

Example 4: Evaluate  x 2 (3x 3  2)10 dx .

Solution: If an integrand involves an expression raised to a power, such as (3x 3  2)10 ,

we often substitute u for the expression. Thus, we let

1
u  3x 2  2, du  9 x 2 dx  du  x 2 dx
9

203
APPLIED MATHEMATICS I MODULE

Comparing du  9 xdx with x 2 dx in the integral suggests that we introduce the factor 9 into the
integrand. Doing this and compensating by multiplying the integral by 1/9, we obtain the
following:

1
x (3x 3  2)10 dx   u 10 du
2

9
1
  u 10du
9
1 u 11
 ( )C
9 11

=
1
99
3x 3  2  C
11

Example 5: Evaluate  x 3x 1dx .

Solution: To simplify the expression 3x  1 ,we let

u = 3x − 1, so that du = 3dx

1
du
 u
 3
Then  x 3x  1dx   x 3x  1dx
Thus we still need to find x in terms of u. From the equation u=3x –1 we deduce that

1
x  (u  1)
3

Therefore

204
APPLIED MATHEMATICS I MODULE

1
du
u  3
(u  1) du
 x 3x  1dx   x 3x  1dx   3 u 3
1  2 
3 1
   u  u 2 du

9  
1 2 2 2 2 
5 3
  u  u   C

95 3 
1 2 
5 3
2
  (3x  1)  (3x  1) 2   C
2
95 3 

Example 6: Evaluate  xe x dx .
2

Solution: We let
du
u  x 2 , du  2 xdx   xdx .Then
2
du
 xe dx   e xdx   e
2 2
x x u

2
1
 eu  C
2
1
 ex  C
2

Example7: Evaluate  sec xdx .

Solution: We first put the integral in the form

(sec x  tan x) sec 2 x  sec x tan x


 sec xdx   sec x sec x  tan x
dx  
sec x  tan x
dx

If we now let u  sex  tan x, du  (sec x tan x  sec 2 x)dx .Then

sec 2 x  sec x tan x


 sec xdx   sec x  tan x
dx

1
  du
u
 ln u  C
 ln sec x  tan x  C

205
APPLIED MATHEMATICS I MODULE

Activity 6.5.1
Evaluate the following integrals.
𝑒𝑥
1.∫ 𝑒 −𝑥 𝑑𝑥 2. ∫ 𝑒 𝑥 +1 𝑑𝑥
𝑐𝑜𝑠𝑥
3.∫(3𝑥 − 2)20 𝑑𝑥 4.∫ 𝑠𝑖𝑛2 𝑥 𝑑𝑥
2
5.∫(𝑥 + 1)√2𝑥 + 𝑥 2 𝑑𝑥 6.∫1 𝑥√𝑥 − 1 𝑑𝑥
4 𝑥 3𝑠𝑖𝑛𝑥
7. ∫0 𝑑𝑥 8.∫ 1+𝑐𝑜𝑠𝑥 𝑑𝑥
√1+2𝑥
1
2 𝑒 ⁄𝑥 2 𝑡
9.∫1 𝑥2
10,∫−1 𝑑𝑥
√𝑡+2

6.5.2 Integration by Parts

If we try to evaluate integrals of the type  xe x dx , and  ln x dx by using the method of substitution
we obviously fail. But don’t worry the next theorem will enable us to evaluate not only these, but
also many other types of integrals.
Every differentiation rule has a corresponding integration rule. For instance, the Substitution Rule
for integration corresponds to the Chain Rule for differentiation. The rule that corresponds to the
Product Rule for differentiation is called the rule for integration by parts.
Integration by parts is a technique for simplifying integrals of the form  f xg xdx It is useful
when ƒ can be differentiated repeatedly and g can be integrated repeatedly without difficulty.

Integration by Parts Formula

If u  f ( x) and v  g ( x) and if f ' and g ' are continuous, then

 f ( x) g ( x)dx  f ( x) g ( x)   g ( x) f
' '
( x)dx

 udv  uv   udv

206
APPLIED MATHEMATICS I MODULE

Proof: By the product rule

[ f ( x) g ( x)]'  f ' ( x) g ( x)  f ( x) g ' ( x)

Or equivalently, f ( x) g ' ( x)  [ f ( x) g ( x)]  g ( x) f ' ( x)

Integrating both sides of the last equation gives us

 f ( x) g ( x)dx  [ f ( x) g ( x)] dx   f ( x) g (x)dx


' ' '

The first integral on the right side equals f(x)g(x)+c. Since another constant of integration

is obtained from the second integral, we may omit c in the formula; that is

 f ( x) g ( x)dx  f ( x) g ( x)   g ( x) f
' '
( x)dx
(1)

Since dv  g ' ( x)dx and du  f ' ( x)dx we may write the preceding formula as

 udv  uv   udv
Since applying (1) involves splitting the integrand into two parts, the use of (1) is referred to as
integrating by parts. A proper choice for dv is crucial. We usually let dv equal the most
complicated part of the integrand that can be readily integrated. The following examples illustrate
this method of integration by parts

Example1: Evaluate∫ 𝑥𝑠𝑖𝑛𝑥𝑑𝑥.

Solution: The integrand 𝑥𝑠𝑖𝑛𝑥 can be split into two parts x and 𝑠𝑖𝑛𝑥. Let

𝑓(𝑥) = 𝑥 and 𝑔′ (𝑥) = 𝑠𝑖𝑛𝑥,then 𝑓 ′ (𝑥) = 1 and 𝑔(𝑥) = −𝑐𝑜𝑠𝑥.

Thus, using the formula we have ∫ 𝑥𝑠𝑖𝑛𝑥𝑑𝑥 =𝑓(𝑥)𝑔(𝑥) − ∫ 𝑔(𝑥)𝑓 ′ (𝑥)dx

= 𝑥(−𝑐𝑜𝑠𝑥) − ∫ −𝑐𝑜𝑠𝑥 𝑑𝑥

= −𝑥𝑐𝑜𝑠𝑥 + ∫ 𝑐𝑜𝑠𝑥𝑑𝑥

207
APPLIED MATHEMATICS I MODULE

= −𝑥𝑐𝑜𝑠𝑥 + 𝑠𝑖𝑛𝑥 + 𝑐

Example 2: Evaluate ∫ 𝒍𝒏𝒙𝒅𝒙

Solution: The integrand 𝑙𝑛𝑥 can be split into two parts 1 and 𝑙𝑛𝑥 . We let

𝑢 = 𝑙𝑛𝑥 𝑑𝑣 = 𝑑𝑥
1
Then 𝑑𝑢 = 𝑥 𝑑𝑥 𝑣=𝑥

𝑑𝑥
Integrating by part we get ∫ 𝑙𝑛𝑥𝑑𝑥 = 𝑥 𝑙𝑛𝑥-∫ 𝑥 𝑥

= 𝑥 𝑙𝑛𝑥-∫ 𝑑𝑥

= 𝑥 𝑙𝑛𝑥-𝑥 + 𝑐

2

x
2
Example 3: Evaluate sin(2 x)dx.
0

Solution: Let u  x 2 and dv  sin 2 xdx

1
Then, du  2 xdx and v  cos 2 x
2

Thus using integration by parts we have;


  

 1 2   1 
2 2
2
0 x sin(2 x)d   2 x cos 2 x 0  0 2 x 2 cos 2 x dx
2

 

 1 2
2
  x 2 cos 2 x    x cos 2 xdx
2 0 0

But, since

 1 2  2 1     2
2

 2 x cos 2 x     cos 2( )  0 
0 2 2 2 8

And

208
APPLIED MATHEMATICS I MODULE

  

 sin 2 x  2
2 2
sin 2 x
0 x cos 2 xdx   x 2  0  0 2 dx

1    1  cos 2 x  2
  sin 2( )  0  
22 2  2 2  0
1   1 1
  cos 2( )  cos 0  [1  1] 
4 2  4 2

Hence,


2
2 1
 x sin(2 x)dx  
2

0
8 2

e
x
Example 4: Evaluate cos xdx.

Solution: We could either let dv  cos xdx or let dv  e x dx , since each of these expression is
readily integral. Let us choose

u  e x and dv  cos xdx

So that du  e x dx and v  sin x

Then by integrating by parts we have;

e cos xdx  e x sin x   e x sin xdx.


x
(1)

We next apply integration by parts to the integral of the right side of equation (1). Since we chose
a trigonometric form for dv in the first integration by parts, we shall also choose a trigonometric
form for the second. Letting

u  e x and dv  sin xdx so that

du  e x dx and v   cos x

Integrating by parts, we have

e sin xdx  e x ( cos x)   ( cos x)e x dx


x

  e x cos x   cos xe x dx (2)

209
APPLIED MATHEMATICS I MODULE

If we now use equation (2) to substitute on the right side of equation (1), we obtain

e cos xdx  e x sin x  [e x cos x   e x cos xdx ]


x

e cos xdx  e x sin x  e x cos x   e x cos xdx


x

e
x
Adding cos xdx to both sides of the last equation gives us

2  e x cos xdx  e x sin x  e x cos x

Finally, dividing both sides by 2 and adding the constant of integration yields

1
e cos xdx  [e x sin x  e x cos x]  C
x

Remark: Integration by parts is effective with integrals involving a polynomial an exponential, a


polynomial and a logarithmic, or a polynomial and a trigonometric function. More specifically,
integration by parts is especially well adapted to integrals of the form

∫(𝑝𝑜𝑙𝑦𝑛𝑜𝑚𝑖𝑎𝑙) sin 𝑎𝑥 𝑑𝑥, ∫(𝑝𝑜𝑙𝑦𝑛𝑜𝑚𝑖𝑎𝑙) cos 𝑎𝑥 𝑑𝑥

∫(𝑝𝑜𝑙𝑦𝑛𝑜𝑚𝑖𝑎𝑙) 𝑒 𝑎𝑥 𝑑𝑥, ∫(𝑝𝑜𝑙𝑦𝑛𝑜𝑚𝑖𝑎𝑙) ln 𝑥 𝑑𝑥

Example 5: Evaluate∫ 𝑡𝑎𝑛−1 𝑥𝑑𝑥.

Solution: Let
1
𝑢 = 𝑡𝑎𝑛−1 𝑎𝑛𝑑 𝑑𝑣 = 𝑑𝑥 𝑡ℎ𝑎𝑡 𝑑𝑢 = 1+𝑥 2 𝑑𝑥 𝑎𝑛𝑑 𝑣 = 𝑥

Then
𝑥
∫ 𝑡𝑎𝑛−1 𝑥𝑑𝑥 = 𝑥𝑡𝑎𝑛−1 𝑥 − ∫ 1+𝑥 2 𝑑𝑥

Now we use substitution to solve the integral to the right. That is let
1
𝑡 = 1 + 𝑥 2 ,𝑑𝑡 = 2𝑥𝑑𝑥 . so 𝑥𝑑𝑥 = 2 𝑑𝑡

We have
1 1 𝑑𝑡 1
∫ 1+𝑥 2 𝑑𝑥 =2 ∫ 𝑡
=2 𝑙𝑛𝑡 + 𝑐

210
APPLIED MATHEMATICS I MODULE

Consequently
1
∫ 𝑡𝑎𝑛−1 𝑥𝑑𝑥 = 𝑥𝑡𝑎𝑛−1 𝑥 − 2 𝑙𝑛𝑡 + 𝑐

Reduction Formulas
Integration by parts is applicable in finding integrals of the form ∫ 𝑠𝑖𝑛𝑛 𝑥 𝑑𝑥 and ∫ 𝑐𝑜𝑠 𝑛 𝑥 𝑑𝑥 for
𝑛 ≥ 2.
Recall that𝑠𝑖𝑛𝑛 𝑥 = (𝑠𝑖𝑛𝑛−1 𝑥)(sin 𝑥), and let
𝑢 = 𝑠𝑖𝑛𝑛−1 𝑥 and 𝑑𝑣 = sin 𝑥 𝑑𝑥
Then 𝑑𝑢 = (𝑛 − 1)𝑠𝑖𝑛𝑛−2 𝑥 cos 𝑥 𝑑𝑥 and 𝑣 = − cos 𝑥
Therefore,

∫ 𝑠𝑖𝑛𝑛 𝑥 𝑑𝑥 = ∫ 𝑠𝑖𝑛𝑛−1 𝑥 sin 𝑥 𝑑𝑥


= (−𝑐𝑜𝑠𝑥)𝑠𝑖𝑛𝑛−1 𝑥— ∫(− cos 𝑥) (𝑛 − 1)𝑠𝑖𝑛𝑛−2 𝑥 cos 𝑥 𝑑𝑥
= −𝑠𝑖𝑛𝑛−1 𝑥 𝑐𝑜𝑠𝑥 + (𝑛 − 1) ∫ 𝑠𝑖𝑛𝑛−2 𝑥 𝑐𝑜𝑠 2 𝑥 𝑑𝑥
= −𝑠𝑖𝑛𝑛−1 𝑥 𝑐𝑜𝑠𝑥 + (𝑛 − 1) ∫ 𝑠𝑖𝑛𝑛−2 𝑥 (1 − 𝑠𝑖𝑛2 𝑥) 𝑑𝑥
Thus

∫ 𝑠𝑖𝑛𝑛 𝑥 𝑑𝑥 = −𝑠𝑖𝑛𝑛−1 𝑥 𝑐𝑜𝑠𝑥 + (𝑛 − 1) ∫ 𝑠𝑖𝑛𝑛−2 𝑥 𝑑𝑥 − (𝑛 − 1) ∫ 𝑠𝑖𝑛𝑛 𝑥 𝑑𝑥


Combining the terms that contain ∫ 𝑠𝑖𝑛𝑛 𝑥 𝑑𝑥 yields
𝑛 ∫ 𝑠𝑖𝑛𝑛 𝑥 𝑑𝑥 = −𝑠𝑖𝑛𝑛−1 𝑥 𝑐𝑜𝑠𝑥 + (𝑛 − 1) ∫ 𝑠𝑖𝑛𝑛−2 𝑥 𝑑𝑥
Therefore,
1 𝑛−1
∫ 𝑠𝑖𝑛𝑛 𝑥 𝑑𝑥 = − 𝑛 𝑠𝑖𝑛𝑛−1 𝑥 cos 𝑥 + 𝑛
∫ 𝑠𝑖𝑛𝑛−2 𝑥 𝑑𝑥 (1

Example 6: Use integration by parts to show


1 𝑛−1
∫ 𝑐𝑜𝑠 𝑛 𝑥 𝑑𝑥 = 𝑛 𝑐𝑜𝑠 𝑛−1 𝑥 sin 𝑥 + 𝑛
∫ 𝑐𝑜𝑠 𝑛−2 𝑥 𝑑𝑥

Solution: Notice that 𝑐𝑜𝑠 𝑛 𝑥 = (𝑐𝑜𝑠 𝑛−1 𝑥)(cos 𝑥), and let
𝑢 = 𝑐𝑜𝑠 𝑛−1 𝑥 and 𝑑𝑣 = cos 𝑥 𝑑𝑥
Then 𝑑𝑢 = (𝑛 − 1)𝑐𝑜𝑠 𝑛−2 𝑥 (−sin 𝑥) 𝑑𝑥 and 𝑣 = sin 𝑥
Now,

∫ 𝑐𝑜𝑠 𝑛 𝑥 𝑑𝑥 = 𝑐𝑜𝑠 𝑛−1 𝑥 sin 𝑥 − ∫(sin 𝑥) (𝑛 − 1)𝑐𝑜𝑠 𝑛−2 𝑥 (−sin 𝑥) 𝑑𝑥

211
APPLIED MATHEMATICS I MODULE

= 𝑐𝑜𝑠 𝑛−1 𝑥 sin 𝑥 + ∫ 𝑠𝑖𝑛2 𝑥(𝑛 − 1)𝑐𝑜𝑠 𝑛−2 𝑥 𝑑𝑥


= 𝑐𝑜𝑠 𝑛−1 𝑥 sin 𝑥 + (𝑛 − 1) ∫ 𝑐𝑜𝑠 𝑛−2 𝑥 (1 − 𝑐𝑜𝑠 2 𝑥) 𝑑𝑥
= 𝑐𝑜𝑠 𝑛−1 𝑥 sin 𝑥 + (𝑛 − 1)[∫ 𝑐𝑜𝑠 𝑛−2 𝑥 𝑑𝑥 − ∫ 𝑐𝑜𝑠 𝑛 𝑥 𝑑𝑥]
= 𝑐𝑜𝑠 𝑛−1 𝑥 sin 𝑥 + (𝑛 − 1) ∫ 𝑐𝑜𝑠 𝑛−2 𝑥 𝑑𝑥 − (𝑛 − 1) ∫ 𝑐𝑜𝑠 𝑛 𝑥 𝑑𝑥
Combining the terms that contain∫ 𝑐𝑜𝑠 𝑛 𝑥 𝑑𝑥, we have

∫ 𝑐𝑜𝑠 𝑛 𝑥 𝑑𝑥 + (𝑛 − 1) ∫ 𝑐𝑜𝑠 𝑛 𝑥 𝑑𝑥 = 𝑛 ∫ 𝑐𝑜𝑠 𝑛 𝑥 𝑑𝑥


= 𝑐𝑜𝑠 𝑛−1 𝑥 sin 𝑥 + (𝑛 − 1) ∫ 𝑐𝑜𝑠 𝑛−2 𝑥 𝑑𝑥
Therefore,
1 𝑛−1
∫ 𝑐𝑜𝑠 𝑛 𝑥 𝑑𝑥 = 𝑛 𝑐𝑜𝑠 𝑛−1 𝑥 sin 𝑥 + 𝑛
∫ 𝑐𝑜𝑠 𝑛−2 𝑥 𝑑𝑥 (2)

The formulas in (1) and (2) are called reduction formulas, because the exponent is reduced from
𝑛 to 𝑛 − 2.
Example 7: Find

a. ∫ 𝑠𝑖𝑛2 𝑥 𝑑𝑥 c. ∫ 𝑐𝑜𝑠 2 𝑥 𝑑𝑥
b. ∫ 𝑠𝑖𝑛4 𝑥 𝑑𝑥 d. ∫ 𝑐𝑜𝑠 3 𝑥 𝑑𝑥

Solution:

a) Using formula (1) with 𝑛 = 2 we have


1 1
∫ 𝑠𝑖𝑛2 𝑥 𝑑𝑥 = − 2 sin 𝑥 cos 𝑥 + 2 ∫ 𝑠𝑖𝑛2−2 𝑥 𝑑𝑥
1 1
= − 2 sin 𝑥 cos 𝑥 + 2 ∫ 1 𝑑𝑥
1 1
= − 2 sin 𝑥 cos 𝑥 + 2 𝑥 + 𝐶

b) Using formula (1) with 𝑛 = 4 we have


1 3
∫ 𝑠𝑖𝑛4 𝑥 𝑑𝑥 = − 4 𝑠𝑖𝑛3 𝑥 cos 𝑥 + 4 ∫ 𝑠𝑖𝑛2 𝑥 𝑑𝑥 and from (a) we have
1 3 1 1
∫ 𝑠𝑖𝑛4 𝑥 𝑑𝑥 = − 𝑠𝑖𝑛3 𝑥 cos 𝑥 + (− sin 𝑥 cos 𝑥 + 𝑥 + 𝐶)
4 4 2 2
1 3 3
= − 4 𝑠𝑖𝑛3 𝑥 cos 𝑥 − 8 sin 𝑥 cos 𝑥 + 8 𝑥 + 𝐾

c) Using formula (2) with 𝑛 = 2 we have


1 1
∫ 𝑐𝑜𝑠 𝑥 𝑑𝑥 = 2 sin 𝑥 cos 𝑥 + 2 ∫ 𝑐𝑜𝑠 2−2 𝑥 𝑑𝑥
1 1
= 2 sin 𝑥 cos 𝑥 + 2 ∫ 1 𝑑𝑥

212
APPLIED MATHEMATICS I MODULE

1 1
= sin 𝑥 cos 𝑥 + 𝑥 + 𝐶
2 2

d) Using formula (2) with 𝑛 = 4 and the result in (c) we get


1 3
∫ 𝑐𝑜𝑠 4 𝑥 𝑑𝑥 = 4 𝑐𝑜𝑠 3 𝑥 sin 𝑥 + 4 ∫ 𝑐𝑜𝑠 2 𝑥 𝑑𝑥
1 3 1 1
= 4 𝑐𝑜𝑠 3 𝑥 sin 𝑥 + 4 (2 sin 𝑥 cos 𝑥 + 2 𝑥 + 𝐶)

1 3 3
= 4 𝑠𝑖𝑛3 𝑥 cos 𝑥 + 8 sin 𝑥 cos 𝑥 + 8 𝑥 + 𝐾

Activity 6.5.2
Evaluate the following integerals.
1.∫ 𝑥𝑙𝑛𝑥𝑑𝑥 2.∫ 𝑥𝑒 −𝑥 𝑑𝑥
3. ∫ 𝑡𝑠𝑖𝑛2𝑡𝑑𝑡 4. ∫ 𝑥𝑐𝑜𝑠3𝑥𝑑𝑥

5.∫ 𝑠𝑒𝑐 2 𝑥𝑑𝑥 6.∫ sin(lnx) 𝑑𝑥 , (ℎ𝑖𝑛𝑡: 𝑙𝑒𝑡 𝑢 = sin(𝑙𝑛𝑥))

7.∫ 𝑠𝑖𝑛−1 𝑥𝑑𝑥 8.∫ 𝑥 2 sin(2𝑥)𝑑𝑥

9.∫ cos(√𝑥)dx 10.∫ sin(𝑙𝑛𝑥) 𝑑𝑥


2.Evalute the integral with the help of the reduction formulas.

6.5.3 Integration by Partial Fraction

In this section we are going to develop strategies by which we can integrate rational functions. If
we are given a rational function we express it as a sum of simpler fractions, called partial fractions,
which we already know how to integrate.

1 3
To make the method simple, take the fractions and and take a common denominator to
𝑥+3 𝑥−2

obtain

213
APPLIED MATHEMATICS I MODULE

1 3 𝑥−2−3(𝑥+3) −2𝑥−11
− = =
𝑥+3 𝑥−2 (𝑥+3)(𝑥−2) 𝑥 2 +𝑥−6

So, integrating the expression on the left hand side is simpler than integrating the one on the right
hand side. So,

−2𝑥−11 1 1
∫ 𝑥 2+𝑥−6 𝑑𝑥 = ∫ 𝑥+3 𝑑𝑥 − 3 ∫ 𝑥−2 𝑑𝑥

= ln|𝑥 + 3| − 3 ln|𝑥 − 2| + 𝐶

To see how the method of partial fractions works in general, let’s consider a rational function
𝑃(𝑥)
𝑓(𝑥) = 𝑄(𝑥)

Where P and Q are polynomials. If the degree of P is less than the degree of Q, it is possible to
express 𝑓 as a sum of simpler fractions. Such a rational function is called proper. If

𝑃(𝑥) = 𝑎𝑛 𝑥 𝑛 + 𝑎𝑛−1 𝑥 𝑛−1 + ⋯ + 𝑎1 𝑥 + 𝑎0

Where𝑎𝑛 ≠ 0, then the degree of P is n and we write deg(𝑃) = 𝑛.

If 𝑓 is improper, that is deg(𝑃) ≥ deg(𝑄), one needs to take the preliminary step of dividing Q
in to P (by long division) until a remainder R(x) is obtained such that deg(𝑅) < deg(𝑄). That is,

𝑃(𝑥) 𝑅(𝑥)
𝑓(𝑥) = 𝑄(𝑥) = 𝑇(𝑥) + 𝑄(𝑥)

Where T and R are also polynomials.

𝑥 2 +3𝑥
Example 1: Find ∫ 𝑑𝑥
𝑥+1

Solution: Since the degree of the numerator is greater than the degree of the denominator, we first
perform the long division and we get

𝑥 2 +3𝑥 2
∫ 𝑑𝑥 = ∫ (𝑥 + 2 − ) 𝑑𝑥
𝑥+1 𝑥+1

2
= ∫(𝑥 + 2) 𝑑𝑥 − ∫ 𝑑𝑥
𝑥+1

𝑥2
= + 2𝑥 − 2 ln|𝑥 + 1| + 𝐶
2

214
APPLIED MATHEMATICS I MODULE

Sometimes it is important to factor the denominator. Any polynomial Q can be factored as a


product of linear factors (factors of the form𝑎𝑥 + 𝑏) and irreducible quadratic factors (factors of
the form 𝑥 2 + 𝑏𝑥 + 𝑐, where 𝑏 2 − 4𝑎𝑐 < 0).

𝑅(𝑥)
After factoring𝑄(𝑥), the next step is to express the proper rational function 𝑄(𝑥) as a sum of partial

fractions of the form

𝐴 𝐴𝑥+𝐵
or
(𝑎𝑥+𝑏)𝑖 (𝑎𝑥 2 +𝑏𝑥+𝑐)𝑗

CASE I: The denominator Q(x) is a product of distinct linear factors

This means that we can write


𝑄(𝑥) = (𝑎1 𝑥 + 𝑏1 )(𝑎2 𝑥 + 𝑏2 ) ∙∙∙ (𝑎𝑛 𝑥 + 𝑏𝑛 ) ( ∗)
where no factor is repeated (and no factor a constant multiple of another). In this case the partial
fraction becomes
𝑅(𝑥) 𝐴1 𝐴2 𝐴𝑛
= (𝑎 + (𝑎 + ⋯ + (𝑎
𝑄(𝑥) 1 𝑥+𝑏1 ) 2 𝑥+𝑏2 ) 𝑛 𝑥+𝑏𝑛 )

1
Example 2: Find ∫ 𝑥 2 −1 𝑑𝑥

Solution:
1 1
= (𝑥−1)(𝑥+1)
𝑥 2 −1

By partial decomposition we have


1 𝐴 𝐵 𝐴(𝑥+1)+𝐵(𝑥−1)
(𝑥−1)(𝑥+1)
= 𝑥−1 + 𝑥+1 = (𝑥−1)(𝑥+1)

(𝐴+𝐵)𝑥+𝐴−𝐵
= (𝑥−1)(𝑥+1)

Ignoring equal denominators on the two extreme sides we have


1 = (𝐴 + 𝐵)𝑥 + 𝐴 − 𝐵
from which we get
(i) 𝐴+𝐵 =0 and
(ii) 𝐴−𝐵 =1

215
APPLIED MATHEMATICS I MODULE

1 −1
Solving (𝑖) 𝑎𝑛𝑑 (𝑖𝑖) simultaneously we get 𝐴 = and 𝐵 = . Thus,
2 2

1 1 1⁄ 1⁄
2 2
= (𝑥−1)(𝑥+1) = 𝑥−1 − 𝑥+1
𝑥 2 −1

Therefore,
1 1 1
∫ 𝑥 2 −1 𝑑𝑥 = ∫ 2(𝑥−1) 𝑑𝑥 − ∫ 2(𝑥+1) 𝑑𝑥
1 1
= 2 ln|𝑥 − 1| − 2 ln|𝑥 + 1| + 𝐶

𝑥
Example 3: Find ∫ 𝑥 2 −3𝑥+2 𝑑𝑥

Solution:
𝑥 𝑥
= (𝑥−2)(𝑥−1)
𝑥 2 −3𝑥+2

By partial decomposition we have


𝑥 𝐴 𝐵 𝐴(𝑥−1)+𝐵(𝑥−2)
(𝑥−2)(𝑥−1)
= 𝑥−2 + 𝑥−1 = (𝑥−2)(𝑥−1)

(𝐴+𝐵)𝑥−𝐴−2𝐵
= (𝑥−2)(𝑥−1)

Ignoring equal denominators on the two extreme sides we have


𝑥 = (𝐴 + 𝐵)𝑥 − 𝐴 − 2𝐵
from which we get
(i) 𝐴+𝐵 =1 and
(ii) −𝐴 − 2𝐵 = 0
Solving (𝑖) 𝑎𝑛𝑑 (𝑖𝑖) simultaneously we get 𝐴 = 2 and 𝐵 = −1. Thus,
𝑥 𝑥 2 1
= (𝑥−2)(𝑥−1) = 𝑥−2 − 𝑥−1
𝑥 2 −3𝑥+2

Therefore,
𝑥 2 1
∫ 𝑥 2 −3𝑥+2 𝑑𝑥 = ∫ 𝑥−2 𝑑𝑥 − ∫ 𝑥−1 𝑑𝑥

= 2 ln|𝑥 − 2| − ln|𝑥 − 1| + 𝐶
(𝑥−2)2
= ln | |
𝑥−1

CASE II: Q(x) is a product of linear factors, some of which are repeated

Suppose that one linear factor, say (𝑎1 𝑥 + 𝑏1 ) is repeated k times; that is,

216
APPLIED MATHEMATICS I MODULE

𝐴1
(𝑎1 𝑥 + 𝑏1 )𝑘 occurs in the factorization of 𝑄(𝑥). Then in place of the single term in
𝑎1 𝑥+𝑏1

equation 1, we use

𝐴1 𝐴2 𝐴𝑘
(𝑎1 𝑥+𝑏1 )
+ (𝑎 𝑥+𝑏 2
+ ⋯ + (𝑎 𝑘
(∗∗)
1 1) 1 𝑥+𝑏1 )

𝑥 4 −2𝑥 2 +4𝑥+1
Example 4: Find the integral∫ 𝑑𝑥.
𝑥 3 −𝑥 2 −𝑥+1

Solution: Since the degree of the numerator is greater than that of the denominator, we use long
division to reduce the degree of the numerator.
𝑥 4 −2𝑥 2 +4𝑥+1 4𝑥
= 𝑥 + 1 + 𝑥 3 −𝑥 2 −𝑥+1.
𝑥 3 −𝑥 2 −𝑥+1

The next step is to factor the denominator𝑄(𝑥) = 𝑥 3 − 𝑥 2 − 𝑥 + 1. Since𝑄(1) = 0, we know by


factor theorem that 𝑥 − 1 is a factor of 𝑄(𝑥). Thus,
𝑥 3 − 𝑥 2 − 𝑥 + 1 = (𝑥 − 1)(𝑥 2 − 1) = (𝑥 − 1)2 (𝑥 + 1).
The partial decomposition becomes
4𝑥 𝐴 𝐵 𝐶
= 𝑥−1 + (𝑥−1)2 + 𝑥+1
𝑥 3 −𝑥 2 −𝑥+1

Multiplying by the least common multiple, (𝑥 − 1)2 (𝑥 + 1), we have


4𝑥 = 𝐴(𝑥 − 1)(𝑥 + 1) + 𝐵(𝑥 + 1) + 𝐶(𝑥 − 1)2
= (𝐴 + 𝐶)𝑥 2 + (𝐵 − 2𝐶)𝑥 + (−𝐴 + 𝐵 + 𝐶)
Equating the coefficients of equal powers of 𝑥, we get
𝐴+𝐶 = 0
𝐵 − 2𝐶 = 4
−𝐴 + 𝐵 + 𝐶 = 0
Solving the system, we get 𝐴 = 1, 𝐵 = 2, and 𝐶 = −1, so
𝑥 4 − 2𝑥 2 + 4𝑥 + 1 1 2 𝐶
∫ 3 𝑑𝑥 = ∫ [𝑥 + 1 + + − ] 𝑑𝑥
2
𝑥 −𝑥 −𝑥+1 𝑥 − 1 (𝑥 − 1) 2 𝑥+1
𝑥2 2
= + 𝑥 + ln|𝑥 − 1| − 𝑥−1 − ln|𝑥 + 1| + 𝐾
2

𝑥2 2 𝑥−1
= + 𝑥 − 𝑥−1 + ln |𝑥+1| + 𝐾
2

217
APPLIED MATHEMATICS I MODULE

Activity 6.5.3
Find the following integrals by the method of integration by partial fractions.

6.6 Trigonometric Integrals


In this section we use trigonometric identities to integrate certain combinations of trigonometric
functions.

i) Integrals of the form ∫ 𝒔𝒊𝒏𝒎 𝒙 𝒄𝒐𝒔𝒏 𝒙 𝒅𝒙

If 𝒎 = 𝟎 or 𝒏 = 𝟎, then the integral will reduce to reduction formula and hence becomes
easy to integrate using integration by parts.
If 𝒏 = 𝟏, then the integral takes the form ∫ 𝒔𝒊𝒏𝒎 𝒙 𝒄𝒐𝒔 𝒙 𝒅𝒙, which can be evaluated by
substituting 𝒖 = 𝐬𝐢𝐧 𝒙. Similarly, if 𝒎 = 𝟏, then the integral takes the form
∫ 𝒔𝒊𝒏 𝒙 𝒄𝒐𝒔𝒏 𝒙 𝒅𝒙 and this is evaluated by substituting 𝒖 = 𝐜𝐨𝐬 𝒙.

Now we investigate the behavior of integrals of the form∫ 𝒔𝒊𝒏𝒎 𝒙 𝒄𝒐𝒔𝒏 𝒙 𝒅𝒙 in which 𝒎 and 𝒏
positive integers with 𝒎 ≥ 𝟐 and 𝒏 ≥ 𝟐. There are cases to consider:

(a) If n is odd (𝒏 = 𝟐𝒌 + 𝟏), then factor out 𝐜𝐨𝐬 𝒙 and use the identity
𝒄𝒐𝒔𝟐 𝒙 = 𝟏 − 𝒔𝒊𝒏𝟐 𝒙 to write the rest of the integrand in terms of 𝐬𝐢𝐧 𝒙. So,

218
APPLIED MATHEMATICS I MODULE

∫ 𝒔𝒊𝒏𝒎 𝒙 𝒄𝒐𝒔𝒏 𝒙 𝒅𝒙 = ∫ 𝒔𝒊𝒏𝒎 𝒙 𝒄𝒐𝒔𝟐𝒌+𝟏 𝒙 𝒅𝒙

𝒌
= ∫ 𝒔𝒊𝒏𝒎 𝒙 (𝒄𝒐𝒔𝟐 𝒙 ) 𝒄𝒐𝒔 𝒙 𝒅𝒙
𝒌
= ∫ 𝒔𝒊𝒏𝒎 𝒙 (𝟏 − 𝒔𝒊𝒏𝟐 𝒙 ) 𝒄𝒐𝒔 𝒙 𝒅𝒙

Then substitute 𝒖 = 𝐬𝐢𝐧 𝒙 so that 𝒅𝒖 = 𝐜𝐨𝐬 𝒙 𝒅𝒙 and then we apply integration by


substitution.

(b) If m is odd (𝒎 = 𝟐𝒌 + 𝟏), then factor out 𝐬𝐢𝐧 𝒙 and use the identity
𝒔𝒊𝒏𝟐 𝒙 = 𝟏 − 𝒄𝒐𝒔𝟐 𝒙 to write the rest of the integrand in terms of 𝐜𝐨𝐬 𝒙. So,

∫ 𝒔𝒊𝒏𝒎 𝒙 𝒄𝒐𝒔𝒏 𝒙 𝒅𝒙 = ∫ 𝒄𝒐𝒔𝒏 𝒙 𝒔𝒊𝒏𝟐𝒌+𝟏 𝒙 𝒅𝒙

= ∫ 𝒄𝒐𝒔𝒏 𝒙 (𝒔𝒊𝒏𝒙 )𝒌 𝒔𝒊𝒏 𝒙 𝒅𝒙


𝒌
= ∫ 𝒄𝒐𝒔𝒏 𝒙 (𝟏 − 𝒄𝒐𝒔𝟐 𝒙 ) 𝒔𝒊𝒏 𝒙 𝒅𝒙

Then substitute 𝒖 = 𝐜𝐨𝐬 𝒙 so that 𝒅𝒖 = − 𝐬𝐢𝐧 𝐱 𝒅𝒙 and then we apply integration by


substitution.

If both 𝑚 and 𝑛 are odd, either (a) or (b) can be used.

(c) If m and n are both even, we use the following trigonometric identities:
𝟏
𝒔𝒊𝒏𝟐 𝒙 = (𝟏 − 𝐜𝐨𝐬 𝟐𝒙)
𝟐
𝟏
𝒄𝒐𝒔𝟐 𝒙 = 𝟐 (𝟏 + 𝐜𝐨𝐬 𝟐𝒙)
𝟏
𝐬𝐢𝐧 𝒙 𝐜𝐨𝐬 𝒙 = 𝟐 𝐬𝐢𝐧 𝟐𝒙

Example 1: Find ∫ 𝑠𝑖𝑛6 𝑥 𝑐𝑜𝑠 3 𝑥 𝑑𝑥

Solution: Here 𝑚 = 6 and 𝑛 = 3. So, we apply case (a).

∫ 𝑠𝑖𝑛6 𝑥 𝑐𝑜𝑠 3 𝑥 𝑑𝑥 = ∫ 𝑠𝑖𝑛6 𝑥 𝑐𝑜𝑠 2 𝑥 cos 𝑥 𝑑𝑥

= ∫ 𝑠𝑖𝑛6 𝑥 (1 − 𝑠𝑖𝑛2 𝑥 ) cos 𝑥 𝑑𝑥

= ∫ 𝑠𝑖𝑛6 𝑥 cos 𝑥 𝑑𝑥 − ∫ 𝑠𝑖𝑛8 𝑥 cos 𝑥 𝑑𝑥

Let 𝑢 = sin 𝑥 then 𝑑𝑢 = cos 𝑥 𝑑𝑥. Substituting these in to the integral we get

219
APPLIED MATHEMATICS I MODULE

∫ 𝑠𝑖𝑛6 𝑥 𝑐𝑜𝑠 3 𝑥 𝑑𝑥 = ∫ 𝑢6 𝑑𝑢 − ∫ 𝑢8 𝑑𝑢

1 1
= 7 𝑢7 − 9 𝑢9 + 𝐶

1 1
= 7 𝑠𝑖𝑛7 𝑥 − 9 𝑠𝑖𝑛9 𝑥 + 𝐶

Example 2: Find ∫ 𝑠𝑖𝑛3 𝑥 𝑐𝑜𝑠 2 𝑥 𝑑𝑥

Solution: In this case 𝑚 = 3 and 𝑛 = 2. So, we follow case (b).

Thus,

∫ 𝑠𝑖𝑛3 𝑥 𝑐𝑜𝑠 2 𝑥 𝑑𝑥 = ∫ 𝑠𝑖𝑛2 𝑥 sin 𝑥 𝑐𝑜𝑠 2 𝑥 𝑑𝑥

= ∫(1 − 𝑐𝑜𝑠 2 𝑥) sin 𝑥 𝑐𝑜𝑠 2 𝑥 𝑑𝑥

= ∫ 𝑐𝑜𝑠 2 𝑥 sin 𝑥 𝑑𝑥 − ∫ 𝑐𝑜𝑠 4 𝑥 sin 𝑥 𝑑𝑥

Let 𝑢 = cos 𝑥 then 𝑑𝑢 = − sin 𝑥 𝑑𝑥 ⟺ −𝑑𝑢 = sin 𝑥 𝑑𝑥 and by substitution we get

∫ 𝑠𝑖𝑛3 𝑥 𝑐𝑜𝑠 2 𝑥 𝑑𝑥 = ∫ 𝑐𝑜𝑠 2 𝑥 sin 𝑥 𝑑𝑥 − ∫ 𝑐𝑜𝑠 4 𝑥 sin 𝑥 𝑑𝑥

= ∫ 𝑢2 (−𝑑𝑢) − ∫ 𝑢4 (−𝑑𝑢)

= − ∫ 𝑢2 𝑑𝑢 + ∫ 𝑢4 𝑑𝑢

1 1 1 1
= − 3 𝑢3 + 5 𝑢5 + 𝐶 = − 3 𝑐𝑜𝑠 3 𝑥 + 5 𝑐𝑜𝑠 5 𝑥 + 𝐶

Example3: Find ∫ 𝑠𝑖𝑛2 𝑥 𝑐𝑜𝑠 4 𝑥 𝑑𝑥

Solution: Using the aforementioned trigonometric identities we have

∫ 𝑠𝑖𝑛2 𝑥 𝑐𝑜𝑠 4 𝑥 𝑑𝑥 = ∫(sin 𝑥 cos 𝑥)2 𝑐𝑜𝑠 2 𝑥 𝑑𝑥

1 2 1+cos 2𝑥
= ∫ (2 sin 2𝑥) ( ) 𝑑𝑥
2

1 1
= 8 ∫ 𝑠𝑖𝑛2 2𝑥 𝑑𝑥 + 8 ∫ 𝑠𝑖𝑛2 2𝑥 cos 2𝑥 𝑑𝑥

For the first integral on the right we let

220
APPLIED MATHEMATICS I MODULE

𝑑𝑢
𝑢 = 2𝑥, so that 𝑑𝑢 = 2𝑑𝑥 ⟹ = 𝑑𝑥
2

and for the second integral we let

𝑑𝑣
𝑣 = sin 2𝑥, so that 𝑑𝑣 = 2 cos 2𝑥 𝑑𝑥 ⟹ cos 2𝑥 𝑑𝑥 = 2

Thus,

1 1
∫ 𝑠𝑖𝑛2 𝑥 𝑐𝑜𝑠 4 𝑥 𝑑𝑥 = ∫ 𝑠𝑖𝑛2 2𝑥 𝑑𝑥 + ∫ 𝑠𝑖𝑛2 2𝑥 cos 2𝑥 𝑑𝑥
8 8
1 1 1 𝑑𝑣
= ∫(𝑠𝑖𝑛2 𝑢 ) 𝑑𝑢 + ∫ 𝑣 2
8 2 8 2

1 1 1 1 1
= 16 (2 𝑢 − 4 sin 2𝑢) + 16 (3 𝑣 3 ) + 𝐶

1 1 1
= 16 (𝑥 − 4 sin 4𝑥) + 48 𝑠𝑖𝑛3 2𝑥 + 𝐶

Example 4: Find ∫ 𝑠𝑖𝑛3 𝑥 𝑐𝑜𝑠 3 𝑥 𝑑𝑥


Solution: In this case 𝑚 = 3 and 𝑛 = 3. So, we follow either case (a) or case (b).

∫ 𝑠𝑖𝑛3 𝑥 𝑐𝑜𝑠 3 𝑥 𝑑𝑥 = ∫ 𝑠𝑖𝑛2 𝑥 𝑐𝑜𝑠 3 𝑥 sin 𝑥 𝑑𝑥


= ∫(1 − 𝑐𝑜𝑠 2 𝑥)𝑐𝑜𝑠 3 𝑥 sin 𝑥 𝑑𝑥
= ∫ 𝑐𝑜𝑠 3 𝑥 sin 𝑥 𝑑𝑥 − ∫ 𝑐𝑜𝑠 5 𝑥 sin 𝑥 𝑑𝑥
Now, let
𝑢 = cos 𝑥 so that 𝑑𝑢 = −𝑠𝑖𝑛 𝑥 𝑑𝑥.
Thus,

∫ 𝑠𝑖𝑛3 𝑥 𝑐𝑜𝑠 3 𝑥 𝑑𝑥 = ∫ 𝑐𝑜𝑠 3 𝑥 sin 𝑥 𝑑𝑥 − ∫ 𝑐𝑜𝑠 5 𝑥 sin 𝑥 𝑑𝑥


= − ∫ 𝑢3 𝑑𝑢 + ∫ 𝑢5 𝑑𝑢
𝑢4 𝑢6
=− + +𝐶
4 6

𝑐𝑜𝑠4 𝑥 𝑐𝑜𝑠6 𝑥
=− + +𝐶
4 6

221
APPLIED MATHEMATICS I MODULE

Activity 6.6.1
Find the following integrals

i) Integrals of the form ∫ 𝒕𝒂𝒏𝒎 𝒙 𝒔𝒆𝒄𝒏 𝒙 𝒅𝒙

a) If n is even (𝑛 = 2𝑘, 𝑘 ≥ 2), then we factor out 𝑠𝑒𝑐 2 𝑥 and use the identity 𝑠𝑒𝑐 2 𝑥 = 1 +
𝑡𝑎𝑛2 𝑥 to write the remaining part of the integrand in terms of tan 𝑥:
∫ 𝑡𝑎𝑛𝑚 𝑥 𝑠𝑒𝑐 𝑛 𝑥 𝑑𝑥 = ∫ 𝑡𝑎𝑛𝑚 𝑥 𝑠𝑒𝑐 2𝑘 𝑥 𝑑𝑥
= ∫ 𝑡𝑎𝑛𝑚 𝑥 (𝑠𝑒𝑐 2 𝑥)𝑘−1 𝑠𝑒𝑐 2 𝑥𝑑𝑥
= ∫ 𝑡𝑎𝑛𝑚 𝑥 (1 + 𝑡𝑎𝑛2 𝑥 )𝑘−1 𝑠𝑒𝑐 2 𝑥𝑑𝑥

Then, substitute𝑢 = tan 𝑥.

b) If m is odd (𝑚 = 2𝑘 + 1, ), then we factor out sec 𝑥 tan 𝑥and use the identity 𝑠𝑒𝑐 2 𝑥 −
1 = 𝑡𝑎𝑛2 𝑥 to write the remaining part of the integrand in terms of sec 𝑥:
∫ 𝑡𝑎𝑛𝑚 𝑥 𝑠𝑒𝑐 𝑛 𝑥 𝑑𝑥 = ∫ 𝑡𝑎𝑛2𝑘+1 𝑥 𝑠𝑒𝑐 𝑛 𝑥 𝑑𝑥
= ∫(𝑡𝑎𝑛2 𝑥)𝑘 𝑠𝑒𝑐 𝑛−1 𝑥 sec 𝑥 tan 𝑥 𝑑𝑥
= ∫(𝑠𝑒𝑐 2 𝑥 − 1)𝑘 𝑠𝑒𝑐 𝑛−1 𝑥 sec 𝑥 tan 𝑥 𝑑𝑥

Then substitute𝑢 = sec 𝑥.

Example1: Find ∫ 𝑡𝑎𝑛5 𝑥 𝑠𝑒𝑐 4 𝑥 𝑑𝑥

Solution: Since the power of sec 𝑥 is even, we apply (a). So,

∫ 𝑡𝑎𝑛5 𝑥 𝑠𝑒𝑐 4 𝑥 𝑑𝑥 = ∫ 𝑡𝑎𝑛5 𝑥 𝑠𝑒𝑐 2 𝑥 𝑠𝑒𝑐 2 𝑥 𝑑𝑥

= ∫ 𝑡𝑎𝑛5 𝑥 (1 + 𝑡𝑎𝑛2 𝑥)𝑠𝑒𝑐 2 𝑥 𝑑𝑥

222
APPLIED MATHEMATICS I MODULE

Now we make the substitution

𝑢 = tan 𝑥, so that 𝑑𝑢 = 𝑠𝑒𝑐 2 𝑥𝑑𝑥

Therefore,

∫ 𝑡𝑎𝑛5 𝑥 𝑠𝑒𝑐 4 𝑥 𝑑𝑥 = ∫ 𝑡𝑎𝑛5 𝑥 (1 + 𝑡𝑎𝑛2 𝑥)𝑠𝑒𝑐 2 𝑥 𝑑𝑥

= ∫ 𝑢5 (1 + 𝑢2 )𝑑𝑢

= ∫ 𝑢5 𝑑𝑢 + ∫ 𝑢7 𝑑𝑢

1 1
= 6 𝑢6 + 8 𝑢8 + 𝐶

1 1
= 6 𝑡𝑎𝑛6 𝑥 + 8 𝑡𝑎𝑛8 𝑥 + 𝐶

Example2: Find ∫ 𝑡𝑎𝑛5 𝑥 𝑠𝑒𝑐 5 𝑥 𝑑𝑥

Solution: Since the power of tan 𝑥 is odd we apply case (b). First factor out tan 𝑥 sec 𝑥. Thus
the integral becomes;

∫ 𝑡𝑎𝑛5 𝑥 𝑠𝑒𝑐 5 𝑥 𝑑𝑥 = ∫ 𝑡𝑎𝑛4 𝑥 𝑠𝑒𝑐 4 𝑥 𝑡𝑎𝑛 𝑥 sec 𝑥 𝑑𝑥

= ∫(𝑡𝑎𝑛2 𝑥)2 𝑠𝑒𝑐 4 𝑥 𝑡𝑎𝑛 𝑥 sec 𝑥 𝑑𝑥

= ∫(𝑠𝑒𝑐 2 𝑥 − 1)2 𝑠𝑒𝑐 4 𝑥 𝑡𝑎𝑛 𝑥 sec 𝑥 𝑑𝑥

= ∫(𝑠𝑒𝑐 4 𝑥 − 2 𝑠𝑒𝑐 2 𝑥 + 1)𝑠𝑒𝑐 4 𝑥 𝑡𝑎𝑛 𝑥 sec 𝑥 𝑑𝑥

= ∫ 𝑠𝑒𝑐 8 𝑥 sec 𝑥 tan 𝑥 𝑑𝑥 − 2 ∫ 𝑠𝑒𝑐 6 𝑥 sec 𝑥 tan 𝑥 𝑑𝑥 +

+ ∫ 𝑠𝑒𝑐 4 𝑥 𝑡𝑎𝑛 𝑥 sec 𝑥 𝑑𝑥

Let 𝑢 = sec 𝑥, so that 𝑑𝑢 = sec 𝑥 tan 𝑥 𝑑𝑥

Substituting these values we have

∫ 𝑡𝑎𝑛5 𝑥 𝑠𝑒𝑐 5 𝑥 𝑑𝑥 = ∫ 𝑢8 𝑑𝑢 − 2 ∫ 𝑢6 𝑑𝑢 + ∫ 𝑢4 𝑑𝑢

1 2 1
= 9 𝑢9 − 7 𝑢7 + 5 𝑢5 + 𝐶

223
APPLIED MATHEMATICS I MODULE

1 2 1
= 𝑠𝑒𝑐 9 𝑥 − 𝑠𝑒𝑐 7 𝑥 + 𝑠𝑒𝑐 5 𝑥 + 𝐶
9 7 5

For the other cases, that is, if n is odd and m is even, the guidelines are not as clear-cut. We may
need to use the identities, integration by parts, and occasionally a little ingenuity.

Example3: Find ∫ sec 𝑥 𝑑𝑥

sec 𝑥 +tan 𝑥
Solution: Multiplying the numerator and the denominator by sec 𝑥 +tan 𝑥 we have

sec 𝑥 +tan 𝑥
∫ sec 𝑥 𝑑𝑥 = ∫ sec 𝑥 sec 𝑥 +tan 𝑥
𝑑𝑥

𝑠𝑒𝑐 2 𝑥+sec 𝑥 tan 𝑥


=∫ 𝑑𝑥
sec 𝑥+tan 𝑥

If we substitute 𝑢 = sec 𝑥 + tan 𝑥, then 𝑑𝑢 = (sec 𝑥 tan 𝑥 + 𝑠𝑒𝑐 2 𝑥)𝑑𝑥, so the integral
1
becomes∫ (𝑢) 𝑑𝑢 = ln|𝑢| + 𝐶. Thus we have

∫ sec 𝑥 𝑑𝑥 = ln|sec 𝑥 + tan 𝑥| + 𝐶

Example4: Find ∫ 𝑠𝑒𝑐 3 𝑥 𝑑𝑥

Solution: Using integration by parts let

𝑢 = sec 𝑥 and 𝑑𝑣 = 𝑠𝑒𝑐 2 𝑥 𝑑𝑥

Then 𝑑𝑢 = sec 𝑥 tan 𝑥 and 𝑣 = tan 𝑥

Therefore

∫ 𝑠𝑒𝑐 3 𝑥 𝑑𝑥 = ∫ sec 𝑥 𝑠𝑒𝑐 2 𝑥 𝑑𝑥

= sec 𝑥 tan 𝑥 − ∫ tan 𝑥 sec 𝑥 tan 𝑥 𝑑𝑥

= sec 𝑥 𝑡𝑎𝑛 𝑥 − ∫ sec 𝑥 𝑡𝑎𝑛2 𝑥 𝑑𝑥

= sec 𝑥 𝑡𝑎𝑛 𝑥 − ∫ sec 𝑥 (𝑠𝑒𝑐 2 𝑥 − 1 )𝑑𝑥 =


sec 𝑥 𝑡𝑎𝑛 𝑥 − ∫ 𝑠𝑒𝑐 3 𝑥 𝑑𝑥 + ∫ sec 𝑥 𝑑𝑥

Combining the terms involving ∫ 𝑠𝑒𝑐 3 𝑥 𝑑𝑥 we have

2 ∫ 𝑠𝑒𝑐 3 𝑥 𝑑𝑥 = sec 𝑥 𝑡𝑎𝑛 𝑥 + ∫ sec 𝑥 𝑑𝑥

224
APPLIED MATHEMATICS I MODULE

= sec 𝑥 𝑡𝑎𝑛 𝑥 + ln|sec 𝑥 + tan 𝑥| + 𝐾

1
Therefore, ∫ 𝑠𝑒𝑐 3 𝑥 𝑑𝑥 = 2 (sec 𝑥 𝑡𝑎𝑛 𝑥 + ln|sec 𝑥 + tan 𝑥|) + 𝐶

Activity 6.6.2
Find the following integrals

1. ∫ 𝑡𝑎𝑛5 𝑥 𝑠𝑒𝑐 6 𝑥 𝑑𝑥
2. ∫ 𝑡𝑎𝑛3 𝑥 𝑠𝑒𝑐 7 𝑥 𝑑𝑥
3. ∫ 𝑡𝑎𝑛2 𝑥 𝑠𝑒𝑐 6 𝑥 𝑑𝑥

iii) Conversion to Sine and Cosine

Trigonometric integrals that are not of the varieties discussed so far can often be simplified by
expressing the integrands in terms of sines and cosines and then using the methods already
discussed.

Example1: Find ∫ 𝑐𝑜𝑠 2 𝑥 𝑡𝑎𝑛5 𝑥 𝑑𝑥

Solution: We write 𝑡𝑎𝑛5 𝑥 in terms of sines and cosines and combine like terms:

𝑠𝑖𝑛5 𝑥 𝑠𝑖𝑛4 𝑥
∫ 𝑐𝑜𝑠 2 𝑥 𝑡𝑎𝑛5 𝑥 𝑑𝑥 = ∫ 𝑐𝑜𝑠 2 𝑥 𝑐𝑜𝑠5 𝑥
𝑑𝑥 = ∫ 𝑐𝑜𝑠3 𝑥 𝑠𝑖𝑛 𝑥 𝑑𝑥

2
(1−𝑐𝑜𝑠2 𝑥)
=∫ sin 𝑥 𝑑𝑥
𝑐𝑜𝑠3 𝑥

Substituting

𝑢 = cos 𝑥, so that 𝑑𝑢 = − sin 𝑥 𝑑𝑥 gives


2
(1−𝑐𝑜𝑠2 𝑥)
∫ 𝑐𝑜𝑠 2 𝑥 𝑡𝑎𝑛5 𝑥 𝑑𝑥 = ∫ 𝑐𝑜𝑠3 𝑥
sin 𝑥 𝑑𝑥

2 2
(1−𝑢2 ) (1−𝑢2 )
=∫ (−𝑑𝑢) = − ∫ 𝑑𝑢
𝑢3 𝑢3

225
APPLIED MATHEMATICS I MODULE

1−2𝑢2 +𝑢4
= −∫ 𝑑𝑢
𝑢3

1 2
= ∫ (− 𝑢3 + 𝑢 − 𝑢) 𝑑𝑢

1 𝑢2
= 2𝑢2 + 2 ln|𝑢| − +𝐶
2

1 𝑐𝑜𝑠2 𝑥
= 2𝑐𝑜𝑠2 𝑥 + 2 ln|cos 𝑥| − +𝐶
2

Activity 6.6.3
Find the following integrals

1. ∫ 𝑐𝑜𝑠 2 𝑥 𝑡𝑎𝑛5 𝑥 𝑑𝑥
2. ∫ 𝑠𝑖𝑛5 𝑥 𝑐𝑜𝑡 3 𝑥 𝑑𝑥
3. ∫ 𝑡𝑎𝑛2 𝑥 𝑑𝑥
4. ∫ 𝑐𝑜𝑡 5 𝑥 𝑑𝑥

Integrals of the form ∫ 𝒔𝒊𝒏 𝒂𝒙 𝒄𝒐𝒔 𝒃𝒙 𝒅𝒙, ∫ 𝒔𝒊𝒏 𝒂𝒙 𝒔𝒊𝒏 𝒃𝒙 𝒅𝒙, 𝒂𝒏𝒅 ∫ 𝒄𝒐𝒔 𝒂𝒙 𝒄𝒐𝒔 𝒃𝒙 𝒅𝒙

To evaluate such integrals use the corresponding identities:

1
(a) sin 𝐴 cos 𝐵 = 2 [sin(𝐴 − 𝐵) + sin(𝐴 + 𝐵)]
1
(b) sin 𝐴 sin 𝐵 = 2 [cos(𝐴 − 𝐵) − cos(𝐴 + 𝐵)]
1
(c) cos 𝐴 cos 𝐵 = 2 [cos(𝐴 − 𝐵) + cos(𝐴 + 𝐵)]

Example 1: Find ∫ sin 3𝑥 cos 2𝑥 𝑑𝑥

Solution: Using identity (a) we have

1
∫ sin 3𝑥 cos 2𝑥 𝑑𝑥 = ∫ 2 [sin(3 − 2)𝑥 + sin(3 + 2)𝑥] 𝑑𝑥

1
= 2 ∫[sin 𝑥 + sin 5𝑥] 𝑑𝑥

226
APPLIED MATHEMATICS I MODULE

1 1
= ∫ sin 𝑥 𝑑𝑥 + ∫ sin 5𝑥 𝑑𝑥
2 2

1 1
= − 2 cos 𝑥 − 10 cos 5𝑥 + 𝐶

Example2: Find ∫ sin 5𝑥 sin 3𝑥 𝑑𝑥

Solution: Using identity (b) we get

1
∫ sin 5𝑥 sin 3𝑥 𝑑𝑥 = ∫ 2 [cos(5 − 3) 𝑥 − cos(5 + 3)𝑥]𝑑𝑥

1
= 2 ∫[cos 2 𝑥 − cos 8𝑥]𝑑𝑥

1 1
= 2 ∫ cos 2𝑥 𝑑𝑥 − 2 ∫ cos 8𝑥 𝑑𝑥

1 1
= 4 sin 2𝑥 − 16 sin 8𝑥 + 𝐶

Example3: Find ∫ cos 3𝑥 cos 4𝑥 𝑑𝑥

Solution: Here we use identity (c)

1
∫ cos 3𝑥 cos 4𝑥 𝑑𝑥 = ∫ 2 [cos(3 − 4)𝑥 + cos(3 + 4)𝑥]𝑑𝑥

1
= 2 ∫[cos(−𝑥) + cos 7𝑥]𝑑𝑥

1 1
= 2 ∫ cos 𝑥 𝑑𝑥 + 2 ∫ cos 7𝑥 𝑑𝑥

1 1
= 2 sin 𝑥 + 17 sin 7𝑥 + 𝐶

Activity 6.6.4
Find the following integrals

227
APPLIED MATHEMATICS I MODULE

6.7 Integration by Trigonometric Substitutions

In finding the area of a circle or an ellipse, an integral of the form ∫ √𝑎2 − 𝑥 2 𝑑𝑥 arises, where𝑎 >
0. This integral, as it stands, is difficult to integrate. In certain cases integrals of the form

∫ √𝑥 2 − 𝑎2 𝑑𝑥, 𝑎𝑛𝑑 ∫ √𝑎2 + 𝑥 2 𝑑𝑥 also occur. To perform such integrals we follow the following
strategies.

i) Integrals Containing ∫ √𝒂𝟐 − 𝒙𝟐 𝒅𝒙

𝜋 𝜋
If we let 𝑥 = 𝑎 sin 𝑢, with 𝑎 > 0 and − 2 ≤ 𝑢 ≤ 2 , then acos 𝑢 ≥ 0, so that

√𝑎2 − 𝑥 2 = √𝑎2 − 𝑎2 𝑠𝑖𝑛2 𝑢 = √𝑎2 (1 − 𝑠𝑖𝑛2 𝑢) = √𝑎2 𝑐𝑜𝑠 2 𝑢 = acos 𝑢

Thus if an integral contains√𝑎2 − 𝑥 2 , we can eliminate the square root by substituting

𝑥 = 𝑎 sin 𝑢, so that 𝑑𝑥 = 𝑎 cos 𝑢 𝑑𝑢

1
Example1: Find ∫ 𝑑𝑥
𝑥 2 √9−𝑥 2

Solution: Since √9 − 𝑥 2 = √32 − 𝑥 2 , we substitute

𝑥 = 3 sin 𝑢, so that 𝑑𝑥 = 3 cos 𝑢 𝑑𝑢

Thus,

1 3 cos 𝑢 𝑑𝑢
∫ 𝑥 2 √9−𝑥 2 𝑑𝑥 = ∫ 9𝑠𝑖𝑛2 𝑢√9−9𝑠𝑖𝑛2

3 cos 𝑢 𝑑𝑢
=∫
(9𝑠𝑖𝑛2 𝑢)3√1−𝑠𝑖𝑛2

1 cos 𝑢 𝑑𝑢
= 9∫
(𝑠𝑖𝑛2 𝑢)√1−𝑠𝑖𝑛2

1 cos 𝑢 𝑑𝑢
= 9 ∫ (𝑠𝑖𝑛2 𝑢) cos 𝑢

1 𝑑𝑢 1
= 9 ∫ 𝑠𝑖𝑛2 𝑢 = 9 ∫ 𝑐𝑠𝑐 2 𝑢 𝑑𝑢

1
= − 9 cot 𝑢 + 𝐶

228
APPLIED MATHEMATICS I MODULE

In order to write the answer in terms of the original variable x, we draw the triangle in figure 4.14.
From this we see that

√16−𝑥 2
cot 𝑢 = 𝑥

1 1 √16−𝑥 2
Therefore, ∫ 2 √9−𝑥 2
𝑑𝑥 = − 16 +𝐶
𝑥 𝑥

𝑥 3

√9 − 𝑥 2

Figure 4
1
Example2: Evaluate the integral ∫02 √1 − 4𝑥 2 𝑑𝑥

Solution: Since √1 − 4𝑥 2 = √12 − (2𝑥)2 , we are led to substitute

cos 𝑢
2𝑥 = sin 𝑢, so that 𝑑𝑥 = 𝑑𝑢
2

For the limits of integration we notice that

1 𝜋
if 𝑥 = 0, then 𝑢 = 0, and if 𝑥 = 2, then 𝑢 = 2

Therefore,
1 𝜋
cos 𝑢
∫0 √1 −
2 4𝑥 2 𝑑𝑥 = ∫0 √1 − 𝑠𝑖𝑛2 𝑢 (
2 ) 𝑑𝑢
2

𝜋
cos 𝑢
= ∫02 cos 𝑢 ( ) 𝑑𝑢
2

𝜋
1
= 2 ∫02 𝑐𝑜𝑠 2 𝑢 𝑑𝑢

1 1 𝜋 1 𝜋 1 1 1
= 2 (4 sin 2 (2 ) + 2 ∙ 2 ) − 2 (4 sin 2(0) + 2 ∙ 0)

𝜋
= 8

229
APPLIED MATHEMATICS I MODULE

ii) Integrals Containing √𝒙𝟐 + 𝒂𝟐

𝜋 𝜋
If we let 𝑥 = a tan 𝑢, with 𝑎 > 0, and − 2 < 𝑢 < 2 , then 𝑎 sec 𝑢 ≥ 0, so that

√𝑥 2 + 𝑎2 = √𝑎2 𝑡𝑎𝑛2 𝑢 + 𝑎2 = √𝑎2 (𝑡𝑎𝑛2 𝑢 + 1) = √𝑎2 𝑠𝑒𝑐 2 𝑢 = 𝑎 sec 𝑢.

Thus if an integral contains √𝑎2 + 𝑥 2 , we can eliminate the square root by substituting
𝑥 = 𝑎 tan 𝑢, so that 𝑑𝑥 = 𝑎 𝑠𝑒𝑐 2 𝑢 𝑑𝑢
1
Example1: Find ∫ √𝑥 2 𝑑𝑥
+1

Solution: Since√𝑥 2 + 1 = √𝑥 2 + 12 , we are led to substitute


𝑥 = tan 𝑢, so that 𝑑𝑥 = 𝑠𝑒𝑐 2 𝑢 𝑑𝑢

Thus, √𝑥 2 + 1
1 𝑠𝑒𝑐 2 𝑢 𝑑𝑢
∫ √𝑥 2 +1 𝑑𝑥 = ∫ √𝑡𝑎𝑛2 𝑢+1 𝑥

= ∫ sec 𝑢 𝑑𝑢 1
= ln|sec 𝑢 + tan 𝑢| + 𝐶 Figure 5

= ln|√𝑥 2 + 1 + 𝑥| + 𝐶
𝑥
Example2: Find ∫ √𝑥 2 𝑑𝑥
+9

Solution: Since√𝑥 2 + 9 = √𝑥 2 + 32 , we are led to substitute


𝑥 = 3 tan 𝑢, so that 𝑑𝑥 = 3 𝑠𝑒𝑐 2 𝑢 𝑑𝑢

Thus,
𝑥 9 tan 𝑢 𝑠𝑒𝑐 2 𝑢 𝑑𝑢
∫ √𝑥 2 +9 𝑑𝑥 = ∫ √9𝑡𝑎𝑛2 𝑢+9
𝑥 √𝑥 2 + 9

tan 𝑢 𝑠𝑒𝑐 2 𝑢 𝑑𝑢
= 3∫ 3
sec 𝑢

= 3 ∫ tan 𝑢 sec 𝑢 𝑑𝑢 Figure 6

= 3 sec 𝑢 + 𝐶
From Figure 4 we have

230
APPLIED MATHEMATICS I MODULE

√𝑥 2 +9
sec 𝑢 = 3

Therefore,
𝑥
∫ √𝑥 2 +9 𝑑𝑥 = 3 sec 𝑢 + 𝐶

= √𝑥 2 + 9 + 𝐶

iii) Integrals Containing √𝒙𝟐 − 𝒂𝟐


𝜋 3𝜋
If we let 𝑥 = 𝑎 sec 𝑢 with 0 ≤ 𝑢 < or 𝜋 ≤ 𝑢 < , then 𝑎 tan 𝑢 ≥ 0, so that
2 2

√𝑥 2 − 𝑎2 = √𝑎2 𝑠𝑒𝑐 2 𝑢 − 𝑎2 = √𝑎2 (𝑠𝑒𝑐 2 𝑢 − 1) = √𝑎2 𝑡𝑎𝑛2 𝑢 = 𝑎 tan 𝑢.

Thus, if an integral contains√𝑥 2 − 𝑎2 , then we can eliminate the square root by substituting
𝑥 = 𝑎 sec 𝑢, so that 𝑑𝑥 = 𝑎 sec 𝑢 tan 𝑢 𝑑𝑢

Example: Find ∫ √𝑥 2 − 9 𝑑𝑥

Solution: Since √𝑥 2 − 9 = √𝑥 2 − 32 , we let


𝑥 = 3 sec 𝑢 then 𝑑𝑥 = 3 sec 𝑢 tan 𝑢 𝑑𝑢
Thus,

∫ √𝑥 2 − 9 𝑑𝑥 = ∫ √9𝑠𝑒𝑐 2 𝑢 − 9 (3 sec 𝑢 tan 𝑢)𝑑𝑢

= 9 ∫ √𝑠𝑒𝑐 2 𝑢 − 1 sec 𝑢 tan 𝑢 𝑑𝑢


= ∫ 𝑡𝑎𝑛2 𝑢 sec 𝑢 𝑑𝑢
= ∫(𝑠𝑒𝑐 2 𝑢 − 1) sec 𝑢 𝑑𝑢
= ∫ 𝑠𝑒𝑐 3 𝑢 𝑑𝑢 − ∫ sec 𝑢 𝑑𝑢

1
= 2 (sec 𝑢 𝑡𝑎𝑛 𝑢 + ln|sec 𝑢 + tan 𝑢|) − ln|sec 𝑢 + tan 𝑢| + 𝐶

1 1
= 2 (sec 𝑢 𝑡𝑎𝑛 𝑢) − 2 (ln|sec 𝑢 + tan 𝑢|) + 𝐶

√𝑥 2 − 9 𝑥

231
APPLIED MATHEMATICS I MODULE

Figure 7 3

From Figure 5. we see that


𝑥 √𝑥 2 −9
sec 𝑢 = 3 and tan 𝑢 = 3

Therefore,
1 𝑥√𝑥 2 −9 𝑥 √𝑥 2 −9
∫ √𝑥 2 − 9 𝑑𝑥 = 2 ( 9
− ln |3 + 3
|) + 𝐶

Table 6.2summarizes the trigonometric substitutions that we have discussed so far.

Expression in integrand Substitution


−𝜋 𝜋
√𝑎2 − 𝑥 2 𝑥 = 𝑎 sin 𝑢, 𝑤𝑖𝑡ℎ ≤𝑢≤
2 2

𝑑𝑥 = 𝑎 cos 𝑢 𝑑𝑢
−𝜋 𝜋
√𝑎2 + 𝑥 2 𝑥 = 𝑎 tan 𝑢, 𝑤𝑖𝑡ℎ 2
<𝑢< 2

𝑑𝑥 = 𝑎 sec 2 𝑢 𝑑𝑢
𝜋 3𝜋
√𝑥 2 − 𝑎2 𝑥 = 𝑎 sec 𝑢, 𝑤𝑖𝑡ℎ 0 ≤ 𝑢 < or 𝜋 ≤ 𝑢 <
2 2

𝑑𝑥 = 𝑎 sec 𝑢 tan 𝑢 𝑑𝑢

iv) Integrals Containing √𝒃𝒙𝟐 + 𝒄𝒙 + 𝒅

By completing the square in 𝑏𝑥 2 + 𝑐𝑥 + 𝑑, we can express √𝑏𝑥 2 + 𝑐𝑥 + 𝑑 in terms of


√𝑎2 − 𝑥 2 , √𝑎2 + 𝑥 2 , 𝑜𝑟√𝑥 2 − 𝑎2 for suitable 𝑎 > 0. Then the trigonometric substitution
eliminates the square root as before.
𝑥 𝑑𝑥
Example1: Find ∫ √3−2𝑥−𝑥 2

Solution: We can transform the integrand in to a form for which trigonometric substitutions
are appropriate. In this case we first complete the square under the square root.
3 − 2𝑥 − 𝑥 2 = 3 − (𝑥 2 + 2𝑥) = 3 + 1 − (𝑥 2 + 2𝑥 + 1)
= 4 − (𝑥 + 1)2
This suggests that we make the substitution𝑢 = 𝑥 + 1. Then 𝑑𝑢 = 𝑑𝑥 and 𝑥 = 𝑢 − 1, so
𝑥 𝑢−1
∫ √3−2𝑥−𝑥 2 𝑑𝑥 = ∫ √4−𝑢2 𝑑𝑢

232
APPLIED MATHEMATICS I MODULE

Now let 𝑢 = 2 sin 𝜃 which gives 𝑑𝑢 = 2 cos 𝜃 𝑑𝜃 and √4 − 𝑢2 = 2 cos 𝜃, and so


𝑥 2 sin 𝜃−1
∫ √3−2𝑥−𝑥 2 𝑑𝑥 = ∫ 2 cos 𝜃
2 cos 𝜃 𝑑𝜃

= ∫(2 sin 𝜃 − 1)𝑑𝜃


= −2 cos 𝜃 − 𝜃 + 𝐶
We can devise the relation between u and 𝜃 from Figure 6.

𝑢 2

√4 − 𝑢2

Figure 6
√4−𝑢2 𝑢
From this, we have cos 𝜃 = and 𝜃 = 𝑠𝑖𝑛−1 ( 2 ) and substituting these values we get
2
𝑥
∫ √3−2𝑥−𝑥 2 𝑑𝑥 = −2 cos 𝜃 − 𝜃 + 𝐶

√4−𝑢2 𝑢
= −2 − 𝑠𝑖𝑛−1 (2) + 𝐶
2

𝑥+1
= −√4 − (𝑥 + 1)2 − 𝑠𝑖𝑛−1 ( )+𝐶
2

Activity 6.7
Find the following integrals

233
APPLIED MATHEMATICS I MODULE

6.8 Improper Integrals

b
The definite integral  f ( x)dx has meaning only when f is continuous on a, b consequently
a

bounded on a, b .

Definition 1: We say f is bounded on an interval I if there is a constant M such that f ( x)  M

for all x in I .
In this section, we shall extend the definition of the definite integral when either the integrand or
the inerval of integraion is unbounded. Such integrals are called improper integrals.

6.8.1 Integrals Over Unbounded(infinite) Intervals



Definition2: If f is continuous on a,   , then the improper integral  f ( x)dx converges if
a

t
lim  f ( x)dx exists. In that case
t 
a

 t

 f ( x)dx  lim  f ( x)dx


t 
1
a a

If the limit does not exist, the improper integral diverges.


Again if f is continuous on  , a , then
a a

 f ( x)dx  lim  f ( x)dx


t 
2
 t

Provided the limit exists.

Example 1: Determine whether the integral converges or diverges, and if it converges find its
value.

1
a.   x  1
0
2
dx

0
1
b.  1  xdx


234
APPLIED MATHEMATICS I MODULE

Solution: (a) Following the discussion above and equation 1 we have
 t
 1 
t
1 1
  x  1
0
2
dx  lim
t  0  x  1
2
dx  lim  
t   x  1  0

 1 1 
 lim    0 1  1.
t   t  1 0  1
Thus, the improper integral converges and has the value 1 .
(b) Following the discussion above and equation 2 we have
0 0
1 1
 x  1dx  lim  dx  lim   ln 1  x  t
0

t  t x  1 t 

 lim  ln 1  0   ln 1  t   0     .


t 

Thus, since the limit does not exist, the improper integral diverges.

 xe dx .
x
Example 2: Evaluate


Solution: we have
0 0

 xe dx  lim  xe dx
x x

 t  

We integrate by part with u  x, dv  e x dx so that du  dx, v  e x .


0 0

 xe dx  xe    e dx  te 1  e
x x x 0
t t
t
t t

We know that et  0 as t   and by L’Hopital’s rule we have


t 1
lim  te  lim e   lim
t

t  t 
t
t  et

lim  e   0
t
=
t 

 xe dx  lim  te  1  et   0 1  0  1
x t
Therefore
 t 

For integrals over the range  ,  , we write

235
APPLIED MATHEMATICS I MODULE

 a 

 f ( x)dx   f ( x)dx   f ( x)dx 3


  a

Provided both of the improper integrals on the right converge.



If either of the integrals on the right in 3 diverges, then  f ( x)dx is said to diverge. It can be


shown that 3 does not depend on the choice of the real number a .

1
Example3: Determine whether the integral  dx converges or diverges, and if it converges
1 x
2


find its value.


Solution: Using 3 , with a  0 , we have
 0 
dx .
1 1 1
 dx   dx  
1 x 1 x 1 x
2 2 2
  0

Next, applying 2 we get

arctan x  lim arctan x


 0 t
1 1 1
  dx  lim 
0 t
dx  lim dx  lim
1 x 1 x 1 x
2 t  2 t  2 t  t t  0
 t 0

 lim arctan 0  arctan t   limarctan t  arctan o


t  t 

        
 0         0     .
  2   2  2 2
Thus,the given improper integral converges and has the value .
Activity 6.8.1
Evaluate the following improper integral

236
APPLIED MATHEMATICS I MODULE

6.8.2 Integrals with Unbounded Integrands

We now consider a function f that is continuous at every point in a, b and unbounded near a .
By assumption f is continuous on the interval t, b for t in a, b  , so that
b

 f ( x)dx is defined for such t . If the one-sided limit


t
b
lim  f ( x)dx
ta t
b
exists, then we define  f ( x)dx to be the limit. This idea leads us the following definitions:
a

Definition1:
a. If f is continuous on a, b  and is discontinuous at b , then
b t

 f ( x)dx  limb  f ( x)dx



4
t
a a

Provided the limit exists.


ii. If f is continuous on a, b and is discontinuous at a , then
b a

 f ( x)dx  lima  f ( x)dx



5
t
a t

Provided the limit exists.


As the preceding section, the integrals defined in 4 and 5 are reffered to as improper integrals
and they converge if the limit exists. The limits are called the values of the improper integrals. If
the limits does not exist, the improper integral diverge.
5
1
Example1: Evaluate  dx .
2 x2
Solution: Since the integrand has an infinite discontinuity at x  2 , we apply 4 and have
5 5
1 1

2 x2
dx  lim 
t  2 t x2
dx


5
 lim 2 x  2 
t
t  2

 lim2
t  2
 3 t 2 
2 3
Thus, the given improper integral converges and has the value 2 3 .

237
APPLIED MATHEMATICS I MODULE

1
1
Example2: Determine whether the improper integral  x dx
0
converges or diverges.

Solution: The integrand is unbounded near 0 . Applying 5 gives us

ln x  lim0 ln1  ln t    .


1 1
1 1
0 x dx  tlim
1
dx  lim 
0
t
x t 0 t t

Consequently the given improper integral diverges, since the limit does not exist.
We give the definition of another improper integral as follows:

Definition2: If f has a discontinuity at a number c in the open interval a, b  but continuous
elsewhere on a, b , then
b c b

 f ( x)dx   f ( x)dx   f ( x)dx


a a c
6
Provided both of the improper integrals on the right converge. If both converge, then the value of
b
the improper integral  f ( x)dx is the sum of the two values.
a
4
1

x 3
Example3: Determine whether the improper integral 2
dx converges or diverges.
0

Solution: The integrand is undefined at x  3 . Since this number is in the interval 0,4 , we use
6 , with c  3 :

4 3 4
1 1 1
 dx   dx   dx .
0 x 32
0 x 3 2
3 x 3 2

For the integral on the left to converge, both integrals on the right must converge.
However, since
t
3
1
t
dx  lim 
 1 
1
 dx  lim 
0 x 3 2
3 x  3t

3 
0
 x 3 
2
t

 1 1
 lim     .
t 3  t  3 3
Thus, the given improper integral diverges.

238
APPLIED MATHEMATICS I MODULE

The other kind of improper integral is found if f is continuous in a, b  and is unbounded near
b
both a and b . We say that  f ( x)dx converges if for some point c in a, b  both the integrals
a
c b


a
f ( x)dx and  f ( x)dx converge. Otherwise we say that the integral is divergent.
c

1 2x
1
Example4: Determine whether  dx converges or diverges.
x x
2
0

Solution: The integrand is unbounded near both the end points 0 and 1 and is coninuous on 0,1
3
. Consequently the integral is of the type under consideretion. If we let c  , then we need to
4
analyze the convergence of
3
1 2x 1 2x
4 1

 dx and  dx
x x x x
2 2
0 3
4

3
For 0  t  , we have
4

2 
3 3
1  2x 1  2x
4 4 3

 
4
dx  lim dx  lim
x x
2
x x x x
2 t 0 2 t 0 t
0 t

  3 2  3
 lim 2  t t   .
t 0 
  16  2
A similar computation shows the second improper integral also converges and that
1  2x
1
3
 dx   .
x x 2
2
3
4

Therefore the original integral converges, and


3
1  2x 1  2x 1  2x 3  3
1 4 1

 dx   dx   dx    
2  2 
  0.
x x x x x x
2 2 2
0 0 3
4

239
APPLIED MATHEMATICS I MODULE

Activity 6.8.2
1. Determine whether the following improper integral converges or diverges, and if it
converges find its value.

Self-Test Exercises
1. Let f be such that f ′ (x) = sin x and f(π) = −5. Then determine the function f.

2. Find the ant derivative F(x) of f(x) = ex which satisfies F(0) = 5.

3. Find the lower and upper sums for the indicated partitions.
−1 π π
i. f(x) = x
; ℘ = {−4, −3, −2, −1} ii. f(x) = sin x ; ℘ = {0, 4 , 2

4. Evaluate each of the integrals.

6. Evaluatethe following improper integral .

1. 2.

3. 4.

240
APPLIED MATHEMATICS I MODULE

CHAPTER SEVEN

APPLICATION OF INTEGRATION
Introduction
In this chapter we look at some applications of integrals in finding the area of a region bounded
by a curve and the volume a solid region.

Objectives:
At the end of this unit each student should able to:
 Apply integration to find area

 Apply integration to find volume

In this chapter, we will see some applications of integration such as: Area of a region bounded by
curves of continuous defined on a closed interval [a, b] and volume of a solid of revolutions.
7.1 Area
Let f be a non-negative continuous function on [a, b], then the area of the region R bounded by
f and the x-axis between x =a & x = b (as shown in the fig. below) is given by:
𝑏
𝐴(𝑅) = ∫𝑎 𝑓(𝑥)𝑑𝑥 (∗)

Example1:
1) Find the area of the region bounded by the graph of the function 𝑓(𝑥) = 𝑥 2 − 3𝑥 + 2 and
the x-axis between x=0 & x=3.
Solution: The region R is the shaded region as shown below.

241
APPLIED MATHEMATICS I MODULE

Now let R1, R2 and R3 be the sub-regions between x = 0 and x = 1, x = 1 and x = 2, and
x = 2 and x = 3, respectively. Then, using (∗):
𝐴(𝑅) = 𝐴(𝑅1 ) + 𝐴(𝑅2 ) + 𝐴(𝑅3 )
Where,
1 1 𝑥3 1 5
𝑥2
𝐴(𝑅1 ) = ∫0 𝑓(𝑥)𝑑𝑥 = ∫0 (𝑥 2 − 3𝑥 + 2)𝑑𝑥 = ( 3 − 3 + 2𝑥) | = 6
0
2

2 𝑥 3 𝑥 2
2 1
𝐴(𝑅2 ) = − ∫1 (𝑥 2 − 3𝑥 + 2)𝑑𝑥 = −( 3 − 3 2 + 2𝑥) | = 6
1
Since the sub-region R2 is below the x- axis, the negative sign is important. Similarly,
3 𝑥3 𝑥2 3 5
𝐴(𝑅3 ) = ∫2 (𝑥 2 − 3𝑥 + 2)𝑑𝑥 = ( 3 − 3 + 2𝑥) | = 6.
2 2
Thus, area of the region is:
5 1 5 11
𝐴(𝑅) = + + = .
6 6 6 6

We can see that the area of a region bounded by a continuous curves y = f(x), y = g(x) and the
lines x = a, x = b such that 𝑓(𝑥) ≥ 𝑔(𝑥) for all x in [a, b] is:
𝑏
𝐴(𝑅) = ∫𝑎 [𝑓(𝑥) − 𝑔(𝑥)]𝑑𝑥 ……………………(∗∗)

Example 2: Find area of the region enclosed by the parabolas 𝑦 = 𝑥 2 and 𝑦 = 2𝑥 − 𝑥 2 .


Solution: The sketch of the region is shown below. So, using (**):

242
APPLIED MATHEMATICS I MODULE

1 1 1
𝐴(𝑅) = ∫ [𝑓(𝑥) − 𝑔(𝑥)]𝑑𝑥 = ∫ [(2𝑥 − 𝑥 2 ) − 𝑥 2 ]𝑑𝑥 = ∫ (2𝑥 − 2𝑥 2 )𝑑𝑥
0 0 0
𝑥3 1 1
= (𝑥 2 − 2 3 ) | = 3.
0
Note that the intersection points can be obtained as: 𝑦 = 2𝑥 − 𝑥 2 = 𝑥 2 ⇒ 2𝑥 2 − 2𝑥 = 0.
Solving this equation for x, we get 𝑥 = 0 𝑜𝑟 𝑥 = 1, then 𝑦 = 0 𝑜𝑟 𝑦 = 1, as indicated in the
graph above.
Example 3: Find the area of the region bounded by the curves y = sin x and
𝜋
y = cos x, between x = 0 & x = 2 .

Solution: The region R with its parts is as shown below:

𝜋 𝜋
The points of intersection occur when sin x = cos x, that is, when 𝑥 = 4 (since 0 ≤ 𝑥 ≤ 2 ).
𝜋 𝜋 𝜋
Observe that cos x ≥ sin x when 0 ≤ 𝑥 ≤ but cos x ≤ sin x when 4 ≤ 𝑥 ≤ 2 . Therefore, area of
4

the region is:


𝜋 𝜋
4 2
𝐴(𝑅) = 𝐴1 + 𝐴2 = ∫ [𝑐𝑜𝑠𝑥 − 𝑠𝑖𝑛𝑥]𝑑𝑥 + ∫ [𝑠𝑖𝑛𝑥 − 𝑐𝑜𝑠𝑥]𝑑𝑥
𝜋
0
4

243
APPLIED MATHEMATICS I MODULE

𝜋
𝜋
= [𝑠𝑖𝑛𝑥 + 𝑐𝑜𝑠𝑥] | 4 + [−𝑐𝑜𝑠𝑥 − 𝑠𝑖𝑛𝑥] |𝜋2 = 2√2.
0 4

Activity7.1
1. Find the area of the region enclosed by the graph of f(x) = sin x and the x-axis between 𝑥 =
𝜋
− 2 and 𝑥 = 2𝜋.

2. Find the area of the region in the first quadrant which is enclosed by the y-axis and the
curves of f(x) = cos x and g(x) = sinx.
3. Determine the area of the region enclosed by the graphs of 𝑥 = −𝑦 2 and 𝑥 = 9 − 2𝑦 2
4. Find the area of the region enclosed by the graphs 𝑦 = 𝑥 2 + 1 and the line𝑦 = 5.

7.2 Volume
In section, we will apply integral calculus to determine the volume of a solid region by considering
cross sections.
Suppose a region rotates about a straight line, then a solid figure called a solid of revolution is
formed. The volume of such a solid is said to be a volume of revolution and the line about which
the region rotates is an axis of symmetry.
Now consider the solid of revolution generated by revolving the region between the curve 𝑦 =
𝑓(𝑥) and the x-axis from 𝑥 = 𝑎 to 𝑥 = 𝑏 as shown below.

Every cross section which is perpendicular to the x-axis at x a circular region with radius, r = f(x).
Thus, the area of the cross section A(x) is 𝐴(𝑥) = 𝜋𝑟 2 = 𝜋(𝑓(𝑥))2 .
Thus, we define the volume of revolution V as by:
𝑏 𝑏
𝑉 = ∫𝑎 𝐴(𝑥) 𝑑𝑥 = ∫𝑎 𝜋(𝑓(𝑥))2 𝑑𝑥 (∗)

244
APPLIED MATHEMATICS I MODULE

Example 4: Find the volume of the solid obtained by rotating about the x-axis the region under the
curve 𝑦 = √𝑥 from 0 to 1.
Solution: The region and the solid figure rotated about the x-axis from 0 to 1 are shown in fig.(a)
and fig.(b) below.

Now, the area of the cross section is 𝐴(𝑥) = 𝜋(𝑓(𝑥))2 = 𝜋(√𝑥)2 = 𝜋𝑥. So, using(∗) , the volume
of revolution V is given by:
1 1 𝑥2 1 𝜋
𝑉 = ∫0 𝐴(𝑥) 𝑑𝑥 = ∫0 𝜋𝑥 𝑑𝑥 = 𝜋 | = 2.
2 0
Note that if f and g are continuous, non-negative on [a, b] such that 𝑓(𝑥) ≥ 𝑔(𝑥) for all x in [a, b].
Let R be a region bounded by f(x) and g(x), on [a, b] as in the following figure. Then, the volume
of the solid of revolution generated by revolving the region R about the x-axis is given by:
𝑏 2 2
𝑉 = ∫𝑎 𝐴(𝑥) 𝑑𝑥, where 𝐴(𝑥) = 𝜋[(𝑓(𝑥)) − (𝑔(𝑥)) ].

Example 5: The region R enclosed by the curves 𝑦 = 𝑥 and 𝑦 = 𝑥 2 is rotated about the x-axis.
Find the volume of the resulting solid.
Solution:

245
APPLIED MATHEMATICS I MODULE

The curves y = x and y = x2 intersect at the points (0,0) and (1, 1). The region and solid of rotation
are shown in the fig. above. The cross-section in the plane has the shape of annular ring with inner
radius x2 and outer radius x. So, 𝐴(𝑥) = 𝜋[𝑥 2 − (𝑥 2 )2 ] = 𝜋(𝑥 2 − 𝑥 4 ). Therefore, we have
1 1
𝑥 3 𝑥 5 1 2𝜋
𝑉 = ∫ 𝐴(𝑥) 𝑑𝑥 = ∫ 𝜋(𝑥 2 − 𝑥 4 ) 𝑑𝑥 = 𝜋[ − ]| =
0 0 3 5 0 15
Consider again the case where a region R is rotated about the y-axis: where, either R is a region
bounded by the curve x = u(y) and the y-axis, between the lines y = c & y = d or R is a region
between two curves v(y) and w(y) on [c, d] as in following figures:

Then the corresponding volume of revolution is given by:


𝑑 𝑑
𝑉 = ∫𝑐 𝜋[𝑢(𝑦)]2 𝑑𝑦 and 𝑉 = ∫𝑐 𝜋([𝑤(𝑦)]2 − [𝑣(𝑦)]2 ) 𝑑𝑦 respectively.
Example 6: Find the volume of the solid obtained by rotating the region bounded by y = x3, y = 8,
and x = 0 about the y-axis.
Solution: The rotation of the region is shown as:

2
Here, u(y) = x = 3√𝑦 . So, the area of the cross-section is 𝐴(𝑦) = 𝜋[𝑢(𝑦)]2 = 𝜋𝑦 3 . Thus, the
volume V of revolution is:
2
𝑑 8 96𝜋
𝑉 = ∫𝑐 𝜋[𝑢(𝑦)]2 𝑑𝑦 = ∫0 𝜋𝑦 3 𝑑𝑦 = .
5

246
APPLIED MATHEMATICS I MODULE

Activity7.2
1. Find the volume of solid of revolution about the x-axis generated by revolving the area
between the lines y = x and y = 4 from x = 1 to x = 3.
2. Find the volume of the solid of revolution about the y-axis generated by revolving the region
enclosed by the curve x = √𝑦 and the y-axis from y = 0 to y = 4.

Self-Test Exercises
1 Find the area A of the region between the graph of f and the x- axis on the given interval.
a. 𝑓(𝑥) = 𝑥, [0, 4]
b. 𝑓(𝑥) = 𝑥 2 + 2𝑥, [−1, 3]
c. 𝑓(𝑥) = 𝑥 3 − 𝑥, [−2, 1]
𝜋
d. 𝑓(𝑥) = 𝑐𝑜𝑠 𝑥 − 𝑠𝑖𝑛 𝑥 , [0, 3 ]
𝜋
e. 𝑓(𝑥) = 𝑐𝑜𝑠 𝑥 − 𝑠𝑖𝑛 𝑥 , [0, ]
3

f. 𝑓(𝑥) = 𝑥√1 − 𝑥 2 , [−1, 1]

2. Find the area A of the region between the graphs of the functions on the given interval.

a. 𝑓(𝑥) = 𝑠𝑖𝑛 𝑥, 𝑔(𝑥) = 𝑐𝑜𝑠 𝑥 ; [0,2𝜋]


b. 𝑓(𝑥) = 𝑥 2 , 𝑔(𝑥) = 𝑥 3 ; [−2, 1]
c. 𝑓(𝑥) = 𝑥 2 + 4𝑥, 𝑘(𝑥) = 𝑥 − 2; [−3, 0]
3. Find the area A of the region between the graphs of y = x 2 − x − 4 and y = x − 1.
4. The area bounded by the graph of y = x2+1 and the line y = 4 rotates about the y – axis,
find the volume of the solid generated.
5. Find the volume of the solid obtained by rotating the region bounded by the curves y2 =
x and x = 2y about y-axis.

247
APPLIED MATHEMATICS I MODULE

Reference

1. 1. James Stewart, Calculus early trancedental, 6th ed., Prentice Hall, 2008
2. 2. Howard Anton,Calculus a new horizon, 6th ed., John wiley and Sons Inc
3. 3. Robert Ellis and Denny Gulick, Calculus with analytic geometry, 6th ed, Harcourt Brace Jovanovich,
Publishers, 5th ed, 1993
4. 4. Adams, Calculus: A complete course, 5th ed, Addison Wesley, 2003
5. 5. R. Wrede and M. R. Spiegel, Theory of advanced calculus, 2nd ed., McGraw-Hill, 2002.
6. 6. A. E. Taylor and W. R. Mann, Advanced calculus, 3rd ed, John-Wiley and Son, INC, 1995.
7. 7. R. T. Smith and R. B. Minton, Calculus concepts and connections, McGram-Hill book company, 2006
8. 8. D. V. Widder, Advanced calculus, Prentice-Hall, 1979
9. 9. Ross L. Finney et al, Calculus, Addison Wesley, 1995
10. 10. E. J. Purcell and D. Varberg, Calculus with analytic geometry, Prentice-Hall INC., 1987
11. 11. Lynne, Garner: Calculus and Analytic Geometry, Dellen Publishing Company12.
12. 12. John A. Tierney: Calculus and Analytic, 4th edition, Allyn and Bacon, Inc. –Boston.
13. Earl W. Swokowski: Calculus with Analytic Geometry, 2nd edition, Prindle, Weber and Schmidt.
14. Goldstein, L. J., Lay D. C. , and Schneider D. I (1987). Calculus and its applications, 4th, edn,
Prentice-Hall, Inc, London
13. 15. Kresying, Advanced Engineering Mathematics.
14. 16. Alan Jeffry , Advanced Engineering Mathematics

248
APPLIED MATHEMATICS I MODULE

ASSIGNMENT -I

1. a) Find a unit vector that is orthogonal to both


b) Find the area of a parallelogram that has
as adjacent sides.
2. Show that

a. If
b. If
3
c. If the cosine of the angle between the vectors 2i + kj and ki + j is then k = 1 or k = 2
√10

3. Let x and y be unit vectors and the angle between them is then find

4. Solve the following system of linear equations.

5. Find the adjoint of

6. Construct two matrices A and B such that the product AB, BA are defined and
AB BA.

249
APPLIED MATHEMATICS I MODULE

ASSIGNMENT -II

1. Evaluate each of the following limits, if it exists.

a. b.

c. d.

2. Show that the equation has atleast one solution.


3. Find the derivative of each of the following functions.

a. c.

b. d. .

4. Find an equation of the tangent line of the gragh of at the point

.
5. Find such that the cubic polynomial function

satisfies the indicated conditions:

Relative maximum:
Re;ative minimum:
Inflection poin:
6. Evaluate the following indefinite integrals

a) c)

b) d)

250

You might also like